You are on page 1of 183

Mathematics Contests

The Australian Scene 2022


Published by

AM T PU BLISHIN G

Australian Maths Trust


170 Haydon Drive
Bruce ACT 2617
AUSTRALIA
Telephone: +61 2 6201 5136
www.amt.edu.au

Copyright © 2023 Australian Mathematics Trust


AMTT Limited ACN 083 950 341
Mathematics Contests: The Australian Scene ISSN 1323-6490

Cover image: via Pexels


From the AMT Chief Executive Officer

For the first year since COVID, after two years competing virtually, the Australian International
Mathematical Olympiad (IMO) team was able to travel to Norway for the event.

The Australian 2022 EGMO team competed virtually for the third year but continued to improve
its strong performance at the competition, placing 3rd out of 57 teams with each of the four team
members scoring a Gold medal. This is an outstanding result for our EGMO program, particularly as it
is only in its fifth year of operation.

The 2022 IMO team also continued our strong tradition of achievement, placing 29 out of 104
countries, with one Gold, four Bronze and an Honourable Mention. Grace Yuan was also awarded the
Maryam Mirzakhani Award for Oceania, the second time an Australian has won this award in two years.

These results are a testament to the incredible work of our students, the Australian Mathematical
Olympiad Committee (AMOC), our Director of Training, our Team Leaders and Deputies, our AMOC
State Directors, the Senior Problems Committee and the dozens of other volunteers and staff who
work to support our training program.

On behalf of the Australian Maths Trust, I would like to thank in particular:


Dr Norman Do, Chair, Australian Mathematical Olympiad Committee (AMOC)
Dr Angelo Di Pasquale, AMOC Director of Training and International Mathematical Olympiad
(IMO) Team Leader
Mr Sampson Wong, IMO Deputy Team Leader
Ms Thanom Shaw, EGMO Team Leader, Deputy Chair AMOC
Dr Sally Tsang, EGMO Deputy Team Leader
Dr Ivan Guo, Chair, AMOC Senior Problems Committee
Dr Kevin McAvaney, Chair, Challenge, Enrichment and AIMO Committees
Mr Mike Clapper, AMT Mathematician
Dr Chris Wetherell, AMT Senior Mathematician
Ms Louise Bilston, AMT Director, Corporate Services
Mr Ben Kirk, AMT Director, Olympiads
Members of the AMTT Ltd Board and AMOC Committee
AMOC tutors, mentors, volunteers and ex-Olympians

Nathan Ford
Chief Executive Officer
Australian Maths Trust
November 2022

Mathematics Contests The Australian Scene 2022 | From the AMT Chief Executive Officer | i
Contents

From the AMT Chief Executive Officer � � � � � � � � � � � � � � � � � � � � � � � � � � � � � � � � � � � � � � � � � � � � � � � � � � � � � � � � � � � � � � � � � � � � � � � � � � � � � � � � � � � � � � � � � � � � � � ������������ i

MATHS FOR YOUNG AUSTRALIANS


About the MYA Program� � � � � � � � � � � � � � � � � � � � � � � � � � � � � � � � � � � � � � � � � � � � � � � � � � � � � � � � � � � � � � � � � � � � � � � � � � � � � � � � � � � � � � � � � � � � � � � � � � � � � � � � � � � � � � � � � � � � � ����������� 1
Membership of MYA Committees� � � � � � � � � � � � � � � � � � � � � � � � � � � � � � � � � � � � � � � � � � � � � � � � � � � � � � � � � � � � � � � � � � � � � � � � � � � � � � � � � � � � � � � � � � � � � � � � � � � � � � � ����������� 3
2022 Maths Challenge � � � � � � � � � � � � � � � � � � � � � � � � � � � � � � � � � � � � � � � � � � � � � � � � � � � � � � � � � � � � � � � � � � � � � � � � � � � � � � � � � � � � � � � � � � � � � � � � � � � � � � � � � � � � � � � � � � � � � � � � � ����������� 5
Middle Primary Problems� � � � � � � � � � � � � � � � � � � � � � � � � � � � � � � � � � � � � � � � � � � � � � � � � � � � � � � � � � � � � � � � � � � � � � � � � � � � � � � � � � � � � � � � � � � � � � � � � � � � � � � � � � � � � � � � ����������� 5
Upper Primary Problems� � � � � � � � � � � � � � � � � � � � � � � � � � � � � � � � � � � � � � � � � � � � � � � � � � � � � � � � � � � � � � � � � � � � � � � � � � � � � � � � � � � � � � � � � � � � � � � � � � � � � � � � � � � � � � � � � ����������� 7
Junior Problems� � � � � � � � � � � � � � � � � � � � � � � � � � � � � � � � � � � � � � � � � � � � � � � � � � � � � � � � � � � � � � � � � � � � � � � � � � � � � � � � � � � � � � � � � � � � � � � � � � � � � � � � � � � � � � � � � � � � � � � � � � � � � � � ��������� 10
Intermediate Problems� � � � � � � � � � � � � � � � � � � � � � � � � � � � � � � � � � � � � � � � � � � � � � � � � � � � � � � � � � � � � � � � � � � � � � � � � � � � � � � � � � � � � � � � � � � � � � � � � � � � � � � � � � � � � � � � � � � � ��������� 15
Middle Primary Solutions � � � � � � � � � � � � � � � � � � � � � � � � � � � � � � � � � � � � � � � � � � � � � � � � � � � � � � � � � � � � � � � � � � � � � � � � � � � � � � � � � � � � � � � � � � � � � � � � � � � � � � � � � � � � � � � � ��������� 19
Upper Primary Solutions � � � � � � � � � � � � � � � � � � � � � � � � � � � � � � � � � � � � � � � � � � � � � � � � � � � � � � � � � � � � � � � � � � � � � � � � � � � � � � � � � � � � � � � � � � � � � � � � � � � � � � � � � � � � � � � � � ��������� 24
Junior Solutions� � � � � � � � � � � � � � � � � � � � � � � � � � � � � � � � � � � � � � � � � � � � � � � � � � � � � � � � � � � � � � � � � � � � � � � � � � � � � � � � � � � � � � � � � � � � � � � � � � � � � � � � � � � � � � � � � � � � � � � � � � � � � � � ��������� 29
Intermediate Solutions � � � � � � � � � � � � � � � � � � � � � � � � � � � � � � � � � � � � � � � � � � � � � � � � � � � � � � � � � � � � � � � � � � � � � � � � � � � � � � � � � � � � � � � � � � � � � � � � � � � � � � � � � � � � � � � � � � � � ��������� 40
Middle Primary Statistics � � � � � � � � � � � � � � � � � � � � � � � � � � � � � � � � � � � � � � � � � � � � � � � � � � � � � � � � � � � � � � � � � � � � � � � � � � � � � � � � � � � � � � � � � � � � � � � � � � � � � � � � � � � � � � � � ��������� 47
Upper Primary Statistics � � � � � � � � � � � � � � � � � � � � � � � � � � � � � � � � � � � � � � � � � � � � � � � � � � � � � � � � � � � � � � � � � � � � � � � � � � � � � � � � � � � � � � � � � � � � � � � � � � � � � � � � � � � � � � � � � ��������� 48
Junior Statistics � � � � � � � � � � � � � � � � � � � � � � � � � � � � � � � � � � � � � � � � � � � � � � � � � � � � � � � � � � � � � � � � � � � � � � � � � � � � � � � � � � � � � � � � � � � � � � � � � � � � � � � � � � � � � � � � � � � � � � � � � � � � � � � ��������� 49
Intermediate Statistics � � � � � � � � � � � � � � � � � � � � � � � � � � � � � � � � � � � � � � � � � � � � � � � � � � � � � � � � � � � � � � � � � � � � � � � � � � � � � � � � � � � � � � � � � � � � � � � � � � � � � � � � � � � � � � � � � � � � ��������� 50
2022 Australian Intermediate Mathematics Olympiad� � � � � � � � � � � � � � � � � � � � � � � � � � � � � � � � � � � � � � � � � � � � � � � � � � � � � � � � � � � � � � � � � � � � � � ��������� 51
Solutions � � � � � � � � � � � � � � � � � � � � � � � � � � � � � � � � � � � � � � � � � � � � � � � � � � � � � � � � � � � � � � � � � � � � � � � � � � � � � � � � � � � � � � � � � � � � � � � � � � � � � � � � � � � � � � � � � � � � � � � � � � � � � � � � � � � � � � � � � � ��������� 53
Statistics� � � � � � � � � � � � � � � � � � � � � � � � � � � � � � � � � � � � � � � � � � � � � � � � � � � � � � � � � � � � � � � � � � � � � � � � � � � � � � � � � � � � � � � � � � � � � � � � � � � � � � � � � � � � � � � � � � � � � � � � � � � � � � � � � � � � � � � � � � � ��������� 66

AUSTRALIAN MATHEMATICAL OLYMPIAD TRAINING PROGRAM


About the Program � � � � � � � � � � � � � � � � � � � � � � � � � � � � � � � � � � � � � � � � � � � � � � � � � � � � � � � � � � � � � � � � � � � � � � � � � � � � � � � � � � � � � � � � � � � � � � � � � � � � � � � � � � � � � � � � � � � � � � � � � � � � � ��������� 68
The Australian Mathematical Olympiad Committee� � � � � � � � � � � � � � � � � � � � � � � � � � � � � � � � � � � � � � � � � � � � � � � � � � � � � � � � � � � � � � � � � � � � � � ��������� 68
The AMOC Senior Problems Committee � � � � � � � � � � � � � � � � � � � � � � � � � � � � � � � � � � � � � � � � � � � � � � � � � � � � � � � � � � � � � � � � � � � � � � � � � � � � � � � � � � � � � � � ��������� 69
Activities of the Australian Mathematical Olympiad Training Program � � � � � � � � � � � � � � � � � � � � � � � � � � � � � � � � � � � � � � ��������� 69
Timetable for Selection of the Teams to the 2022 EGMO and IMO� � � � � � � � � � � � � � � � � � � � � � � � � � � � � � � � � � � � � � � � � � � � � � � � � � ��������� 71
Australia's Achievements at EGMO � � � � � � � � � � � � � � � � � � � � � � � � � � � � � � � � � � � � � � � � � � � � � � � � � � � � � � � � � � � � � � � � � � � � � � � � � � � � � � � � � � � � � � � � � � � � � � � � ��������� 72
Australia's Achievements at IMO � � � � � � � � � � � � � � � � � � � � � � � � � � � � � � � � � � � � � � � � � � � � � � � � � � � � � � � � � � � � � � � � � � � � � � � � � � � � � � � � � � � � � � � � � � � � � � � � � � � � ��������� 72
Support for the Australian Mathematical Olympiad Training Program � � � � � � � � � � � � � � � � � � � � � � � � � � � � � � � � � � � � � � � � � � � � ��������� 74
2021 AMOC School of Excellence � � � � � � � � � � � � � � � � � � � � � � � � � � � � � � � � � � � � � � � � � � � � � � � � � � � � � � � � � � � � � � � � � � � � � � � � � � � � � � � � � � � � � � � � � � � � � � � � � � � � � � ��������� 75
Participants at the 2021 AMOC School of Excellence� � � � � � � � � � � � � � � � � � � � � � � � � � � � � � � � � � � � � � � � � � � � � � � � � � � � � � � � � � � � � � � � � ��������� 76
2022 Australian Mathematical Olympiad � � � � � � � � � � � � � � � � � � � � � � � � � � � � � � � � � � � � � � � � � � � � � � � � � � � � � � � � � � � � � � � � � � � � � � � � � � � � � � � � � � � � � � � � � � � ��������� 79
Solutions � � � � � � � � � � � � � � � � � � � � � � � � � � � � � � � � � � � � � � � � � � � � � � � � � � � � � � � � � � � � � � � � � � � � � � � � � � � � � � � � � � � � � � � � � � � � � � � � � � � � � � � � � � � � � � � � � � � � � � � � � � � � � � � � � � � � � � � � � � ��������� 81
Statistics� � � � � � � � � � � � � � � � � � � � � � � � � � � � � � � � � � � � � � � � � � � � � � � � � � � � � � � � � � � � � � � � � � � � � � � � � � � � � � � � � � � � � � � � � � � � � � � � � � � � � � � � � � � � � � � � � � � � � � � � � � � � � � � � � � � � � � � � � � � ��������� 96
XXXIV Asian Pacific Mathematics Olympiad� � � � � � � � � � � � � � � � � � � � � � � � � � � � � � � � � � � � � � � � � � � � � � � � � � � � � � � � � � � � � � � � � � � � � � � � � � � � � � � � � � � � � ��������� 97
Solutions � � � � � � � � � � � � � � � � � � � � � � � � � � � � � � � � � � � � � � � � � � � � � � � � � � � � � � � � � � � � � � � � � � � � � � � � � � � � � � � � � � � � � � � � � � � � � � � � � � � � � � � � � � � � � � � � � � � � � � � � � � � � � � � � � � � � � � � � � � ��������� 98
2022 APMO Awards � � � � � � � � � � � � � � � � � � � � � � � � � � � � � � � � � � � � � � � � � � � � � � � � � � � � � � � � � � � � � � � � � � � � � � � � � � � � � � � � � � � � � � � � � � � � � � � � � � � � � � � � � � � � � � � � � � � � � � � � ������� 106

Mathematics Contests The Australian Scene 2022 | Contents | ii


2022 AMOC Selection School� � � � � � � � � � � � � � � � � � � � � � � � � � � � � � � � � � � � � � � � � � � � � � � � � � � � � � � � � � � � � � � � � � � � � � � � � � � � � � � � � � � � � � � � � � � � � � � � � � � � � � � � � � � � � ������� 107
2022 Australian EGMO team� � � � � � � � � � � � � � � � � � � � � � � � � � � � � � � � � � � � � � � � � � � � � � � � � � � � � � � � � � � � � � � � � � � � � � � � � � � � � � � � � � � � � � � � � � � � � � � � � � � � � � � � � � � ������� 108
2022 Australian IMO team� � � � � � � � � � � � � � � � � � � � � � � � � � � � � � � � � � � � � � � � � � � � � � � � � � � � � � � � � � � � � � � � � � � � � � � � � � � � � � � � � � � � � � � � � � � � � � � � � � � � � � � � � � � � � � � ������� 108
Participants at the 2022 AMOC Selection School� � � � � � � � � � � � � � � � � � � � � � � � � � � � � � � � � � � � � � � � � � � � � � � � � � � � � � � � � � � � � � � � � � � � � � � � � ������� 109
The 11th European Girls' Mathematical Olympiad � � � � � � � � � � � � � � � � � � � � � � � � � � � � � � � � � � � � � � � � � � � � � � � � � � � � � � � � � � � � � � � � � � � � � � � � � � � � ������� 111
Paper ��������������� � � � � � � � � � � � � � � � � � � � � � � � � � � � � � � � � � � � � � � � � � � � � � � � � � � � � � � � � � � � � � � � � � � � � � � � � � � � � � � � � � � � � � � � � � � � � � � � � � � � � � � � � � � � � � � � � � � � � � � � � � � � � � � � � � ������� 117
Solutions � � � � � � � � � � � � � � � � � � � � � � � � � � � � � � � � � � � � � � � � � � � � � � � � � � � � � � � � � � � � � � � � � � � � � � � � � � � � � � � � � � � � � � � � � � � � � � � � � � � � � � � � � � � � � � � � � � � � � � � � � � � � � � � � � � � � � � � � � � ������� 119
2022 Mentor Program � � � � � � � � � � � � � � � � � � � � � � � � � � � � � � � � � � � � � � � � � � � � � � � � � � � � � � � � � � � � � � � � � � � � � � � � � � � � � � � � � � � � � � � � � � � � � � � � � � � � � � � � � � � � � � � � � � � � � � � � � ������� 126
2022 IMO Team Preparation School � � � � � � � � � � � � � � � � � � � � � � � � � � � � � � � � � � � � � � � � � � � � � � � � � � � � � � � � � � � � � � � � � � � � � � � � � � � � � � � � � � � � � � � � � � � � � � � � � � � ������� 127
Mathematics Ashes � � � � � � � � � � � � � � � � � � � � � � � � � � � � � � � � � � � � � � � � � � � � � � � � � � � � � � � � � � � � � � � � � � � � � � � � � � � � � � � � � � � � � � � � � � � � � � � � � � � � � � � � � � � � � � � � � � � � � � � � � ������� 128
Results������������� � � � � � � � � � � � � � � � � � � � � � � � � � � � � � � � � � � � � � � � � � � � � � � � � � � � � � � � � � � � � � � � � � � � � � � � � � � � � � � � � � � � � � � � � � � � � � � � � � � � � � � � � � � � � � � � � � � � � � � � � � � � � � � � � � ������� 129
The 63rd International Mathematical Olympiad� � � � � � � � � � � � � � � � � � � � � � � � � � � � � � � � � � � � � � � � � � � � � � � � � � � � � � � � � � � � � � � � � � � � � � � � � � � � � � � � � � ������� 130
Some IMO Country Totals � � � � � � � � � � � � � � � � � � � � � � � � � � � � � � � � � � � � � � � � � � � � � � � � � � � � � � � � � � � � � � � � � � � � � � � � � � � � � � � � � � � � � � � � � � � � � � � � � � � � � � � � � � � � � � � ������� 134
Paper ��������������� � � � � � � � � � � � � � � � � � � � � � � � � � � � � � � � � � � � � � � � � � � � � � � � � � � � � � � � � � � � � � � � � � � � � � � � � � � � � � � � � � � � � � � � � � � � � � � � � � � � � � � � � � � � � � � � � � � � � � � � � � � � � � � � � � ������� 138
Solutions � � � � � � � � � � � � � � � � � � � � � � � � � � � � � � � � � � � � � � � � � � � � � � � � � � � � � � � � � � � � � � � � � � � � � � � � � � � � � � � � � � � � � � � � � � � � � � � � � � � � � � � � � � � � � � � � � � � � � � � � � � � � � � � � � � � � � � � � � � ������� 140
2022 AMOC Senior Contest� � � � � � � � � � � � � � � � � � � � � � � � � � � � � � � � � � � � � � � � � � � � � � � � � � � � � � � � � � � � � � � � � � � � � � � � � � � � � � � � � � � � � � � � � � � � � � � � � � � � � � � � � � � � � � � � ������� 154
Solutions ��������� � � � � � � � � � � � � � � � � � � � � � � � � � � � � � � � � � � � � � � � � � � � � � � � � � � � � � � � � � � � � � � � � � � � � � � � � � � � � � � � � � � � � � � � � � � � � � � � � � � � � � � � � � � � � � � � � � � � � � � � � � � � � � � � � � ������� 155
Results������������� � � � � � � � � � � � � � � � � � � � � � � � � � � � � � � � � � � � � � � � � � � � � � � � � � � � � � � � � � � � � � � � � � � � � � � � � � � � � � � � � � � � � � � � � � � � � � � � � � � � � � � � � � � � � � � � � � � � � � � � � � � � � � � � � � ������� 163
Statistics���������� � � � � � � � � � � � � � � � � � � � � � � � � � � � � � � � � � � � � � � � � � � � � � � � � � � � � � � � � � � � � � � � � � � � � � � � � � � � � � � � � � � � � � � � � � � � � � � � � � � � � � � � � � � � � � � � � � � � � � � � � � � � � � � � � � ������� 166
Origin of Some Questions � � � � � � � � � � � � � � � � � � � � � � � � � � � � � � � � � � � � � � � � � � � � � � � � � � � � � � � � � � � � � � � � � � � � � � � � � � � � � � � � � � � � � � � � � � � � � � � � � � � � � � � � � � � � � � � � � � ������� 167

HONOUR ROLL
Honour Roll 1979–2022� � � � � � � � � � � � � � � � � � � � � � � � � � � � � � � � � � � � � � � � � � � � � � � � � � � � � � � � � � � � � � � � � � � � � � � � � � � � � � � � � � � � � � � � � � � � � � � � � � � � � � � � � � � � � � � � � � � � � � � ������� 169
Interim Committee 1979–1980� � � � � � � � � � � � � � � � � � � � � � � � � � � � � � � � � � � � � � � � � � � � � � � � � � � � � � � � � � � � � � � � � � � � � � � � � � � � � � � � � � � � � � � � � � � � � � � � � � � � � � � ������� 169
Australian Mathematical Olympiad Committee � � � � � � � � � � � � � � � � � � � � � � � � � � � � � � � � � � � � � � � � � � � � � � � � � � � � � � � � � � � � � � � � � � � � � � � � � � � � � ������� 169
Maths for Young Australians � � � � � � � � � � � � � � � � � � � � � � � � � � � � � � � � � � � � � � � � � � � � � � � � � � � � � � � � � � � � � � � � � � � � � � � � � � � � � � � � � � � � � � � � � � � � � � � � � � � � � � � � � � � ������� 172
Maths Enrichment Development � � � � � � � � � � � � � � � � � � � � � � � � � � � � � � � � � � � � � � � � � � � � � � � � � � � � � � � � � � � � � � � � � � � � � � � � � � � � � � � � � � � � � � � � � � � � � � � � � � � � ������� 174
Australian Intermediate Mathematics Olympiad Committee� � � � � � � � � � � � � � � � � � � � � � � � � � � � � � � � � � � � � � � � � � � � � � � � � � � � � � � � ������� 176
AMOC Senior Problems Committee� � � � � � � � � � � � � � � � � � � � � � � � � � � � � � � � � � � � � � � � � � � � � � � � � � � � � � � � � � � � � � � � � � � � � � � � � � � � � � � � � � � � � � � � � � � � � � � ������� 176
Mathematics School of Excellence � � � � � � � � � � � � � � � � � � � � � � � � � � � � � � � � � � � � � � � � � � � � � � � � � � � � � � � � � � � � � � � � � � � � � � � � � � � � � � � � � � � � � � � � � � � � � � � � � ������� 177
International Mathematical Olympiad Selection School� � � � � � � � � � � � � � � � � � � � � � � � � � � � � � � � � � � � � � � � � � � � � � � � � � � � � � � � � � � � � � � ������� 177

Mathematics Contests The Australian Scene 2022 | Contents | iii


Maths for Young Australians
About the MYA Program

The Maths for Young Australians program (MYA or formerly MCYA) started on a national scale in
1992. It was set up to cater for the needs of the top 10 percent of secondary students in years 7–10,
especially in country schools and schools where the number of students may be quite small. Teachers
with a handful of talented students spread over a number of classes and working in isolation can
find it very difficult to cater for the needs of these students. The program provides materials and an
organised structure designed to enable teachers to help talented students reach their potential. At
the same time, teachers in larger schools, where there are more of these students, are able to use the
materials to better assist the students in their care.

The aims of the program include:

• encouraging and fostering

- a greater interest in and awareness of the power of mathematics


- a desire to succeed in solving interesting mathematical problems
- the discovery of the joy of solving problems in mathematics

• identifying talented young Australians, recognising their achievements nationally and providing
support that will enable them to reach their own levels of excellence
• providing teachers with

- interesting and accessible problems and solutions as well as detailed and motivating teaching
discussion and extension materials
- comprehensive Australia-wide statistics of students' achievements in the Maths Challenge.

There are three independent stages:

• Maths Challenge (up to four weeks during the period March–June)


• Maths Enrichment (12–16 weeks between April–October)
• Australian Intermediate Mathematics Olympiad (September).

Maths Challenge
The Challenge consists of four levels. Middle Primary (years 3–4) and Upper Primary (years 5–6)
present students with four problems each to be attempted over three to four weeks. Students are
allowed to work on the problems in groups of up to three participants, but each must write their
solutions individually. The Junior (years 7–8) and Intermediate (Years 9–10) levels present students
with six problems to be attempted over three to four weeks. Students are allowed to work on the
problems with a partner but each must write their solutions individually.

There were 10 177 submissions (1400 Middle Primary, 2604 Upper Primary, 4199 Junior, 1974
Intermediate) for the Maths Challenge in 2022. The 2022 problems and solutions, together with some
statistics, appear later in this book.

Maths Enrichment
This is a program running from April to October, which consists of seven different parallel stages of
comprehensive student and teacher support notes. Each student participates in only one of these
stages. The materials for all stages are designed to be a systematic structured course over a flexible
12–16 week period. This enables schools to timetable the program at convenient times during their
school year.

Maths Enrichment is completely independent of Maths Challenge; however, they have the common
feature of providing challenging mathematics problems for students, as well as accessible support
materials for teachers.

Mathematics Contests The Australian Scene 2022 | About the MYA Program | 1
Ramanujan (years 4–5) includes fast arithmetic, counting techniques, polyominoes, special numbers
such as squares, primes, and triangular numbers, and specific problem-solving techniques (draw a
diagram, make a list, guess and check). There were 374 entries in 2022.

Newton (years 5–6) includes estimation, colouring problems, polyhedra, arithmetic in other bases,
divisibility, problem-solving techniques and patterns. There were 852 entries in 2022.

Dirichlet (years 6–7) includes mathematics concerned with tessellations, arithmetic in other bases,
time/distance/speed, patterns, recurring decimals and specific problem-solving techniques. There
were 902 entries in 2022.

Euler (years 7–8) includes primes and composites, least common multiples, highest common factors,
arithmetic sequences, figurate numbers, congruence, properties of angles and pigeonhole principle.
There were 2040 entries in 2022.

Gauss (years 8–9) includes parallels, similarity, Pythagoras' theorem, using spreadsheets, Diophantine
equations, counting techniques and congruence. Gauss builds on the Euler program. There were 1213
entries in 2022.

Noether (top 10% years 9–10) includes expansion and factorisation, inequalities, sequences and series,
number bases, methods of proof, congruence, circles and tangents. There were 637 entries in 2022.

Pólya (top 10% year 10) includes angle chasing, combinatorics, number theory, graph theory and
symmetric polynomials. There were 200 entries in 2022.

Australian Intermediate Mathematics Olympiad


This four-hour competition for students up to year 10 offers a range of challenging and interesting
questions. It is suitable for students who have performed well in the AMC (Distinction and above), and
is designed as an endpoint for students who have completed the Gauss or Noether stage. There were
2255 entries for 2022 and 4 perfect scores.

Mathematics Contests The Australian Scene 2022 | About the MYA Program | 2
Membership of MYA Committees

2022 Challenge Committee


Chair:Dr Kevin McAvaney, Victoria

Members:
Mrs Barbara Denney, New South Wales
Mr Andy Edwards, Queensland
Mrs Lorraine Mottershead, New South Wales
Ms Anna Nakos, South Australia
Prof. Mike Newman, Australian Capital Territory
Dr Ian Roberts, Northern Territory
Ms Thanom Shaw, New South Wales
Mrs Katrina Sims, New South Wales
Dr Steve Thornton, Australian Capital Territory
Ms Giovanna Vardaro, Victoria
Dr Chris Wetherell, Australian Capital Territory

2022 Moderators
Mr P Swain, Victoria
Ms K Trudgian, Queensland
Ms K McAsey, Victoria
Ms C Smith, Queensland
Dr D Wells, USA
Dr F Rose Victoria
Mr J Rizos, Victoria
Dr S Blasberg, USA
Mr M Bamman, South Australia
Mr L Katz, Victoria
Mr T Love, Western Australia
Mr L Bao, Victoria
Mr J Dowsey, Victoria
Ms T McNamara, Victoria
Ms C Stanley, Queensland
Mr M O’Connor, Victoria
Mr S Ewington, New South Wales
Ms R Humberstone, Victoria
Mr P Christensen, Queensland
Mr J Watts, New South Wales
Prof. M Clapper, Victoria
Dr E Casling, Australian Capital Territory
Mr G Meiklejohn, Queensland
Dr P Swedosh, Victoria
Ms J Hartnett Queensland
Mr S Gardiner, New South Wales
Mr A Peck, Tasmania
Ms R Stone, South Australia
Dr M Sun, New South Wales
Dr Zltan Bacskai, Australian Capital Territory
Mr T Ogawa, New South Wales

Mathematics Contests The Australian Scene 2022 | Membership of MYA Committees | 3


Dr N Hoffman, Western Australia
Mr B Akhurst, New South Wales
Mr B Akhurst, New South Wales

2022 Australian Intermediate Mathematics Olympiad Committee


Dr K McAvaney, Victoria (Chair)
Mr M Clapper, Australian Maths Trust, Australian Capital Territory
Dr M Evans, International Centre of Excellence for Education in Mathematics, Victoria
Mr B Henry, Victoria
Dr D Mathews, Monash University, Victoria
Dr D Treeby, Presbyterian Ladies' College, Victoria

Enrichment

Editors
Mr M Clapper, Mathematician, Australian Maths Trust, Victoria
Dr K McAvaney, Chair of Maths for Young Australians, Victoria
Dr D Treeby, Mathematician, Australian Maths Trust, Victoria
Dr C Wetherell, Senior Mathematician, Australian Maths Trust, Australian Capital Territory

Mathematics Contests The Australian Scene 2022 | Membership of MYA Committees | 4


2022 Maths Challenge
Maths Challenge
Middle Primary
2022Problems
Challenge Problems - Middle Primary
Students may work on each of these four problems in groups of up to three, but must write their solutions individually.

MP1 Outer
MP1 OuterSums
Sums
Mali has a large number of wooden triangles. The sides of each triangle are labelled with 1, 2, 3 on both the front
and back faces. So the triangles can be flipped over and used with either face up.

1 2 2 1
3 3

front back

Mali joins triangles by bringing together edges that have matching numbers. For example, Mali can join three triangles
as shown on the left, but the arrangement on the right is not allowed.

2 2
1 3 1 3
2 1 3 2 2 1 2 1
3 1 3 3

allowed not allowed (2 ̸= 3)

Mali then adds the numbers on the outside edges of the arrangement. This is called its outer sum. For example, the
outer sum of the allowed arrangement above is 2 + 2 + 2 + 1 + 3 = 10.

a Join four triangles to make an outer sum of 10.


b Join five triangles to make an outer sum of 10.
c Join six triangles to make an outer sum of 10.
d Join six triangles to make an outer sum of 8.

MP2 Palindromes
MP2 Palindromes
In this problem, a palindrome is a whole number greater than 10 that reads the same backwards as it does forwards.
For example, 66, 484, 1001, 44044 are all palindromes.

a Find the 10 palindromes between 2000 and 3000.


b The palindrome 121 can be written as the sum of three different palindromes. For example, 121 = 11 + 22 + 88.
Find four other triples of different palindromes that total 121.
c What is the 110th palindrome?

MP3 Quad
MP3 QuadCuts
Cuts

On the left is a convex quadrilateral. On the right is a concave quadrilateral (it contains a reflex angle). If a line is
drawn across either quadrilateral, then it divides the quadrilateral into two or more parts, depending on the position
of the line. For instance, in the example shown, the convex quadrilateral is divided into a triangle (T) and another
quadrilateral (Q), and the concave quadrilateral is divided into two triangles.

Mathematics Contests The Australian Scene 2022 | Maths Challenge Middle Primary Problems | 5
1
T
T T
Q

convex concave

a Draw a diagram showing how a single line on the convex quadrilateral above can divide it into two triangles, and
another diagram showing how a single line on the convex quadrilateral above can divide it into two quadrilaterals.
b Draw a diagram showing how a single line on the convex quadrilateral above can divide it into a pentagon and one
other shape.
c Draw four separate diagrams showing how a single line on the concave quadrilateral above can divide it into:
• two triangles different from the example above
• a triangle and a quadrilateral
• two quadrilaterals
• a triangle and a pentagon (P).
d Draw a diagram showing how a single line on the concave quadrilateral above can divide it into three triangles, and
another diagram showing how a single line on the concave quadrilateral above can divide it into two triangles and
a hexagon (H).

MP4 Near-square
MP4 Near-squareRectangles
Rectangles

A near-square rectangle (NSR) is a rectangle that can be drawn on centimetre grid paper and is made up of a whole
number of grid squares. The longer side of an NSR is 1 cm longer than its shorter side. NSRs are named according
to their shorter sides. For example, NSR1 is a 1 cm × 2 cm rectangle with 2 grid squares and NSR2 is a 2 cm × 3 cm
rectangle with 6 grid squares. These are the two smallest NSRs:

3 cm

2 cm
2 cm
1 cm

NSR1 NSR2

Note that NSR1 and NSR2 can be joined to make a 2 cm × 4 cm rectangle:

1 cm 3 cm

NSR1 NSR2
2 cm 2 cm

1 cm 3 cm

a Draw a labelled diagram to show how NSR1, NSR2, and NSR3 can be joined to form another near-square rectangle.
Which near-square rectangle is it and how many grid squares does it contain?
b Draw a labelled diagram to show how NSR1, NSR2, NSR3, and NSR4 can be joined to form a rectangle which is
not a near-square rectangle. What are the dimensions (length and width) of the rectangle formed?
c Draw a labelled diagram to show how NSR2, NSR3, NSR4, NSR5, and NSR6 can be joined to form another
near-square rectangle. Which near-square rectangle is it?
d Explain why no other size rectangle can be formed by joining the five NSRs in Part c.

Mathematics Contests The Australian Scene 2022 | Maths Challenge


2 Middle Primary Problems | 6
Maths Challenge
2022
Upper Primary Challenge Problems - Upper Primary
Problems
Students may work on each of these four problems in groups of up to three, but must write their solutions individually.

UP1 Zipline
UP1 ZiplineNumbers
Numbers

A Zipline number has exactly five digits. The digits are all different and in descending order. Four of the digits are
even and the other is odd. For example,
• 86540 is a Zipline number
• 86450 is not a Zipline number because the digits are not in descending order
• 86530 is not a Zipline number because it has two odd digits.

a What are the largest and smallest Zipline numbers?


b Find all the odd Zipline numbers and explain why there are no others.
c Find two pairs of Zipline numbers that have a sum of 186282.
d Find the smallest number greater than 200000 that is the sum of different Zipline numbers.

UP2 Near-square
UP2 Near-squareRectangles
Rectangles

A near-square rectangle (NSR) is a rectangle that can be drawn on centimetre grid paper and is made up of a whole
number of grid squares. The longer side of an NSR is 1 cm longer than its shorter side. NSRs are named according to
their shorter sides. For example, NSR1 is a 1 cm × 2 cm rectangle and NSR2 is a 2 cm × 3 cm rectangle. These are the
two smallest NSRs.
3 cm

2 cm
2 cm
1 cm

NSR1 NSR2

Note that NSR1 and NSR2 can be joined to make a 2 cm × 4 cm rectangle:

1 cm 3 cm

NSR1 NSR2
2 cm 2 cm

1 cm 3 cm

a Draw a labelled diagram to show how NSR1, NSR2, NSR3, and NSR4 can be joined to form a rectangle which is
not a near-square rectangle. What are the dimensions (length and width) of the rectangle formed?
b Draw a labelled diagram to show how NSR2, NSR3, NSR4, NSR5, and NSR6 can be joined to form another
near-square rectangle. Which near-square rectangle is it?
c Explain why no other size rectangle can be formed by joining the five near-square rectangles in Part b.
d NSR1, NSR2, and NSR3 can be joined to form another NSR. Find the next longest sequence of NSRs starting with
NSR1 and not skipping any, that can be joined to form another NSR.

Mathematics Contests The Australian Scene 2022 | Maths Challenge


3 Upper Primary Problems | 7
UP3 PolyCuts
UP3 Poly Cuts

On the left is a heptagon (it has 7 sides and 7 angles). Drawing one straight line, it can be divided into one octagon,
one quadrilateral and one triangle, as shown on the right.

For each of the following statements, draw a hexagon and a single straight line which divides it into the shapes listed.

a One quadrilateral and one pentagon.


b Three triangles.
c Four triangles.
d Two triangles.

UP4 Mixed-up
UP4 Mixed-upBirthdays
Birthdays

Annie, Basil, Carol, Dylan, and Ebony are the five children in the Day family. Their names are on the birthday board
shown below. Each child has a card with his/her name and four more cards for his/her birthday: two green cards for
the day and two yellow cards for the month.

Day Month

Annie

Basil

Carol

Dylan

Ebony

For example, if a child’s birthday was 17 September (17/09), the row for that child would be:

Name 1 7 0 9

We have the following information.

Mathematics Contests The Australian Scene 2022 | Maths Challenge


4 Upper Primary Problems | 8
• This table shows the number and colour of cards with each digit.

Digit 0 1 2 3 4 5 6 7 8 9
Green cards 1 2 2 0 0 1 1 0 1 2
Yellow cards 5 1 1 0 1 0 0 1 1 0

• The children’s birthdays are in the same order in a calendar year as their names alphabetically.
• Basil’s birthday is 33 days after Annie’s.
• Ebony’s birthday is exactly three weeks after Dylan’s.

a Find the birthday month for each child.


b Find the second green digit for each of Annie and Basil’s birthdays.
c Find the second green digit for each of Dylan and Ebony’s birthdays.
d Complete the birthday board.

Mathematics Contests The Australian Scene 2022 | Maths Challenge Upper Primary Problems | 9
5
Maths Challenge
Junior Problems 2022 Challenge Problems - Junior
Students may work on each of these six problems with a partner but each must write their solutions individually.

J1 Dropdown
J1 DropdownNumbers
Numbers

All digits in a dropdown number are different and one of its digits is the average of all its digits. It has at least two
digits. For example, 5021 is a dropdown number, but neither 4389 nor 6033 is a dropdown number.

a Find the smallest dropdown number.


b What are the smallest and largest 4-digit dropdown numbers?
c How many 3-digit dropdown numbers are there?
d Is it possible for a pair of consecutive numbers to both be dropdown numbers? If not, explain why not. If it is
possible, find the smallest such pair.

J2 Phancy
J2 PhancyPhotos
Photos

Bryn has a stack of photos measuring 12 cm wide and 16 cm high. He wants to display them on a large rectangular
poster in such a way that all photos are equally spaced on each side, including at the edges of the poster, as illustrated.

a Bryn wants to display 9 photos (with longer side vertical) in 3 rows of 3 photos each with 2 cm spacing around each
photo. What are the dimensions of the poster?
b Bryn wants to display 24 photos (with longer side vertical) with 2 cm spacing around each photo. He wants more
columns than rows of photos. Find the smallest poster area that is needed.
c Bryn wants to display 18 photos in 3 rows of 6 photos each with 3 cm spacing around each photo. The photos
may all be displayed in portrait orientation (longer side vertical) or in landscape (longer side horizontal). Which
orientation uses the lesser poster area?
d Bryn wants to display 24 photos (with longer side vertical) in 6 rows of 4 photos each with equal spacing around
each photo. He has a poster that is 62 cm wide and 120 cm high. He wants to use the full width of the poster but
is prepared to trim some of its height to achieve the equal spacing. How much of the poster does he need to trim?

Mathematics Contests The Australian Scene 2022 | Maths Challenge


6 Junior Problems | 10
J3 Unit
J3 UnitTriangles
Triangles

One way to measure the area of a shape is to count, or work out, the number of unit squares the shape covers when
placed on a square grid. For example, the area of the shaded hexagon below is the sum of 2 unit squares and 4 half-unit
squares, a total of 4 unit squares.

A different way to measure the area of a shape is to count, or work out, the number of triangles the shape covers
when placed on an isometric grid. The equilateral triangle shown below has an area of 4 unit triangles (ut), while the
regular hexagon has an area of 6 ut, and the isosceles triangle has an area of 3 ut, consisting of one whole triangle and
4 half triangles.

a Calculate the areas of the following shapes in unit triangles.

b Calculate the area of the following triangle in unit triangles.

Mathematics Contests The Australian Scene 2022 | Maths Challenge


7 Junior Problems | 11
c Calculate the area of the following triangle in unit triangles.

d Given a parallelogram with its sides drawn along isometric grid lines, a spanning triangle is a triangle ABC with
the following properties:
• A, B, C are grid points on the sides of the parallelogram
• A is a vertex of the parallelogram
• no side of triangle ABC is parallel to a side of the parallelogram.
For the parallelogram shown below, find two non-congruent spanning
triangles, each of which has one-third the area of the parallelogram.

J4 GoGo
J4 GoGo
Tokens are put on some squares of an 8 × 8 grid to make a pattern, such as the one shown in this diagram.

8 • • •
7 • •
6 • • •
5 • •
4 • • • •
3 • •
2 • •
1 • • •
A B C D E F G H

We try to remove the tokens in a sequence of clearing steps. A clearing step consists of choosing a row or a column
which contains only 1 or 2 tokens and removing them. For example, we cannot initially remove all tokens from column
D. But we can first clear row 5 and then, in some later step, remove the two remaining tokens in column D.
If all tokens in a pattern can be removed from the grid using some sequence of clearing steps, then the pattern is called
clearable. The above pattern is clearable because we can remove all tokens using the following sequence of 13 clearing
steps (among other possibilities):
2, E, 8, G, 7, F, D, 5, 6, 3, 4, A, 1.
These 13 steps are represented by arrows numbered 1 to 13 in the following diagram.

Mathematics Contests The Australian Scene 2022 | Maths Challenge


8 Junior Problems | 12
12 7 2 6 4

8 • • • 3
7 • • 5
6 • • • 9
5 • • 8
4 • • • • 11
3 • • 10
2 • • 1
1 • • • 13
A B C D E F G H

a Show that the pattern above can be cleared in 11 steps.


b Explain why it is impossible to clear the pattern above in fewer than 11 steps.
c Explain why the following pattern is not clearable.

8 • • •
7 • •
6 • • •
5
4 • •
3 • • •
2 •
1 • • • •
A B C D E F G H

d What is the smallest number of tokens that can appear in a non-clearable pattern? Explain your answer.

J5 Towers
J5 Towersand
andCities
Cities
Towers and Cities is a game that can be played by one or more players. Each player has a bag of one or more blocks
with which to construct one or more towers on a table to form a ‘city’ of towers. Each block is either kept in the bag
or placed on the table to start a new tower or placed on top of an existing tower. The towers are kept in height order.
Each tower has a value associated with it. Each block on the bottom has value 1, each block on the next level has
value 2, and so on. Unused blocks have value 0. The value of a tower is the sum of the values of all its blocks. The
value of a city is the sum of the values of all its towers.
For example, the city in the following diagram consists of 11 blocks arranged in 4 towers. The values of the towers
are 10, 6, 6, 1, hence the value of the city, denoted CV, is 23.

3 3 3

2 2 2

1 1 1 1

10 6 6 1
CV=23

Mathematics Contests The Australian Scene 2022 | Maths Challenge


9 Junior Problems | 13
a Annaya has 4 blocks in her bag. Draw all possible cities she can make and calculate their values.
b Find a city with value 26 using the least number of blocks possible.
c Dion has 19 blocks in his bag. Find the 5th and 10th largest city values he can produce.
d Shakira has 6 blocks in her bag. Determine which of 14, 16, 18 are possible city values she can produce.

J6 Mixed-up
J6 Mixed-upBirthdays
Birthdays

Annie, Basil, Carol, Dylan, and Ebony are the five children in the Day family. Their names are on the birthday board
shown below. Each child has a card with his/her name and four more cards for his/her birthday: two for the day and
two for the month.

Day Month

Annie

Basil

Carol

Dylan

Ebony

For example, if a child’s birthday was 17 September (17/09), the row for that child would be:

Name 1 7 0 9

We have the following information.


• This table shows the number of cards with each digit.

Digit 0 1 2 3 4 5 6 7 8 9
Cards 6 3 3 0 1 1 1 1 2 2

• The children’s birthdays are in the same order in a calendar year as their names alphabetically.
• No two birthdays are in the same month.
• Ebony’s birthday is exactly three weeks after Dylan’s.
• Annie’s birthday was on the same day of the week as Dylan’s in 2020, but not in 2021.
• Basil’s birthday is 33 days after Annie’s every year.
• Ebony was not born in July.

a In which months were Annie and Basil born?


b In what month was Dylan born?
c Complete all possible birthday boards.

Mathematics Contests The Australian Scene 2022 | Maths Challenge


10 Junior Problems | 14
Maths Challenge
Intermediate Problems
2022 Challenge Problems - Intermediate
Students may work on each of these six problems with a partner but each must write their solutions individually.

I1 DPS
I1 DPSChains
Chains
A Digit Product Subtraction (DPS ) chain starts with any number with two or more digits. The product of its digits is
subtracted from the number to form the next number in the chain. This is repeated until the answer does not change.
For example, starting with the 3-digit number 381 gives

3 × 8 × 1 = 24 381 − 24 = 357
3 × 5 × 7 = 105 357 − 105 = 252
2 × 5 × 2 = 20 252 − 20 = 232
2 × 3 × 2 = 12 232 − 12 = 220
2×2×0=0 220 − 0 = 220.

So 220 is the last number in this DPS chain. The DPS chain can be written as 381 → 357 → 252 → 232 → 220. It
has 5 numbers so we say the chain has length 5.

a Find a DPS chain of length 5 that starts with a 2-digit number.


b Find a DPS chain of length 3 that ends at 40.
c Explain why a DPS chain that starts with a 2-digit number contains only 2-digit numbers.
d Find a DPS chain of length 3 that ends at 307.

I2 Triangle
I2 TriangleFolding
Folding

A square piece of paper has side length 1. Esha cuts it in half along one of its diagonals. She labels the corners of one
of the triangular halves A, B, C as shown in the left diagram below.
Esha folds the triangle so that B is moved to any point B ′ on the side AC, making a triangle B ′ GH as shown shaded
on the right.

C C

H
B′

B A B G A

a Find the area of △B ′ GH if B ′ is at C.


b Find the area of △B ′ GH if B ′ is at A.
c Find the length of AG if HB ′ is perpendicular to AC.
d Find the area of △B ′ GH if HB ′ is perpendicular to AC.

Mathematics Contests The Australian Scene 2022 | Maths Challenge Intermediate


11 Problems | 15
I3 GoGo
I3 GoGo
Tokens are put on some squares of an 8 × 8 grid to make a pattern, such as the one shown in this diagram.

8 • • •
7 • •
6 • • •
5 • •
4 • • • •
3 • •
2 • •
1 • • •
A B C D E F G H

We try to remove the tokens in a sequence of clearing steps. A clearing step consists of choosing a row or a column
which contains only 1 or 2 tokens and removing them. For example, we cannot initially remove all tokens from column
D. But we can first clear row 5 and then, in some later step, remove the two remaining tokens in column D.
If all tokens in a pattern can be removed from the grid using some sequence of allowable steps, then the pattern is
called clearable. The above pattern is clearable because we can remove all tokens using the following sequence of 13
allowable steps (among other possibilities):

2, E, 8, G, 7, F, D, 5, 6, 3, 4, A, 1.

These 13 steps are represented by arrows numbered 1 to 13 in the following diagram.

12 7 2 6 4

8 • • • 3
7 • • 5
6 • • • 9
5 • • 8
4 • • • • 11
3 • • 10
2 • • 1
1 • • • 13
A B C D E F G H

a Show that the pattern above can be cleared in 11 steps.


b Explain why it is impossible to clear the pattern above in fewer than 11 steps.
c Explain why the following pattern is not clearable.

Mathematics Contests The Australian Scene 2022 | Maths Challenge Intermediate


12 Problems | 16
8 • • •
7 • •
6 • • •
5
4 • •
3 • • •
2 •
1 • • • •
A B C D E F G H

d What is the smallest number of tokens that can appear in a non-clearable pattern? Explain your answer.

I4
SlotSlot
CarsCars
Pip is building model car racing tracks using a large supply of the two types of square plate shown. Ignoring the
tabs and slots where plates are snapped together, both types measure 20 cm × 20 cm. In the first type, the track is a
straight line joining the middles of opposite sides. In the second, the track is a quarter circle joining the middles of
adjacent sides. Plates can be snapped together in any orientation, as long as the track is continuous.

20 cm

20 cm 20 cm

Pip wants to complete the track shown below but, because there is limited space, additional plates can only be put in
the 4 × 4 grid indicated by dotted lines.

a Show how Pip can complete the circuit by adding less than 120 cm of track.
b Show how Pip can complete the circuit by adding more than 270 cm of track.
c Explain why, no matter how the circuit is completed, Pip must place an even number of additional curved plates.
d Explain why, no matter how the circuit is completed, Pip must place an odd number of additional straight plates.

Mathematics Contests The Australian Scene 2022 | Maths Challenge Intermediate


13 Problems | 17
I5 Stick
I5 StickororRoll
Roll
Stick or roll is a game involving two players, A and B, and a die with four faces (numbered 1, 2, 3, 4). The faces are
equally likely to occur when the die is rolled.
Player A rolls the die once and sticks with that number as the score or rolls it again and scores the sum of the two
numbers.
• If A’s score is greater than 4 then A loses.
• If A’s score is 4 then A wins.

If A’s score is less than 4, then B rolls the die once and sticks with that number as the score or rolls it again and
scores the sum of the two numbers.
• If B’s score is greater than 4 then B loses.
• If B’s score is 4 or less and equal to A’s or less, then B loses.
• If B’s score is 4 or less and greater than A’s, then B wins.

Player B sticks on the first roll if that number wins and rolls again if it doesn’t win and it is possible to win with a
second roll.

Some example games might be:


∗ A rolls a 2, chooses to roll again and rolls a 3. A’s score is then 5 so A loses and B wins.
∗ A rolls a 2 and chooses not to roll again. B rolls a 2, so rolls again and rolls another 2. B’s score is then 4, so B
wins.
∗ A rolls a 1, chooses to roll again and rolls a 2. B rolls a 2, rolls again and rolls a 3. B’s score is then 5 so B loses
and A wins.

In solutions to the following questions express all probabilities exactly.


a If A’s score is 3, describe the ways in which B could win.
b If A’s score is 3, what is the probability that B wins?
c If A’s score is 2, what is the probability that B wins?
d Player A decides to stick if the first roll is 3 or 4, otherwise rolls again. What is the probability that A wins the
game using this strategy?

I6 Tandem
I6 TandemCycling
Cycling

Triplets Peter, Reeta and Nikita have two ways for getting home from school each day: cycle on a tandem bike or walk.
The bike can carry either one or two riders at a time. Regardless of the number of people pedalling, cycling speed
is 5 times walking speed. The triplets always leave school at the same time and always use the same path between
school and home, whether walking or cycling. The school is 5 km from home and their walking speed is 4 kilometres
per hour.

a On Monday, Nikita and Peter cycle and Reeta walks. On reaching the point four-fifths of the way home the bike
gets a puncture, so Nikita and Peter walk the rest of the way home. How far from school is Reeta when the cyclists
arrive home?
b On Tuesday, Peter and Reeta ride the bike and Nikita walks. When the cyclists arrive home, Peter hops off the
bike and Reeta rides back towards school to collect Nikita. How far from school is Nikita when Reeta reaches her?
c On Wednesday, Reeta and Nikita take the bike and Peter walks. When the cyclists are halfway home, Reeta hops
off and walks the rest of the way, while Nikita heads back to pick up Peter. How far from school is Reeta when her
siblings pass her on the bike?
d On Thursday, it is Reeta’s turn to walk. Peter drops Nikita off at a certain point leaving her to walk home.
Meanwhile he returns to pick up Reeta and they cycle home together. If all three arrive home at the same time,
how far from school are the drop-off and pick-up points?

Mathematics Contests The Australian Scene 2022 | Maths Challenge Intermediate


14 Problems | 18
Maths Challenge
2022
Middle Primary Challenge Solutions - Middle Primary
Solutions

MP1 Outer
MP1 OuterSums
Sums
Note that, although alternative arrangements are presented, only one correct diagram is required for each part.
a
1
2 2
2 3
3 1 3 1 1 2 3 1
1 3 1 3
3 2
2 2 2

Outer sum = 1 + 2 + 2 + 1 + 2 + 2 = 10. 3 1

Outer sum = 1 + 1 + 1 + 1 + 3 + 3 = 10.

There are other possible arrangements.

b
1
1 1
2 3
2 3 2 3 1 2 3 1
1 2 3 2 3 1
3 2
3 1 2 3 2

Outer sum = 1 + 1 + 1 + 1 + 2 + 1 + 3 = 10. 2 1 1 3

Outer sum = 1 + 1 + 1 + 3 + 1 + 1 + 2 = 10.

There are other possible arrangements.

1
2 3
1 2 3 1
3 2
1 1 1 2 3
3 2 3 2 3 2 1 3 2 1
2 3 2 3 2 3 3 2
1 1 1 1

Outer sum = 2 + 1 + 1 + 1 + 2 + 1 + 1 + 1 = 10. Outer sum = 2 + 1 + 1 + 1 + 2 + 1 + 1 + 1 = 10.

There are other possible arrangements.

d
1
3 2
2 3 2 1
1 3
1 3
2 3 2 1
3 2
1

Outer sum = 2 + 1 + 1 + 1 + 1 + 2 = 8.

Mathematics Contests The Australian Scene 2022 | Maths Challenge Middle


15 Primary Solutions | 19
MP2 Palindromes
MP2 Palindromes
a The first digit is 2. So the last digit is 2. The middle two digits must form a palindrome. Thus there are 10
palindromes between 2000 and 3000:

2002, 2112, 2222, 2332, 2442, 2552, 2662, 2772, 2882, 2992.

b The palindromes up to 121 are 11, 22, 33, 44, 55, 66, 77, 88, 99, 101, 111. Working systematically, we find there
are only five triples of different palindromes that total 121:

121 = 88 + 33 = 88 + 22 + 11 (given)

121 = 77 + 44 = 77 + 33 + 11
121 = 66 + 55 = 66 + 44 + 11
121 = 66 + 55 = 66 + 33 + 22
121 = 55 + 66 = 55 + 44 + 22

c The following table lists all palindromes less than 2000.

Palindromes Number Subtotal


11, 22, 33, . . . , 99 9 9
101, 111, . . . , 191 10 19
202, 212, . . . , 292 10 29
303, 313, . . . , 393 10 39
404, 414, . . . , 494 10 49
505, 515, . . . , 595 10 59
606, 616, . . . , 696 10 69
707, 717, . . . , 797 10 79
808, 818, . . . , 898 10 89
909, 919, . . . , 999 10 99
1001, 1111, 1221, . . . , 1991 10 109

So the 110th palindrome is 2002.

MP3 Quad
MP3 QuadCuts
Cuts
a A line along either diagonal will divide the quadrilateral into two triangles. A line from any edge to the opposite
edge will produce two quadrilaterals.

Q
T

T
Q

b The quadrilateral can be divided into a pentagon (P) and a triangle by a line from any edge to an adjacent edge.

Mathematics Contests The Australian Scene 2022 | Maths Challenge Middle


16 Primary Solutions | 20
c

T T Q T

Q Q P

T H

T
T T
T

MP4 Near-square
MP4 Near-squareRectangles
Rectangles

Note that each diagram below gives one way to draw the internal lines between NSRs, but there are other ways.

a NSR1, NSR2, and NSR3 can be joined to form NSR4.

3 2
NSR1 1

4 NSR3
NSR2 3

3 2

Mathematics Contests The Australian Scene 2022 | Maths Challenge Middle


17 Primary Solutions | 21
b Alternative i
NSR1, NSR2, NSR3, and NSR4 can be joined to form a 5 × 8 rectangle.

1 3 4

NSR1
2 NSR2

NSR4 5

3 NSR3

4 4

Alternative ii
NSR1, NSR2, NSR3, and NSR4 can be joined to form a 4 × 10 rectangle.

2 3 5

3 NSR2
NSR3 NSR4 4

1 NSR1
2 3 5

c NSR2, NSR3, NSR4, NSR5, and NSR6 can be joined to form NSR10.

5 6

6 NSR5
NSR6 7

4 NSR4
NSR3 NSR2 3

5 4 2

Mathematics Contests The Australian Scene 2022 | Maths Challenge Middle


18 Primary Solutions | 22
d The total number of grid squares in NSR2, NSR3, NSR4, NSR5, and NSR6 is

(2 × 3) + (3 × 4) + (4 × 5) + (5 × 6) + (6 × 7) = 6 + 12 + 20 + 30 + 42 = 110.

Alternative i
An NSR6 cannot fit in a rectangle whose shorter side is less than 6 cm. A rectangle whose shorter side is more that
10 cm has more than 110 grid squares. The following table gives the number of grid squares in various rectangles
with shorter side 6 cm to 10 cm.
Rectangle Number of squares Rectangle Number of squares
6 × 18 108 6 × 19 114
7 × 15 105 7 × 16 112
8 × 13 104 8 × 14 112
9 × 12 108 9 × 13 117
10 × 11 110

Thus no rectangle with shorter side 6, 7, 8, or 9 cm contains exactly 110 grid squares. So no rectangle other than
NSR10 can be formed by joining NSR2, NSR3, NSR4, NSR5, and NSR6.
Alternative ii
The only rectangles with 110 grid squares and whole number side lengths have dimensions 1 × 110, 2 × 55, 5 × 22,
and 10 × 11. Of these, only 10 × 11 is wide enough to include NSR6. So no rectangle other than NSR10 can be
formed by joining NSR2, NSR3, NSR4, NSR5, and NSR6.

Mathematics Contests The Australian Scene 2022 | Maths Challenge Middle


19 Primary Solutions | 23
Maths Challenge
Upper Primary Solutions
2022 Challenge Solutions - Upper Primary

UP1 Zipline
UP1 Numbers
Zipline Numbers
a If a Zipline number starts with 9, then its next four digits must be even. So the largest Zipline number is 98642.
Since there are only three even digits less than 5, the first digit of a Zipline number is at least 6. The only Zipline
numbers starting with 6 are 65420, 64320, 64210. So the smallest Zipline number is 64210.
b The last digit of an odd number must be odd. So the first four digits of an odd Zipline number must be even. Also,
none of the digits can be 0, otherwise the five digits would not be in descending order. So the first four digits must
be 8, 6, 4, 2, in that order. Then the last digit can only be 1.
Thus the only odd Zipline number is 86421.
c Alternative i
As half of 186282 is more than 90000, at least one of the two numbers must begin with a 9.
The only Zipline numbers beginning with 9 are 98642, 98640, 98620, 98420 and 96420. Taking each of those away
from 186282 gives 87640, 87642, 87662, 87862, 89862. Of these, only the first two are Zipline numbers as the others
all have a repeated digit.
So the two pairs are {98642, 87640} and {98640, 87642}.
Alternative ii
Since a Zipline number has four even digits and one odd digit and the five digits are in descending order, its last
digit must be 2 or less, its second last digit must be 4 or less, and its third last digit must be 6 or less.
We require two Zipline numbers that total 186282.
The sum of the last digits in the two Zipline numbers must end in 2. Since the only Zipline number ending in 1 is
86421 and 86421 + 86421 ̸= 186282, one of the Zipline numbers we want must end in 0 and the other must end in 2.
Hence the sum of the second last digits in the Zipline numbers must end in 8. So both digits must be 4 and we have

∗∗∗40
+∗∗∗42
186282

So the third last digits must be 5 or 6. Since their sum ends in 2, both must be 6.
So the fourth last digits must be 7, 8, or 9. Since their sum must end in 5, one must be 7 and the other must be 8.
Hence we have two possibilities:
∗7640 ∗8640
+∗8642 +∗7642
186282 186282

In both cases, the digit before 8 must be 9 and the digit before 7 must be 8. So we have

87640 98640
+98642 +87642
186282 186282

Thus the two required pairs of Zipline numbers are {87640, 98642} and {87642, 98640}.
d All Zipline numbers are less than 100000. So at least three are needed to sum to a number greater than 200000.
From Part a, the smallest Zipline number is 64210. So the four smallest Zipline numbers are: 64210, 64320, 65420,
76420. Hence the smallest sum of three Zipline numbers is 64210 + 64320 + 65420 = 193950, the next smallest sum
of three Zipline numbers is 64210 + 64320 + 76420 = 204950, and the smallest sum of four or more Zipline numbers
is 64210 + 64320 + 65420 + 76420 = 270370. Therefore the required sum is 204950.

Mathematics Contests The Australian Scene 2022 | Maths Challenge Upper


20 Primary Solutions | 24
UP2 Near-square
UP2 Near-squareRectangles
Rectangles

Note that each diagram below gives one way to draw the internal lines between NSRs, but there are other ways.
a Alternative i
NSR1, NSR2, NSR3, and NSR4 can be joined to form a 5 × 8 rectangle.

1 3 4

NSR1
2 NSR2

NSR4 5

3 NSR3

4 4

Alternative ii
NSR1, NSR2, NSR3, and NSR4 can be joined to form a 4 × 10 rectangle.

2 3 5

3 NSR2
NSR3 NSR4 4

1 NSR1
2 3 5

b NSR2, NSR3, NSR4, NSR5, and NSR6 can be joined to form NSR10.

5 6

6 NSR5
NSR6 7

4 NSR4
NSR3 NSR2 3

5 4 2

c The total number of grid squares in NSR2, NSR3, NSR4, NSR5, and NSR6 is

(2 × 3) + (3 × 4) + (4 × 5) + (5 × 6) + (6 × 7) = 110.

The only rectangles with 110 grid squares and whole number side lengths have dimensions 1 × 110, 2 × 55, 5 × 22,
and 10 × 11. None of the first three are wide enough to accommodate NSR6. So no rectangle other than NSR10
can be formed by joining NSR2, NSR3, NSR4, NSR5, and NSR6.

Mathematics Contests The Australian Scene 2022 | Maths Challenge Upper


21 Primary Solutions | 25
d The total number of grid squares in NSR1, NSR2, NSR3, NSR4, . . . must be the number of grid squares in a larger
near-square rectangle.
In the following the table, the second row shows the number of grid squares S in each NSR. The third row shows
the total number of grid squares in all the NSRs up to and including the one in that column, starting from NSR1.

NSR 1 2 3 4 5 6 7 8 9 10 11 12 13 14 15
S 2 6 12 20 30 42 56 72 90 110 132 156 182 210 240
Total 2 8 20 40 70 112 168 240

So the next possibility is NSR15, which has the same number of grid squares as all of NSR1, NSR2, . . . , NSR8.
The diagram shows how NSR15 can be formed from these smaller NSRs.

7 9

6 NSR6
NSR1 NSR8 8

5 NSR5
NSR2

NSR7 7

4 NSR4 NSR3

5 3 8

UP3 Poly
UP3 PolyCuts
Cuts

Two essentially different solutions are given for some parts, but there are other variations. Note that solutions for
Parts b, c, d must use concave hexagons.

or

or

Mathematics Contests The Australian Scene 2022 | Maths Challenge Upper


22 Primary Solutions | 26
c

or

or

UP4 Mixed-up
UP4 Mixed-upBirthdays
Birthdays
a There are 5 yellow cards with digit 0 and 5 yellow cards with non-zero digits. So the birthday months are 01, 02,
04, 07, 08 or 02, 04, 07, 08, 10. Hence either Dylan was born in July and Ebony in August, or Dylan in August
and Ebony in October. In the latter case, their birthdays would be more than 30 days apart, which is greater than
3 weeks. So we have:

Annie ∗ ∗ 0 1
Basil ∗ ∗ 0 2
Carol ∗ ∗ 0 4
Dylan ∗ ∗ 0 7
Ebony ∗ ∗ 0 8

b Since a month has at most 31 days, all of the five green digits 5, 6, 8, 9, 9 must be second digits in the five birthdays.
From Part a, Annie’s birthday is in January and Basil’s birthday is in February. Basil’s birthday is 33 days after
Annie’s and there are 31 days in January. Hence, to get the day of Basil’s birthday in February, we add 33 to the
day of Annie’s birthday in January and subtract 31, that is, add 2. Hence the second green digits for Annie and
Basil’s birthdays are 6 and 8 respectively.
c Since a month has at most 31 days, all of the five green digits 5, 6, 8, 9, 9 must be second digits in the five birthdays.
From Part a, Dylan’s birthday is in July and Ebony’s is in August. Ebony’s birthday is 21 days after Dylan’s and
July has 31 days. Hence, to get the day of Ebony’s birthday in August, we add 21 to the day of Dylan’s birthday
in July and subtract 31, that is, subtract 10. This means the second green digits of their birthdays are the same.
Hence the second green digits for Dylan and Ebony’s birthdays are both 9.
d From Parts a, b, c, we have:

Annie ∗ 6 0 1
Basil ∗ 8 0 2
Carol ∗ ∗ 0 4
Dylan ∗ 9 0 7
Ebony ∗ 9 0 8

This leaves the following six green cards to fill the six vacant places: 0, 1, 1, 2, 2, 5.
Since a month has at most 31 days, the second green digit for Carol must be 5.
From the solution to Part c, the day of Dylan’s birthday in July is 10 days more than the day of Ebony’s birthday
in August. Hence their first green digits are either 1 and 0, or 2 and 1, respectively.
From the solution to Part b, the day of Basil’s birthday in February is 2 days more than the day of Annie’s birthday
in January. Hence their first green digits are the same. Since 1 must be used for Dylan or Ebony, Annie and Basil’s
first green digits must be 2.
This means the first green digits for Dylan and Ebony’s birthdays are 1 and 0 respectively, which leaves 1 for Carol
and gives us the complete birthday board:

Mathematics Contests The Australian Scene 2022 | Maths Challenge Upper


23 Primary Solutions | 27
Day Month

Annie 2 6 0 1

Basil 2 8 0 2

Carol 1 5 0 4

Dylan 1 9 0 7

Ebony 0 9 0 8

Mathematics Contests The Australian Scene 2022 | Maths Challenge Upper


24 Primary Solutions | 28
Maths Challenge
Junior Solutions 2022 Challenge Solutions - Junior

J1 Dropdown
J1 DropdownNumbers
Numbers
a No 2-digit number can be a dropdown number because the digits must be different so neither digit can be the
average. The smallest 3-digit dropdown number is 102.
b The three smallest numbers with 4 different digits are 1023, 1024, 1025. Of these only 1025 is a dropdown number,
so it is the smallest.
The three largest numbers with 4 different digits are 9876, 9875, 9874. Of these only 9874 is a dropdown number,
so it is the largest.
c Since all its digits are different, the average of the digits in a 3-digit dropdown number must be one of 1, 2, 3, 4,
5, 6, 7, 8. The corresponding digit-sums are 3, 6, 9, 12, 15, 18, 21, 24 respectively. The following table shows all
possible sets of digits for each digit sum.

Digit Digit Digits


sum average
3 1 {0, 1, 2}
6 2 {0, 2, 4} {1, 2, 3}
9 3 {0, 3, 6} {1, 3, 5} {2, 3, 4}
12 4 {0, 4, 8} {1, 4, 7} {2, 4, 6} {3, 4, 5}
15 5 {1, 5, 9} {2, 5, 8} {3, 5, 7} {4, 5, 6}
18 6 {3, 6, 9} {4, 6, 8} {5, 6, 7}
21 7 {5, 7, 9} {6, 7, 8}
24 8 {7, 8, 9}

There are 4 permissible arrangements of digits for each of the triples {0, 1, 2}, {0, 2, 4}, {0, 3, 6}, {0, 4, 8}. There
are 6 arrangements of digits for each of the remaining 16 triples. So the total number of 3-digit dropdown numbers
is 16 × 6 + 4 × 4 = 96 + 16 = 112.
d From Part a, there are no 2-digit dropdown numbers.
The digit sum of a 3-digit dropdown number is divisible by 3 and hence the number itself is divisible by 3. No two
consecutive numbers can both be divisible by 3 so there are no consecutive pairs of 3-digit dropdown numbers.
Suppose two consecutive 4-digit numbers have digit sums divisible by 4. Then the difference in their digit sums
must be a multiple of 4, hence can’t be 1. So the last digit of the smaller number must be 9. It must also have
the digit 4. So the smallest such dropdown number is 1249. The next integer is 1250, which is also a dropdown
number.
Comment
It turns out that there are only 8 pairs of consecutive 4-digit integers that are both dropdown numbers:
(1249, 1250), (2149, 2150), (4529, 4530), (4569, 4570),
(5429, 5430), (5469, 5470), (7849, 7850), (8749, 8750).

Mathematics Contests The Australian Scene 2022 | Maths Challenge Junior


25 Solutions | 29
J2 PhancyPhotos
J2 Phancy Photos
a Alternative i
The height of the poster must be the total of 3 photo heights of 16 cm each and 4 rows of 2 cm spacing, that is
(3 × 16) + (4 × 2) = 48 + 8 = 56 cm.
Similarly, the width of the poster must be
(3 × 12) + (4 × 2) = 36 + 8 = 44 cm.
Alternative ii
The height of one photo together with one spacing gap is 16 + 2 = 18 cm.
So the height of the poster is (3 × 18) + 2 = 54 + 2 = 56 cm.
Similarly, the poster width is (3 × 14) + 2 = 42 + 2 = 44 cm.
b There are four possible arrangements of photos:
1 row of 24 photos
2 rows of 12 photos each
3 rows of 8 photos each
4 rows of 6 photos each.
Alternative i
Using the second method of solving Part a gives the corresponding poster areas in cm2 as follows:

((24 × 14) + 2) × (18 + 2) = 338 × 20 = 6760


((12 × 14) + 2) × ((2 × 18) + 2) = 170 × 38 = 6460
((8 × 14) + 2) × ((3 × 18) + 2) = 114 × 56 = 6384
((6 × 14) + 2) × ((4 × 18) + 2) = 86 × 74 = 6364

So the smallest poster area is 6364 cm2 .


Alternative ii
Irrespective of the display, each photo has a 2 cm space above and on the right of it. Thus each poster consists of 24
rectangles, each 14 cm wide and 18 cm high, plus 2 cm spacing along its base and on its left side. The total length
of that spacing for each display in cm is as follows:

(24 × 14) + (1 × 18) + 2 = 336 + 18 + 2 = 356


(12 × 14) + (2 × 18) + 2 = 168 + 36 + 2 = 206
(8 × 14) + (3 × 18) + 2 = 112 + 54 + 2 = 168
(6 × 14) + (4 × 18) + 2 = 84 + 72 + 2 = 158

So the smallest poster area is (2 × 158) + (24 × 14 × 18) = 316 + 6048 = 6364 cm2 .
c Alternative i
We use the second method of solving Part a to find the poster areas.
With the photos in portrait orientation, the poster area is ((6 × 15) + 3) × ((3 × 19) + 3) = 93 × 60 = 5580 cm2 .
With the photos in landscape orientation, the poster area is ((6 × 19) + 3) × ((3 × 15) + 3) = 117 × 48 = 5616 cm2 .
So the portrait orientation requires the smaller poster area.
Alternative ii
Irrespective of the display, each photo has a 3 cm space above and on the right of it. Thus each poster consists of
18 rectangles, each 15 cm × 19 cm, plus 3 cm spacing along its base and on its left side. The total length of that
spacing for each display is as follows.
With the photos in portrait orientation the length is (6 × 15) + (3 × 19) + 3 = 90 + 57 + 3 = 150 cm.
With the photos in landscape orientation the length is (6 × 19) + (3 × 15) + 3 = 114 + 45 + 3 = 162 cm.
So the portrait orientation requires the smaller poster area.
d Since the total width of 4 photos is 48 cm, the vertical spacing either side of the photos is (62−48)/5 = 14/5 = 2.8 cm.
So the total height of the poster needed for 6 rows of photos is 6 × 16 + 7 × 2.8 = 96 + 19.6 = 115.6 cm.
Therefore the length of the poster that needs to be trimmed is 120 − 115.6 = 4.4 cm.

Mathematics Contests The Australian Scene 2022 | Maths Challenge Junior


26 Solutions | 30
J3 Unit
J3 UnitTriangles
Triangles
a Alternative i
1
The rectangle covers 6 whole triangles and 4 half triangles, so its area is 6 + 4 × 2 = 8 ut.
The triangle’s area is half that of the rectangle, that is, 4 ut.
Alternative ii
If a parallelogram is sheared (tilted) as shown, its area remains the same because the triangle that is removed on
the left is congruent to the triangle added on the right. So the rectangle has the same area as the parallelogram,
which is 8 ut.

If a triangle is sheared as shown, its area remains the same (half the rectangle with the same base and height). So
the given triangle has the same area as the equilateral triangle, which is 4 ut.

b Alternative i
The triangle’s area is half that of the rectangle shown.

The rectangle covers 28 whole triangles and 8 half triangles, so its area is 32 ut.
Therefore the given triangle’s area is 16 ut.

Mathematics Contests The Australian Scene 2022 | Maths Challenge Junior


27 Solutions | 31
Alternative ii
The triangle’s area is half that of the parallelogram shown.

The area of the parallelogram is 4 × 8 = 32 ut.


Therefore the given triangle’s area is 16 ut.
Alternative iii
The area of the given triangle equals that of the equilateral triangle shown as they have the same base and height.
So the area of the triangle is 16 ut.

c Alternative i
The area of the enclosing rectangle on the left is 4 × 21 + 12 = 14 ut. The area of each light grey triangle is half the
area of its own enclosing rectangle, as shown in the other diagrams.

The enclosing rectangles have areas 8, 3, 7 ut. So the areas of the light grey triangles are 4, 1.5, 3.5 ut respectively.
Subtracting the areas of the light grey triangles, we see that the area of the shaded triangle is 14−(4+1.5+3.5) = 5 ut.
Alternative ii
The area of the enclosing parallelogram is 12 ut. The area of each light grey triangle is half the area of its own
enclosing parallelogram, as shown.

Mathematics Contests The Australian Scene 2022 | Maths Challenge Junior


28 Solutions | 32
The enclosing parallelograms have areas 4, 4, 6 ut. So the areas of the light grey triangles are 2, 2, 3 ut respectively.
Subtracting the areas of the light grey triangles, we find that the area of the shaded triangle is 12−(2+2+3) = 5 ut.
Alternative iii
Triangles ABC and AB ′ C have the same base, AC. Since BB ′ and AC are parallel, the triangles also have the
same height, hence the same area. Similarly, triangles AB ′ C and AB ′ C ′ have the same area. The area of triangle
AB ′ C ′ is half the area of the parallelogram AB ′ DC ′ . So the area of triangle ABC is 21 × 10 = 5 ut.

C C′
D

B′ A

d A parallelogram with its vertices at grid points and adjacent sides of length a and b drawn along grid lines contains
ab upward-pointing triangles and ab downward-pointing triangles, so its area is 2ab ut. Cutting the parallelogram
along either of its diagonals gives two congruent triangles each with area ab ut.
The area of the given parallelogram is 2 × 4 × 6 = 48 ut. We will now show that the area of each of the following
spanning triangles ABC is 16 ut.

Alternative i
The areas of the white triangles with sides AB, BC, CA in the first diagram are 4 × 4 = 16, 2 × 2 = 4, and
6 × 2 = 12. So the area of the spanning triangle ABC is 48 − 16 − 4 − 12 = 16 ut.
Similarly, the white triangles in the second diagram have areas 2 × 2 = 4, 4 × 4 = 16 and 2 × 6 = 12. So the second
spanning triangle also has area 16 ut.

Mathematics Contests The Australian Scene 2022 | Maths Challenge Junior


29 Solutions | 33
Alternative ii
Triangles ABC and ABC ′ in the following diagrams have the same base, AB. Since AB and CC ′ are parallel, the
triangles also have the same height, hence the same area. The area of triangle ABC ′ is 4 × 4 = 16 ut.

A
C′

A
C′

Comment
The solution of Extension 1 shows that the two triangles ABC are the only non-congruent spanning triangles whose
areas are one-third the area of the parallelogram.

J4 GoGo
J4 GoGo
a As much as possible, try to remove 2 tokens with each clearing step. Here are four sequences of 11 clearing steps.
There are others. Only one is required.
2, 3, 5, 7, A, C, G, 8, D, 4, H
E, 8, 7, F, 5, D, 4, C, A, 6, H
5, D, 7, F, E, 8, A, 6, 3, 1, 4
7, G, 6, 8, H, 5, F, 4, C, 2, 1
b Each step removes at most 2 tokens. So 10 or fewer steps will remove at most 20 tokens. Therefore at least 11 steps
are needed to remove all 21 tokens in the given pattern.
c In any clearing sequence, the first four steps must be chosen from rows 2, 4, 7 and columns A and F in some order.
If a fifth step is possible it will also be one of these five options. So after four or five steps we have the following
pattern.

Mathematics Contests The Australian Scene 2022 | Maths Challenge Junior


30 Solutions | 34
8 • • •
7
6 • • •
5
4
3 • • •
2
1 • • •
A B C D E F G H

Since each row and column now contains either 0 or 3 tokens, no further tokens can be removed.
d If every square in a 3 × 3 sub-grid within an 8 × 8 grid has a token, then none of those tokens can be removed. So
the least number of tokens in a non-clearable pattern is 9 or fewer.
Suppose a pattern has 8 or fewer tokens. We show that we can remove at least 1 token, and the pattern is therefore
clearable by repetition. If a row has only 1 or 2 tokens, then these can be removed. So we may assume that all
non-empty rows have at least 3 tokens. Since the pattern has at most 8 tokens, this means there are at most two
non-empty rows. Then each column has at most 2 tokens and can therefore be cleared.
Hence the least number of tokens in a non-clearable pattern is 9.

J5 Towers
J5 Towersand
andCities
Cities
a 1 block:
1

1
CV=1

2 blocks:
2

1 1 1

3 1 1
CV=3 CV=2

3 blocks:
3

2 2

1 1 1 1 1 1

6 3 1 1 1 1
CV=6 CV=4 CV=3

4 blocks:
4

3 3

2 2 2 2

1 1 1 1 1

10 6 1 3 3
CV=10 CV=7 CV=6

Mathematics Contests The Australian Scene 2022 | Maths Challenge Junior


31 Solutions | 35
2

1 1 1 1 1 1 1

3 1 1 1 1 1 1
CV=5 CV=4

b Here are two cities with value 26. Each has 10 blocks.

5 5

4 4 4

3 3 3

2 2 2 2

1 1 1 1 1 1 1

So a city with value 26 needs at most 10 blocks. We now show that any city with value 26 must have at least 10
blocks.
Alternative i
The value of a tower with 7 or more blocks is at least 28. So the tallest tower in a city of value 26 has at most 6
blocks.
Suppose the tallest tower in a city has 6 blocks. A tower of 6 blocks has value 21. From Part a, adding up to 3
blocks will not achieve a city value of 26. So the city must have at least 10 blocks.
Suppose the tallest tower in a city has at most 5 blocks and the city has at most 9 blocks. If two towers have less
than 5 blocks each, then we can increase the value of the city by moving the block on top of one of those towers to
the top of the other. So the city will have maximum value with one tower of 5 blocks and one tower of 4 blocks.
Hence its maximum value is 15 + 10 = 25, which is less than 26. So the city must have at least 10 blocks.
Thus the minimum number of blocks in a city with value 26 is 10.
Alternative ii
If a city has more than one tower, then the value of the city can be increased by moving a block on top of a tower
to the top of another tower that is the same height or higher. Hence, if each tower in a city has at most t blocks,
then the maximum value of the city is achieved when at most one of its towers has less than t blocks. In particular,
if a city has b blocks and there is no restriction on tower heights, then its maximum value is 1 + 2 + 3 + · · · + b.
So the value of a city with at most 6 blocks is at most 1 + 2 + 3 + 4 + 5 + 6 = 21. Therefore, a city of value 26 must
have at least 7 blocks.
The value of a tower with 7 or more blocks is at least 1 + 2 + 3 + 4 + 5 + 6 + 7 = 28. Therefore, every tower in a
city of value 26 has at most 6 blocks.
Suppose a city has 7 blocks. Since each tower has at most 6 blocks, the city has its maximum value when it has
one tower of 6 blocks and one tower of 1 block. Thus its maximum value is 21 + 1 = 22.
Suppose a city has 8 blocks. Since each tower has at most 6 blocks, the city has its maximum value when it has
one tower of 6 blocks and one tower of 2 block. Thus its maximum value is 21 + 3 = 24.
Suppose a city has 9 blocks. If it has a tower of 6 blocks, then the city’s value is 21 + 6 = 27 or 21 + 3 + 1 = 25 or
21 + 1 + 1 + 1 = 24. If all its towers have at most 5 blocks, then its maximum value is 15 + 10 = 25.
So a city with value 26 must have at least 10 blocks.
Thus the minimum number of blocks in a city with value 26 is 10.
c Alternative i
If the tallest tower in the city has 19 blocks, then the city’s value is 1 + 2 + 3 + · · · + 19 = 190.
If the tallest tower in the city has 18 blocks, then the city’s value is 1 + 2 + 3 + · · · + 18 = 171 or 171 + 1 = 172.
If the tallest tower in the city has 17 blocks, then the other 2 blocks contribute either 0, 1, 2, or 3 to the city value
giving a total value of 1 + 2 + 3 + · · · + 17 = 153, or one of 154, 155, 156.

Mathematics Contests The Australian Scene 2022 | Maths Challenge Junior


32 Solutions | 36
If the tallest tower in the city has 16 blocks, then its other 3 blocks contribute either 0, 1, 2, 3, 4, or 6 to the city
value giving a total value of 1 + 2 + 3 + · · · + 16 = 136, or one of 137, 138, 139, 140, 142.
If the tallest tower in the city has 15 blocks, then, as in Part b, it has its maximum value when it has one tower of
15 blocks and one tower of 4 blocks. So its maximum value is (1 + 2 + 3 + · · · + 15) + 10 = 130.
Similarly, if the tallest tower in the city has less than 15 blocks, then the city’s value is less than 130.
So the city values in decreasing order are: 190, 172, 171, 156, 155, 154, 153, 142, 140, 139, 138, 137, 136, . . .
Thus the 5th and 10th largest city values made from 19 blocks are 155 and 139 respectively.
Alternative ii
The only values for a city with only one tower with at most 19 blocks are:
1 + 2 + 3 + · · · + 19 = 190,
1 + 2 + 3 + · · · + 18 = 171,
1 + 2 + 3 + · · · + 17 = 153,
1 + 2 + 3 + · · · + 16 = 136, or lower.
The only values for two towers with a total of 19 blocks are:
171 + 1 = 172,
153 + 3 = 156,
136 + 6 = 142,
120 + 10 = 130, or lower.
The only values for two towers with a total of 18 blocks are:
153 + 1 = 154,
136 + 3 = 139, or lower.
The only values for two towers with a total of 17 blocks are 136 + 1 = 137 or lower.
The value for two towers with at most 16 blocks is at most 120 + 1 = 121.
The only values for three towers with 19 blocks are:
153 + 1 + 1 = 155,
136 + 3 + 1 = 140,
120 + 6 + 1 = 127, or lower.
The value for three towers with at most 18 blocks is at most 136 + 1 + 1 = 138.
The only values for four towers with a total of 19 blocks are 136 + 1 + 1 + 1 = 139 or lower.
The value for four towers with at most 18 blocks is at most 120 + 1 + 1 + 1 = 123.
The value for at least five towers with at most 19 blocks is at most 120 + 1 + 1 + 1 + 1 = 124.
So the city values in decreasing order are: 190, 172, 171, 156, 155, 154, 153, 142, 140, 139, 138, 137, 136, . . .
Thus the 5th and 10th largest city values made from 19 blocks are 155 and 139 respectively.
d Alternative i
If the tallest tower in Shakira’s city has 6 blocks, then the city’s value is 1 + 2 + 3 + 4 + 5 + 6 = 21.
If the tallest tower in Shakira’s city has 5 blocks, then the city’s value is 1 + 2 + 3 + 4 + 5 = 15 or 15 + 1 = 16.
If the tallest tower in Shakira’s city has 4 blocks, then that tower has value 1 + 2 + 3 + 4 = 10. The other 2 blocks
contribute at most 3 to the city value. So the maximum city value is 13.
If the tallest tower in Shakira’s city has 3 blocks, then the maximum city value is 6 + 6 = 12.
If the tallest tower in Shakira’s city has 2 blocks, then the maximum city value is 3 + 3 + 3 = 9.
If the tallest tower in Shakira’s city has 1 block, then the city’s value is 1 + 1 + 1 + 1 + 1 + 1 = 6.
So, of 14, 16, 18, only 16 is a city value, and this is achieved with one tower of 5 blocks and one tower of 1 block.
Alternative ii
If Shakira’s city has just one tower, then the city’s value is one of: 1, 3, 6, 10, 15, 21.
If Shakira’s city has just two towers, then the city’s value is one of: 15 + 1 = 16, 10 + 3 = 13, 6 + 6 = 12.
If Shakira’s city has exactly three towers, then the city’s value is one of: 10 + 1 + 1 = 12, 6 + 3 + 1 = 10, 3 + 3 + 3 = 9.
If Shakira’s city has exactly four towers, then the city’s value is one of: 6 + 1 + 1 + 1 = 9, 3 + 3 + 1 + 1 = 8.
If Shakira’s city has exactly five towers, then the city’s value is 3 + 1 + 1 + 1 + 1 = 7.
If Shakira’s city has exactly six towers, then the city’s value is 1 + 1 + 1 + 1 + 1 + 1 = 6.
So, of 14, 16, 18, only 16 is a city value, and this is achieved with one tower of 5 blocks and one tower of 1 block.

Mathematics Contests The Australian Scene 2022 | Maths Challenge Junior Solutions | 37
33
Mixed-up
J6 Birthdays
Mixed-up Birthdays
a The birthdays are in separate months and in alphabetical order. So Dylan’s birthday must be after February. Since
the number of days between Annie and Dylan’s birthdays changes with the years, and 29 February is the only date
that is present in some years and not others, Annie’s birthday must be in January or February.
If Basil was born after February, then his birthday would not always be the same number of days after Annie’s. So
Basil was born in February and Annie in January.
b We show by elimination of months that Dylan was born in July.
From Part a, we have:

Annie ∗ ∗ 0 1
Basil ∗ ∗ 0 2
Carol ∗ ∗ ∗ ∗
Dylan ∗ ∗ ∗ ∗
Ebony ∗ ∗ ∗ ∗

Since none of the eight cards 4, 5, 6, 7, 8, 8, 9, 9 can be in the first or third columns, they must fill the eight vacant
places in the second and fourth columns. This means all digits in the third column are 0. So we have:

Annie ∗ ∗ 0 1
Basil ∗ ∗ 0 2
Carol ∗ ∗ 0 ∗
Dylan ∗ ∗ 0 ∗
Ebony ∗ ∗ 0 ∗

Since no two birthdays are in the same month and there is no card 3, Dylan must have been born after April.
Since Ebony was born in September at the latest, Dylan must have been born before September. Since Dylan and
Ebony’s birthdays are exactly three weeks apart, they were born in consecutive months. Since Ebony was not born
in July, Dylan was not born in June. So Dylan must have been born in May, July, or August.
If Dylan was born in August, we would have the following board and therefore five of the cards 4, 5, 6, 7, 8, 9 must
go in the second column.

Annie ∗ ∗ 0 1
Basil ∗ ∗ 0 2
Carol ∗ ∗ 0 ∗
Dylan ∗ ∗ 0 8
Ebony ∗ ∗ 0 9

Ebony’s birthday is exactly 21 days after Dylan’s and there are 31 days in August. So to get the day of Ebony’s
birthday in September we add 21 to the day of Dylan’s birthday in August and subtract 31, that is, subtract 10.
This means the second digits of their birthdays are the same. There are no available repeated digits. So Dylan was
not born in August.
If Dylan was born in May, we would have the following board and therefore all five of the cards 7, 8, 8, 9, 9 must
go in the second column and all five of the cards 0, 1, 1, 2, 2 must go in the first column.

Annie ∗ ∗ 0 1
Basil ∗ ∗ 0 2
Carol ∗ ∗ 0 4
Dylan ∗ ∗ 0 5
Ebony ∗ ∗ 0 6

Basil’s birthday is exactly 33 days after Annie’s and there are 31 days in January. So to get the day of Basil’s
birthday in February we add 33 to the day of Annie’s birthday in January and subtract 31, that is, add 2. Hence
the second digits of Annie and Basil’s birthdays are 7 and 9 respectively and the first digits are both 1 or both 2.
Since their birthdays are 33 days apart every year, Basil was not born on 29 February. So we have:

Annie 1 7 0 1
Basil 1 9 0 2
Carol ∗ ∗ 0 4
Dylan ∗ ∗ 0 5
Ebony ∗ ∗ 0 6

Mathematics Contests The Australian Scene 2022 | Maths Challenge Junior


34 Solutions | 38
Ebony’s birthday is exactly 21 days after Dylan’s and there are 31 days in May. So the day of Ebony’s birthday in
June is 10 less than the day of Dylan’s birthday in May. This is impossible since the only cards available for their
first digits are 0 or 2. So Dylan was not born in May.
So the only month that remains for Dylan’s birthday is July.
c From Parts a and b, we have the following table and therefore all six of the cards 4, 5, 6, 8, 9, 9 must go in the
second and fourth columns and all five of the cards 0, 1, 1, 2, 2 must go in the first column.

Annie ∗ ∗ 0 1
Basil ∗ ∗ 0 2
Carol ∗ ∗ 0 ∗
Dylan ∗ ∗ 0 7
Ebony ∗ ∗ 0 8

As in the solution to Part b, the day of Ebony’s birthday in August is 10 less than the day of Dylan’s birthday in
July. So we have:
Annie ∗ ∗ 0 1
Basil ∗ ∗ 0 2
Carol ∗ ∗ 0 ∗
Dylan 1/2 9 0 7
Ebony 0/1 9 0 8

As in the solution to Part b, the day of Basil’s birthday in February is 2 more than the day of Annie’s birthday
in January. Hence their first digits are the same. This eliminates the second alternative birthdays for Dylan and
Ebony and we have:

Annie 2 4/6 0 1
Basil 2 6/8 0 2
Carol ∗ ∗ 0 ∗
Dylan 1 9 0 7
Ebony 0 9 0 8

If Annie was born on 24 January, then the number of days to Dylan’s birthday in 2020, a leap year, would be
7 + 29 + 31 + 30 + 31 + 30 + 19 = 177, which is not divisible by 7. So Annie and Dylan’s birthdays could not have
been on the same day of the week in 2020. On the other hand, if Annie’s birthday is 26 January, then the number
of days to Dylan’s birthday in 2020 would be 5 + 29 + 31 + 30 + 31 + 30 + 19 = 175, which is divisible by 7. Therefore
Annie was born on 26 January and we have only two possible birthday boards:

Day Month

Annie 2 6 0 1

Basil 2 8 0 2

Carol 1 5/4 0 4/5

Dylan 1 9 0 7

Ebony 0 9 0 8

Mathematics Contests The Australian Scene 2022 | Maths Challenge Junior


35 Solutions | 39
Maths Challenge
2022 Challenge Solutions - Intermediate
Intermediate Solutions

I1 DPS
I1 DPSChains
Chains
a There are ten chains of length 5 starting with a 2-digit number. Any one of the following is an acceptable answer:

42 → 34 → 22 → 18 → 10
43 → 31 → 28 → 12 → 10
51 → 46 → 22 → 18 → 10
54 → 34 → 22 → 18 → 10
63 → 45 → 25 → 15 → 10
74 → 46 → 22 → 18 → 10
76 → 34 → 22 → 18 → 10
85 → 45 → 25 → 15 → 10
87 → 31 → 28 → 12 → 10
97 → 34 → 22 → 18 → 10
b Alternative i
A term preceding 40 in a DPS chain of length 3 must be greater than 40. Checking numbers from 41 upwards,
we find that 64 may precede 40. A term preceding 64 → 40 in a DPS chain of length 3 must be greater than 64.
Checking numbers from 65 upwards, we find that 71 may precede 64. This gives the chain 71 → 64 → 40.
Alternative ii
Let N be a term preceding 40 in a DPS chain, let P be the product of the digits of N , and let d be the first digit
of N . So N = 40 + P and P is a multiple of d. Note that N does not end in 0 and d is greater than or equal to 4.
If d = 4, then the only possible values for N are 40 + 4 = 44 or 40 + 8 = 48, but neither of these work.
If d = 5, then the only possible value for N is 40 + 15 = 55, but this doesn’t work.
If d = 6, then the only possible value for N is 40 + 24 = 64, which does work.
Checking numbers from 65 upwards, we find that 71 may precede 64. This gives the chain 71 → 64 → 40.
c Alternative i
Let the 2-digit number 10a + b be a member of a DPS chain, where 1 ≤ a ≤ 9 and 0 ≤ b ≤ 9. The next number in
the chain is 10a + b − ab, which is less than or equal to 10a + b and therefore has at most 2 digits. We need to show
that it has at least 2 digits, that is, 10a + b − ab ≥ 10.

10a + b − ab = (10 − b)a + b


≥ (10 − b) + b
= 10.

Alternative ii
Let the 2-digit number 10a + b be a member of a DPS chain, where 1 ≤ a ≤ 9 and 0 ≤ b ≤ 9. The next number in
the chain is 10a + b − ab, which is less than or equal to 10a + b and therefore has at most 2 digits. We need to show
that it has at least 2 digits, that is, 10a + b − ab ≥ 10.

10a + b − ab − 10 = 10(a − 1) + b(1 − a)


= 10(a − 1) − b(a − 1)
= (a − 1)(10 − b)
≥ 0.

Alternative iii
An exhaustive list of all DPS numbers that follow starting numbers from 10 to 99 establishes that the next term in
the chain is also a 2-digit number. Thus all subsequent terms will also be 2-digit numbers.
d Alternative i
First note that, in a DPS chain, an even number will have an even digit product and hence will produce an even
number. Also, a number containing all odd digits will have an odd digit product and hence will also produce an
even number. Therefore, to obtain an odd number in a DPS chain, we require the preceding number to be odd but
contain an even digit.
So the number preceding 307 must be odd and have an even digit that is not 0. We can ignore any number ending
in 5 since all terms following it will end in 5 or 0. We can also ignore any number less than 307.

Mathematics Contests The Australian Scene 2022 | Maths Challenge Intermediate


36 Solutions | 40
We search in increasing order for possible numbers preceding 307:
321 → 315
323 → 305
327 → 285
329 → 275
341 → 329
343 → 307
We now search in the same way for a term preceding 343:
347 → 263
349 → 241
361 → 343
Thus we have the chain 361 → 343 → 307.
Alternative ii
Let N be a term preceding 307 in a DPS chain. As in the last solution of Part b, if the first digit of N is 3, then
N is 307 plus a multiple of 3. Since 307 is odd, N must be odd. So N is 307 plus a multiple of 6. Then the only
possible values for N are 313, 319, 325, 331, 337, 343, . . ., 397. Of these, 343 is the first that works.
Similarly, any predecessor of 343 that starts with 3 must be from 349, 355, 361, . . ., 397. Of these, 361 is the first
that works.
Thus we have the chain 361 → 343 → 307.
Comment
The chain 623 → 587 → 307 is also possible. It turns out there are no other chains of length 3 that end in 307.
However, this chain is part of the DPS chain of length 4: 839 → 623 → 587 → 307.

I2 TriangleFolding
I2 Triangle Folding
a Since the folding line HG is the perpendicular bisector of BB ′ and AB = AC, moving B ′ to C places G at A and
folds triangle ABC in half as shown.

C = B′

B A=G

1
So the area of triangle B ′ GH is one-quarter of the area of the unit square, that is, 4 = 0.25.
′ ′
b Since the folding line HG is the perpendicular bisector of BB , moving B to A creates two congruent right-angled
isosceles triangles BGH and B ′ GH as shown.

B G A = B′

Hence B ′ H is perpendicular to BC. Therefore triangles B ′ HB and B ′ HC are congruent. So the area of triangle
B ′ GH is one-eighth of the area of the unit square, that is, 18 = 0.125.

Mathematics Contests The Australian Scene 2022 | Maths Challenge Intermediate


37 Solutions | 41
c If HB ′ is perpendicular to AC, then we have:

H
B′

B G A

Since triangles BGH and B ′ GH are congruent, ̸ GB ′ H = ̸ GBH = 45◦ . Therefore ̸ AB ′ G = 45◦ . So B ′ AG is an
isosceles right-angled triangle.

Let AG = b. Then AB ′ =√b and, by √
√ Pythagoras’ theorem, GB ′ = 2b. Since GB ′ = GB = 1 − AG, we have
2b = 1 − b. Hence b = 1/( 2 + 1) = 2 − 1 ≈ 0.414.
d Alternative i
1

From Part c, the area of △AB ′ G is 2 × b2 = 12 (3 − 2 2).
Since ̸ B ′ CH = 45◦ , △B ′ CH is isosceles and B ′ H = B ′ C√= 1 − b. √
Therefore the area of △B ′ CH is 12 × (1 − b)2 = 12 × (2 − 2)2 = 3 − 2 2.
The area of △ABC is 12 . Since triangles B ′ GH and BGH have the same area, x say, we have

1
√ √
2= 2x + 12 (3 − 2 2) + 3 − 2 2

x = 12 ( 12 − 32 (3 − 2 2))

= 14 (1 − 9 + 6 2)

= 32 2 − 2 ≈ 0.121.

Alternative ii
From the solution to Part c, △AGB ′ is similar to △ABC with scale factor AG/AB = b. Hence the area of △AGB ′
is 12 b2 .
Since HB ′ and AC are perpendicular, △B ′ HC is similar to △ABC with scale factor B ′ C/AC = 1 − b. Hence the
area of △B ′ HC is 12 (1 − b)2 .
Therefore the area of B ′ HBG is
1
2 − 12 b2 − 12 (1 − b)2 = 12 (1 − b2 − (1 − b)2 )
= 12 (2b − 2b2 )
= b(1 − b)
√ √
= ( 2 − 1)(2 − 2)

= 3 2 − 4.
√ √
Hence the area of △B ′ HG is 21 (3 2 − 4) = 32 2 − 2 ≈ 0.121.

I3 GoGo
I3 GoGo
a As much as possible, try to remove 2 tokens with each clearing step. Here are four sequences of 11 clearing steps.
There are others. Only one is required.
2, 3, 5, 7, A, C, G, 8, D, 4, H
E, 8, 7, F, 5, D, 4, C, A, 6, H
5, D, 7, F, E, 8, A, 6, 3, 1, 4
7, G, 6, 8, H, 5, F, 4, C, 2, 1
b Each step removes at most 2 tokens. So 10 or fewer steps will remove at most 20 tokens. Therefore at least 11 steps
are needed to remove all 21 tokens in the given pattern.

Mathematics Contests The Australian Scene 2022 | Maths Challenge Intermediate


38 Solutions | 42
c In any clearing sequence, the first four steps must be chosen from rows 2, 4, 7 and columns A and F in some order.
If a fifth step is possible it will also be one of these five options. So after four or five steps we have the following
pattern.

8 • • •
7
6 • • •
5
4
3 • • •
2
1 • • •
A B C D E F G H

Since each row and column now contains either 0 or 3 tokens, no further tokens can be removed. 1
d If every square in a 3 × 3 sub-grid within an 8 × 8 grid has a token, then none of those tokens can be removed. So
the least number of tokens in a non-clearable pattern is 9 or fewer.
Suppose a pattern has 8 or fewer tokens. We show that we can remove at least 1 token, and the pattern is therefore
clearable by repetition. If a row has only 1 or 2 tokens, then these can be removed. So we may assume that all
non-empty rows have at least 3 tokens. Since the pattern has at most 8 tokens, this means there are at most two
non-empty rows. Then each column has at most 2 tokens and can therefore be cleared.
Hence the least number of tokens in a non-clearable pattern is 9.

I4 Slot
I4 SlotCars
Cars
a Each straight plate contributes 20 cm to the track length and each curved plate contributes 14 × 2π × 10 = 5π ≈
15.7 cm. So to reduce the length of track on a given number of additional plates we try to maximise the number of
curved plates.
The following completed circuits have 1 additional straight plate and 6 additional curved plates (only one diagram
is required):

The total length of the additional track in each case is

(1 × 20) + (6 × 5π) = 20 + 30π ≈ 114.25 cm,

which is less than 120 cm.


Comment
It can be shown that these 4 completed circuits are the only ones for which the length of additional track is less
than 120 cm.

Mathematics Contests The Australian Scene 2022 | Maths Challenge Intermediate


39 Solutions | 43
b The following completed circuits have 9 additional straight plates and 6 additional curved plates (only one diagram
is required):

The total length of the additional track is

(9 × 20) + (6 × 5π) = 180 + 30π ≈ 274.25 cm,

which is greater than 270 cm.


c Each curved plate changes the direction of the track at its edges from east-west to north-south, or vice versa. Since
the additional track starts and finishes travelling east-west, the number of such changes in the 4 × 4 grid must be
even.
d Consider the chessboard-like colouring of the 4 × 4 grid, as shown.

Any track which is traced through the 4 × 4 grid must follow a sequence of squares which alternate black and white.
Since such a track starts and finishes at two white squares, marked X, it must visit an odd number of squares along
the way, so an odd number of new plates is used altogether. By Part c, there is an even number of new curved
plates, so the number of new straight plates must be odd.

I5 Stick
I5 StickororRoll
Roll
a If A’s score is 3, then B must score 4 to win. This can be done in four ways:
B rolls 1 then 3,
B rolls 2 then 2,
B rolls 3 then 1,
B rolls 4 and sticks.
b From Part a, there are four ways in which B could win.
1 1 1
The probability of B rolling 1 then 3 is 4 × 4 = 16 .
1 1 1
The probability of B rolling 2 then 2 is 4 × 4 = 16 .
1 1 1
The probability of B rolling 3 then 1 is 4 × 4 = 16 .
The probability of B rolling 4 then sticking is 14 .
1 1 1 1 7
Hence the probability that B wins if A’s score is 3 is 16 + 16 + 16 + 4 = 16 .

c If A’s score is 2, then B must score 3 or 4 to win. This can be done as follows.
1 2 2
B rolls 1 then 2 or 3. Probability = 4 × 4 = 16 = 81 .
1 2 2
B rolls 2 then 1 or 2. Probability = 4 × 4 = 16 = 81 .
1
B rolls 3 then sticks. Probability = 4.
1
B rolls 4 then sticks. Probability = 4.
1 1 1 1 6
Hence the probability that B wins if A’s score is 2 is 8 + 8 + 4 + 4 = 8 = 34 .

Mathematics Contests The Australian Scene 2022 | Maths Challenge Intermediate


40 Solutions | 44
d This table shows all possible scores for A and how they are obtained with the given strategy.

A’s first A’s next roll


roll 1 2 3 4
1 2 3 4 5
2 3 4 5 6
3 stick with 3
4 stick with 4
1 1 1
The probability A scores 2 is 4 × 4 = 16 .
1 1 1 6
The probability A scores 3 is 16 + 16 + 4 = 16 = 38 .
1 1 1 6
The probability A scores 4 is 16 + 16 + 4 = 16 = 38 .
3 1
If A scores 2, then from Part c the probability that B loses is 1 − 4 = 4.
7 9
If A scores 3, then from Part b the probability that B loses is 1 − 16 = 16 .
If A scores 4, then A wins.
1
So the probability that A wins is ( 16 × 14 ) + ( 38 × 9
16 ) + 3
8 = 1
64 + 27
128 + 3
8 = 77
128 .

I6 Tandem
I6 TandemCycling
Cycling
a The distance from the school to where the puncture occurs is 45 × 5 = 4 km. As Reeta walks at 15 of the speed of
the cyclists, when the puncture occurs she will have walked 45 km. She then walks another 1 km while the cyclists
walk their remaining 1 km home. So Reeta will have walked a total of 1 + 45 = 95 = 1.8 km.
b Let d km be the distance from the school that Nikita has walked when Reeta collects her. The following diagram
represents the movement of the tandem by a sequence of arrows starting at the bottom.
R

P&R
school

home
• • •
0 d 5

Since Reeta cycles at 5 times the rate at which Nikita walks, we have
5d = 5 + (5 − d) = 10 − d
6d = 10
d = 53 .

c Let x km be the distance from the school that Peter has walked when Nikita collects him. Let y km be the distance
from the school that Reeta has reached when Nikita and Peter pass her.

N&P

N&R
school

home

• • • • •
0 x 5 y 5
2

1 5
As in Part b, x = 3 × 2 = 56 . Since the tandem travels at 5 times the rate at which Reeta walks, we have
5(y − 52 ) = ( 52 − 56 ) + (y − 56 )
25 5 10
5y − 2 =y+ 2 − 6
30 10
4y = 2 − 6
90 10 80
= 6 − 6 = 6
20 10
y= 6 = 3 .

Mathematics Contests The Australian Scene 2022 | Maths Challenge Intermediate


41 Solutions | 45
d Let x km be the distance from school to the point where Nikita hops off.
Let y km be the distance from school to the point where Peter picks up Reeta.

P&R

P&N
school

home
• • • •
0 y x 5

Alternative i
As in Part b, y = 13 x. Since the tandem travels at 5 times the rate at which Nikita walks, we have

5(5 − x) = (x − y) + (5 − y) = x + 5 − 2y
25 − 5x = x + 5 − 32 x = 5 + 13 x
20 = 5x + 13 x = 16
3 x
15
x= 4
5
y= 4.

Alternative ii
If the triplets reversed their trip from school to home, then the pick-up and drop-off points would be reversed.
Therefore 5 − x = y.
Since the tandem travels at 5 times the rate at which Nikita walks, we have

5(5 − x) = (x − y) + (5 − y) = x + 5 − 2y
25 − 5x = x + 5 − 2(5 − x) = x + 5 − 10 + 2x
30 = 8x
15
x= 4
15
y =5− 4 = 54 .

Mathematics Contests The Australian Scene 2022 | Maths Challenge Intermediate


42 Solutions | 46
Maths Challenge
Middle Primary Statistics

Mean Score/School Year/Problem

Challenge Problem
Number of Overall
School Year
Students Mean
1 2 3 4

3 564 9.9 2.9 2.6 2.5 1.9


4 867 11.8 3.4 3.0 3.1 2.4
*ALL YEARS 1431 11.0 3.2 2.8 2.8 2.2

Please note:
* This total includes students who did not provide their school year.

Score Distribution (%)/Problem

Challenge Problem
Score
4 Near-square
1 Outer Sums 2 Palindromes 3 Quad Cuts
Rectangles

0 8% 6% 12% 21%

1 7% 8% 8% 11%

2 6% 18% 13% 18%

3 17% 33% 20% 28%

4 62% 35% 47% 23%

Mean 3.2 2.8 2.8 2.2


Discrimination
0.6 0.5 0.7 0.7
Factor

Please note:
The discrimination factor for a particular problem is calculated as follows:

1. The students are ranked in regard to their overall scores.

2. The mean score for the top 25% of these overall ranked students is calculated for that particular
problem including no attempts. Call this mean score the ‘mean top score’.

3. The mean score for the bottom 25% of these overall ranked students is calculated for that
particular problem including no attempts. Call this mean score the ‘mean bottom score’.

4. The discrimination factor = mean top score – mean bottom score


4

Thus the discrimination factor ranges from 1 to –1. A problem with a discrimination factor of 0.4 or
higher is considered to be a good discriminator.

Mathematics Contests The Australian Scene 2022 | Maths Challenge Middle Primary Statistics | 47
Maths Challenge
Upper Primary Statistics

Mean Score/School Year/Problem

Challenge Problem
Number of Overall
School Year
Students Mean
1 2 3 4

5 1172 10.9 2.9 2.5 3.0 2.5


6 1421 11.7 3.1 2.7 3.2 2.8
*ALL YEARS 2593 11.4 3.0 2.6 3.1 2.6

Please note:
* This total includes students who did not provide their school year.

Score Distribution (%)/Problem

Challenge Problem
Score
2 Near-square 4 Mixed-up
1 Zipline Numbers 3 Poly Cuts
Rectangles Birthdays

0 4% 10% 7% 18%

1 6% 11% 5% 10%

2 17% 24% 13% 11%

3 31% 24% 18% 14%

4 42% 32% 57% 47%

Mean 3.0 2.6 3.1 2.6


Discrimination
0.5 0.6 0.5 0.8
Factor

Please note:
The discrimination factor for a particular problem is calculated as follows:

1. The students are ranked in regard to their overall scores.

2. The mean score for the top 25% of these overall ranked students is calculated for that particular
problem including no attempts. Call this mean score the ‘mean top score’.

3. The mean score for the bottom 25% of these overall ranked students is calculated for that
particular problem including no attempts. Call this mean score the ‘mean bottom score’.

4. The discrimination factor = mean top score – mean bottom score


4

Thus the discrimination factor ranges from 1 to –1. A problem with a discrimination factor of 0.4 or
higher is considered to be a good discriminator.

Mathematics Contests The Australian Scene 2022 | Maths Challenge Upper Primary Statistics | 48
Maths Challenge
Junior Statistics

Mean Score/School Year/Problem

Challenge Problem
Number of Overall
School Year
Students Mean
1 2 3 4 5 6

7 2471 11.1 2.0 2.2 2.2 2.1 1.5 1.2


8 2053 12.0 2.1 2.2 2.2 2.4 1.6 1.4
*ALL YEARS 4524 11.5 2.0 2.2 2.2 2.2 1.6 1.3

Please note:
* This total includes students who did not provide their school year.

Score Distribution (%)/Problem

Challenge Problem
Score 1 2 3 4 5 6
Dropdown Phancy Unit GoGo Towers and Mixed-up
Numbers Photos Triangles Cities Birthdays
0 15% 20% 16% 24% 30% 46%

1 16% 15% 16% 10% 18% 17%

2 32% 19% 22% 14% 28% 13%

3 24% 22% 28% 21% 16% 10%

4 13% 25% 19% 31% 9% 15%

Mean 2.0 2.2 2.2 2.2 1.6 1.3


Discrimination
0.5 0.7 0.7 0.8 0.6 0.7
Factor

Please note:
The discrimination factor for a particular problem is calculated as follows:

1. The students are ranked in regard to their overall scores.

2. The mean score for the top 25% of these overall ranked students is calculated for that particular
problem including no attempts. Call this mean score the ‘mean top score’.

3. The mean score for the bottom 25% of these overall ranked students is calculated for that
particular problem including no attempts. Call this mean score the ‘mean bottom score’.

4. The discrimination factor = mean top score – mean bottom score


4

Thus the discrimination factor ranges from 1 to –1. A problem with a discrimination factor of 0.4 or
higher is considered to be a good discriminator.

Mathematics Contests The Australian Scene 2022 | Maths Challenge Junior Statistics | 49
Maths Challenge
Intermediate Statistics

Mean Score/School Year/Problem

Challenge Problem
Number of Overall
School Year
Students Mean
1 2 3 4 5 6

9 1452 14.9 3.1 2.5 2.9 2.6 1.8 2.0


10 688 15.1 3.0 2.4 2.8 2.6 1.9 2.4
*ALL YEARS 2140 15.0 3.1 2.5 2.9 2.6 1.8 2.1

Please note:
* This total includes students who did not provide their school year.

Score Distribution (%)/Problem

Challenge Problem
Score 1 2 3 4 5 6
DPS Chains Triangle GoGo Slot Cars Stick or Roll Tandem
Folding Cycling
0 8% 15% 12% 17% 23% 22%

1 4% 8% 6% 7% 30% 17%

2 8% 27% 12% 16% 12% 15%

3 31% 13% 20% 20% 13% 19%

4 48% 37% 49% 40% 22% 26%

Mean 3.1 2.5 2.9 2.6 1.8 2.1


Discrimination
0.5 0.7 0.6 0.7 0.7 0.7
Factor

Please note:

The discrimination factor for a particular problem is calculated as follows:

1. The students are ranked in regard to their overall scores.

2. The mean score for the top 25% of these overall ranked students is calculated for that particular
problem including no attempts. Call this mean score the ‘mean top score’.

3. The mean score for the bottom 25% of these overall ranked students is calculated for that
particular problem including no attempts. Call this mean score the ‘mean bottom score’.

4. The discrimination factor = mean top score – mean bottom score

Thus the discrimination factor ranges from 1 to –1. A problem with a discrimination factor of 0.4 or
higher is considered to be a good discriminator.

Mathematics Contests The Australian Scene 2022 | Maths Challenge Intermediate Statistics | 50
Thursday 8
September 2022
2022 Australian Intermediate Mathematics Olympiad - Questions
2022 Australian Intermediate Mathematics Olympiad

Questions
Time allowed: 4 hours. NO calculators are to be used.
Questions 1 to 8 only require their numerical answers all of which are non-negative integers less than 1000.
Questions 9 and 10 require written solutions which may include proofs.
The bonus marks for the Investigation in Question 10 may be used to determine prize winners.

1. A 3-digit number abc is multiplied by 3 to give the 4-digit number c0ba. Find the number abc. [2 marks]

2. A point D lies on the side AC of a triangle ABC. Triangle ADB is isosceles with DA = DB. Triangle DBC is
also isosceles with BC = BD. All angles in triangles ADB and DBC are an integer number of degrees. What is
the difference between the largest and smallest values that angle ADB could have? [2 marks]

3. Amy has 14 cousins aged 2, 3, 4, . . . , 15 respectively. She also has some cards separately numbered 16, 17, 18, . . . ,
k, for some integer k. Amy manages to give one card to each cousin so that the number on the card is a multiple
of that cousin’s age. Find the least k for which this is possible. [3 marks]


4. If 3x − 3−x = 285, what is 3x + 3−x ? [3 marks]

5. There are 5 lily pads on a pond, arranged in a circle. A frog can only jump from each lily pad to an adjacent lily
pad on either side. How many ways are there for the frog to start on one of these lily pads, make 11 jumps, and
end up where it started? [3 marks]

6. Find the sum of all (not necessarily distinct) values of a over all triples (a, b, c) of real numbers that satisfy the
equations:

(a + b)(c + 1) = 22 (1)
(a + c)(b + 1) = 22 (2)
(b + c)(a + 1) = 22 (3) [4 marks]

7. A triangle has sides of length x, y, 20 where x > y > 20 and x and y are integers. Let h be the length of the altitude
of the triangle from the side of length 20. Let hx and hy be the lengths of the altitudes from the sides of length x
and y respectively. These altitudes are such that h = hx + hy . Find the perimeter of this triangle. [4 marks]

8. A word is a sequence of zero or more letters taken from the set {A, B, C, D, E, F, G, H, I}. Two words are said
to be related if one can be obtained from the other by a sequence of the following operations:
(a) swapping two adjacent letters,
(b) deleting two adjacent letters which are the same,
(c) inserting two adjacent letters which are the same.
Thus, for instance, the words BACA and BCAA are related by swapping adjacent letters A and C, and BCAA is
related to BC by deleting or inserting two adjacent letter As. Note that the empty word with zero letters is related
to any word consisting of just two adjacent letters.
What is the maximum number of words in a set in which no word is related to any other? [4 marks]

The Olympiad program is supported by the Australian Government Department of Industry, Science, Energy
and Resources through the Science Competitions: Mathematics and Informatics Olympiads grant opportunity.

© 2022 Australian Mathematics Trust

Mathematics Contests The Australian Scene 2022 | Australian Intermediate Mathematics Olympiad Paper | 51
Thursday 8
September 2022

n n
9. Find all positive integers n for which 99 + 9191 is divisible by 100. [5 marks]

10. Identical wind turbines are equally spaced in a straight line on level ground. Each turbine tower is a vertical
cylinder of radius 1 metre. Let L be the line through the base centres of the towers. There is an observer fixed at
O on the same level ground as the towers. Let A be the point on L that is closest to O.
If A is the base centre of a tower and OA = 15 metres, what is the maximum number of towers that are completely
visible to the observer over all possible distances d metres between the base centres of adjacent towers?
[5 marks]

Investigation
If OA = 16 metres and A is not the base centre of a tower, what is the maximum number of towers that are
completely visible to the observer? [4 bonus marks]

The Olympiad program is supported by the Australian Government Department of Industry, Science, Energy
and Resources through the Science Competitions: Mathematics and Informatics Olympiads grant opportunity.

© 2022 Australian Mathematics Trust

Mathematics Contests The Australian Scene 2022 | Australian Intermediate Mathematics Olympiad Paper | 52
Thursday 8
September 2022

2022 Australian Intermediate Mathematics Olympiad - Solutions


Solutions
1. Method 1
When a 3-digit number is multiplied by 3, it cannot be larger than 2997. Hence c is either 1 or 2. It follows that
a = 3c as there would be no carry in the multiplication.
We have 3(100a + 10b + c) = 1000c + 10b + a. Substituting a = 3c gives 903c + 30b = 1003c + 10b which simplifies
to b = 5c. Since b < 10, the only solution is c = 1, b = 5, a = 3.
Therefore abc = 351.

Method 2
When a 3-digit number is multiplied by 3, it cannot be larger than 2997. Hence c is either 1 or 2. It follows that
a = 3c as there would be no carry in the multiplication.
The last digit of 3b is b. Hence b = 0 or 5.
If b = 0, we have 3 × abc = 3 × 301 = 903 ̸= 1003, or 3 × abc = 3 × 602 = 1806 ̸= 2006.
If b = 5, we have 3 × abc = 3 × 351 = 1053 = c0ba, or 3 × abc = 3 × 652 = 1956 ̸= 2056.
Therefore abc = 351.

Method 3
We have 3(100a + 10b + c) = 1000c + 10b + a
20b = 1000c − 3c + a − 300a
b = 50c − 15a + (a − 3c)/20
Since 1 ≤ a, c ≤ 9, we have −26 ≤ a − 3c ≤ 6, hence a − 3c = 0 or −20.
If a − 3c = −20, then b = 50c − 15(3c − 20) − 1 = 5c + 299 ≥ 304, which contradicts b ≤ 9.
If a − 3c = 0, then b = 50c − 45c = 5c. Since b and c are digits, c = 1, b = 5, and a = 3.
Therefore abc = 351.

2. Method 1
Let ̸ ADB = x◦ .

x◦
D

B C

Thus x < 180. Since ̸ BAD = 90 − x2 , x is even. So x ≤ 178.


Since ̸ CBD = 180 − 2(180 − x) = 2x − 180, 2x > 180. So x ≥ 92.
All angles are integers if x = 92 and all angles are integers if x = 178.
Hence the difference between the largest and smallest values of ̸ ADB is 178 − 92 = 86.

The Olympiad program is supported by the Australian Government Department of Industry, Science, Energy
and Resources through the Science Competitions: Mathematics and Informatics Olympiads grant opportunity.

© 2022 Australian Mathematics Trust

Mathematics Contests The Australian Scene 2022 | Australian Intermediate Mathematics Olympiad Paper | 53
Thursday 8
September 2022

Method 2
Let ̸ BAD = y ◦ . Then we have

y◦

D
2y ◦
y◦
2y ◦
B C

Since ̸ CBD = 180 − 4y > 0, we have y < 45, hence 1 ≤ y ≤ 44.


So ̸ ADB = 180 − 2y ≥ 180 − 88 = 92 and ̸ ADB ≤ 180 − 2 = 178.
All angles are integers if ̸ ADB = 92 and all angles are integers if ̸ ADB = 178.
Hence the difference between the largest and smallest values of ̸ ADB is 178 − 92 = 86.

3. We want at least 14 composite numbers in the range 16 to k. The first 14 composites from 16 are 16, 18, 20, 21,
22, 24, 25, 26, 27, 28, 30, 32, 33, 34. Therefore k ≥ 34.
Working from m = 15 down to m = 2, the table shows all multiples of m that can be selected from the range above
and have not been selected for previous values of m.

m multiples m multiples m multiples m multiples


15 30 11 22, 33 7 21 3 27, 33
14 28 10 20 6 18 2 16, 22, 32, 34
13 26 9 18, 27 5 25
12 24 8 16, 32 4 16, 32

The only allowable multiple for 6 is 18, hence 27 for 9, 33 for 3, and 22 for 11. Choosing 16 as the multiple for 8,
leaves only 32 for 4, and 34 for 2. So we have 14 acceptable multiples:
30, 28, 26, 24, 22, 20, 27, 16, 21, 18, 25, 32, 33, 34.
Therefore the minimum value of k is 34.
Comment
Choosing 32 as the multiple for 8 gives the only other acceptable distribution from the 14 cards:
30, 28, 26, 24, 22, 20, 27, 32, 21, 18, 25, 16, 33, 34.

4. Method 1
Observe that 285 = (3x − 3−x )2 = 32x + 3−2x − 2.
Also (3x + 3−x )2 = 32x + 3−2x + 2 = 285 + 4 = 289.

Since 3x and 3−x are positive, we have 3x + 3−x = 289 = 17.

Method 2
Let 3x + 3−x = a. √ √
Adding 3x − 3−x = 285 gives 2 × 3x = a + 285.
√ √
Subtracting 3x − 3−x = 285 gives 2 × 3−x = a − 285.

Multiplying gives 4 = a2 − 285. Since a > 0, a = 289 = 17.

The Olympiad program is supported by the Australian Government Department of Industry, Science, Energy
and Resources through the Science Competitions: Mathematics and Informatics Olympiads grant opportunity.

© 2022 Australian Mathematics Trust

Mathematics Contests The Australian Scene 2022 | Australian Intermediate Mathematics Olympiad Paper | 54
4
Thursday 8
September 2022

Method 3
√ √
Let t = 3x , then t − 1/t = 285, hence t2 − 285t − 1 = 0.
√ √
Since 3x > 0, we have t = 12 ( 285 + 285 + 4)
√ √
= 12 ( 285 + 289)

= 12 ( 285 + 17)

So 3x + 3−x = 12 ( 285 + 17) + √285+17
2

√ √
2( 285−17)
= 12 ( 285 + 17) + (√285+17)( √
285−17)
1
√ √
2( 285−17)
= 2 ( 285 + 17) + 285−172
√ √
= 12 ( 285 + 17) − 12 ( 285 − 17)
= 17
Method 4
√ √
Let t = 3x , then t − 1/t = 285, hence t2 − 285t − 1 = 0.
√ √ √ √ √
The roots of this quadratic are 12 ( 285 ± 285 + 4) = 12 ( 285 ± 289) = 12 ( 285 ± 17).
Since the product of these roots√is −1 (the constant
√ coefficient), the roots are t and −t−1 .
x −x −1 1 1
Hence 3 + 3 = t + t = 2 ( 285 + 17) − 2 ( 285 − 17) = 17.

5. Method 1
Suppose, in some order, the frog makes c clockwise jumps and 11 − c anticlockwise jumps. The net number of
anticlockwise jumps is 11 − 2c. Hence, the frog ends where it started if and only if 11 − 2c is divisible by 5. Since
0 ≤ c ≤ 11, the only values of c are 3 and 8.
Suppose the frog makes 3 clockwise and 8 anticlockwise jumps. Let k be the number of jumps between the first
and third clockwise jumps. For a given value of k = 1, 2, 3, . . . , 9, the first and third clockwise jumps could occupy
10 − k pairs of positions. For each of these positions, the second clockwise jump could occupy k positions. So the
number of ways the frog could make 3 clockwise and 8 anticlockwise jumps is

(9 × 1) + (8 × 2) + (7 × 3) + (6 × 4) + (5 × 5) + (4 × 6) + (3 × 7) + (2 × 8) + (1 × 9) = 165.

Similarly, the number of ways the frog could make 8 clockwise and 3 anticlockwise jumps is 165. So the number of
ways for the frog to make the 11 jumps is 165 + 165 = 330.
Comment
Alternatively, for students
  who are familiar with binomial coefficients, the number of ways exactly 311of
 the 11
jumps are clockwise is 113 = 165, and the number of ways exactly 8 of the 11 jumps are clockwise is 8 = 165.

The Olympiad program is supported by the Australian Government Department of Industry, Science, Energy
and Resources through the Science Competitions: Mathematics and Informatics Olympiads grant opportunity.

© 2022 Australian Mathematics Trust

Mathematics Contests The Australian Scene 2022 | Australian Intermediate Mathematics Olympiad Paper | 55
Thursday 8
September 2022

Method 2
Label the lily pads A, B, C, D, E in rotational order as shown. Let an , bn , cn , dn , en be the number of ways the frog
can take n jumps from A to A, B, C, D, E, respectively. We may assume the frog starts at pad A. So the aim is to
find a11 .
A

B E

C D

As the frog may only jump from a given lily pad to an adjacent lily pad,

an = en−1 + bn−1
bn = an−1 + cn−1
cn = bn−1 + dn−1
dn = cn−1 + en−1
en = dn−1 + an−1 .

As the frog starts at A, by symmetry we have that bn = en and cn = dn . Therefore we need only consider just
three recursive equations an = 2bn−1 , bn = an−1 + cn−1 and cn = bn−1 + cn−1 , together with the initial condition
(a1 , b1 , c1 ) = (0, 1, 0).
From these we generate this table:

n 1 2 3 4 5 6 7 8 9 10 11
an 0 2 0 6 2 20 14 70 72 254 330
bn 1 0 3 1 10 7 35 36 127 167 474
cn 0 1 1 4 5 15 22 57 93 220 385

Thus a11 = 330.

Method 3
Label the lily pads A, B, C, D, E in rotational order. Represent each of the lily pads with a column of 12 circles
as shown. Draw a line connecting a circle in a row to a circle in the next row if the frog can jump from one
corresponding lily pad to the next in one step. We may assume the frog starts at C. A number k in a circle in row
n is the number of ways for the frog to reach the corresponding lily pad from C in n jumps.
All circles in row 0 have the number 0, the circles in row 1 have the numbers 0, 1, 0, 1, 0 respectively, and the
number in a circle in any other row is the sum of the numbers in the two adjacent circles in the previous row. We
want the number in circle C in row 11.

The Olympiad program is supported by the Australian Government Department of Industry, Science, Energy
and Resources through the Science Competitions: Mathematics and Informatics Olympiads grant opportunity.

© 2022 Australian Mathematics Trust

Mathematics Contests The Australian Scene 2022 | Australian Intermediate Mathematics Olympiad Paper | 56
Thursday 8
September 2022

A B C D E

n=0 0 0 0 0 0

n=1 0 1 0 1 0

n=2 1 0 2 0 1

n=3 1 3 0 3 1

n=4 4 1 6 1 4

n=5 5 10 2 10 5

n=6 15 7 20 7 15

n=7 22 35 14 35 22

n=8 57 36 70 36 57

n=9 93 127 72 127 93

n = 10 220 167 254 167 220

n = 11 385 474 330 474 385

Thus the number of ways for the frog to take 11 jumps from C to C is 330.

6. Method 1
From (2) and (1) we have (a + b)(c + 1) = (a + c)(b + 1)
ac + a + bc + b = ab + a + bc + c
a(c − b) + (b − c) = 0
(a − 1)(c − b) = 0

From (3) and (1) we have (a + b)(c + 1) = (b + c)(a + 1)


ac + a + bc + b = ab + b + ac + c
a(b − 1) + c(1 − b) = 0
(b − 1)(a − c) = 0

So a = 1 or b = c, and b =The Olympiad program is supported by the Australian Government Department of Industry, Science, Energy
1 or a = c.
and Resources through the Science Competitions: Mathematics and Informatics Olympiads grant opportunity.
If a = 1 and b = 1, then the original
© 2022 equations
Australian give Trust
Mathematics 2(c + 1) = 22, hence c = 10.
If a = 1 and a = c, then the original equations give 2(b + 1) = 22, hence b = 10.
Mathematics Contests The Australian Scene 2022 | Australian Intermediate Mathematics Olympiad Paper | 57
If b = c and b = 1, then the original equations give 2(a + 1) = 22, hence a = 10.
n = 10

n = 11 385 474 330 474 385 Thursday 8


September 2022
Thus the number of ways for the frog to take 11 jumps from C to C is 330.

6. Method 1
From (2) and (1) we have (a + b)(c + 1) = (a + c)(b + 1)
ac + a + bc + b = ab + a + bc + c
a(c − b) + (b − c) = 0
(a − 1)(c − b) = 0

From (3) and (1) we have (a + b)(c + 1) = (b + c)(a + 1)


ac + a + bc + b = ab + b + ac + c
a(b − 1) + c(1 − b) = 0
(b − 1)(a − c) = 0

So a = 1 or b = c, and b = 1 or a = c.
If a = 1 and b = 1, then the original equations give 2(c + 1) = 22, hence c = 10.
If a = 1 and a = c, then the original equations give 2(b + 1) = 22, hence b = 10.
If b = c and b = 1, then the original equations give 2(a + 1) = 22, hence a = 10.

If b = c and a = c, then the original equations give 2a(a + 1) = 22, hence a = − 21 ± 3
2 5.

Thus there are only 5 solutions for (a, b, c): 7


√ √ √
(1, 1, 10), (1, 10, 1), (10, 1, 1), (− 12 ± 3
2 5, − 12 ± 3
2 5, − 12 ± 3
2 5).
√ √
So the sum of all values of a is 1 + 1 + 10 + (− 12 + 3
2 5) + (− 12 − 3
2 5) = 11.

Method 2
From (2) and (1) we have (a + b)(c + 1) = (a + c)(b + 1)
ac + a + bc + b = ab + a + bc + c
a(c − b) + (b − c) = 0
(a − 1)(b − c) = 0 (4)

Since equations (1), (2), (3) are symmetric in a, b, c, we also have:

(b − 1)(a − c) = 0 (5)
(c − 1)(a − b) = 0 (6)

If no two of a, b, c are equal, then a = 1, b = 1, and c = 1, a contradiction.


If a = b ̸= c, then equation (4) gives a = 1. Then equation (1) gives 2(c + 1) = 22, hence c = 10.
So (a, b, c) = (1, 1, 10).
By symmetry, if a = c ̸= b, then (a, b, c) = (1, 10, 1), and if b = c ̸= a, then (a, b, c) = (10, 1, 1).

If a = b = c, then equation (1) gives a(a + 1) = 11, hence a = − 21 ± 32 5.
Thus there are only 5 solutions for (a, b, c):
√ √ √
(1, 1, 10), (1, 10, 1), (10, 1, 1), (− 12 ± 3
2 5, − 12 ± 3
2 5, − 12 ± 3
2 5).
√ √
So the sum of all values of a is 1 + 1 + 10 + (− 12 + 3
2 5) + (− 12 − 3
2 5) = 11.

1 1 1
7. The area of the triangle is 2 × 20 × h = 2 × x × hx = 2 × y × hy .

The Olympiad program is supported by the Australian Government Department of Industry, Science, Energy
and Resources through the Science Competitions: Mathematics and Informatics Olympiads grant opportunity.

© 2022 Australian Mathematics Trust

Mathematics Contests The Australian Scene 2022 | Australian Intermediate Mathematics


h Olympiad Paper | 58
2 2
Thus there are only 5 solutions for (a, b, c):
√ √ √
(1, 1, 10), (1, 10, 1), (10, 1, 1), (− 12 ± 3
2 5, − 12 ± 3
2 5, − 12 ± 3
2
Thursday
5). 8
√ √ September 2022
So the sum of all values of a is 1 + 1 + 10 + (− 12 + 3
2 5) + (− 12 − 3
2 5) = 11.

1 1 1
7. The area of the triangle is 2 × 20 × h = 2 × x × hx = 2 × y × hy .

h
hy
hx y
20

1 1
Since h = hx + hy , we have 20 = x + y1 .

Method 1
20x 400
Hence y = x−20 = x−20 + 20. Therefore, x − 20 is a factor of 400.
20x

A triangle inequality gives 20 + > x, hence x2 − 60x + 400 < 0 and x < 10( 5 + 3) < 53.
x−20
20x

A triangle inequality gives x + 20 > x−20 , hence x2 − 20x − 400 > 0 and x > 10( 5 + 1) > 32.
So 12 < x − 20 < 33. Hence x − 20 = 16, 20, or 25.
Therefore x = 36, 40, or 45, with y = 45, 40, or 36 respectively.
Since x > y, we have x = 45, y = 36 and the perimeter is 20 + 36 + 45 = 101.

Method 2
1
From 20 = x1 + y1 we have (x − 20)(y − 20) = 400.
Thus x − 20 and y − 20 are cofactors of 400.

Since x > y, x − 20 > 400 = 20 and y − 20 < 20.
8
A triangle inequality gives x < 20 + y.
Thus 20 < x − 20 < y < 40.
So x − 20 = 25 and y − 20 = 16.
Hence x = 45, y = 36, and the perimeter is 45 + 36 + 20 = 101.

8. Method 1
A word that has its letters in alphabetical order and no repeated letter is called a standard word. Applying
operations (a) and (b) shows that any word relates to a standard word. We claim that two words are related if
and only if they relate to the same standard word.
First suppose that words X and Y relate to the same standard word S. Let Q denote a sequence of operations
(a) and (b) that converts Y to S. Let Q′ be the sequence Q with each operation (b) replaced with (c). Then the
sequence of operations that convert X to S followed by Q′ , converts X to Y . Thus X and Y are related.
Conversely, suppose two words X and Y are related. None of the operations (a), (b), (c) change the parity of the
number of times a letter L appears from one word to the next. So L appears an odd number of times in X if and
only if it appears an odd number of times in Y . Also, a standard form that relates to X has exactly those letters in
X that appear an odd number of times in X. The same applies to Y . Hence X and Y relate to the same standard
form.
So the claim is true. In particular no two standard words are related. Hence the maximum number of unrelated
words in a set is the number of standard words. We count the number of standard words by considering each of
the nine letters in turn: it may be either included or excluded. So the maximum number of unrelated words is
29 = 512.
The Olympiad program is supported by the Australian Government Department of Industry, Science, Energy
Method 2 and Resources through the Science Competitions: Mathematics and Informatics Olympiads grant opportunity.

We show that two words X© 2022and Australian Mathematics Trust


Y are related if and only if, for each letter L, the parity of the number of times L
appears Contests
Mathematics in X is the
The same as the
Australian parity
Scene 2022of| Australian
the number of times Mathematics
Intermediate L appears in Y.
Olympiad Paper | 59

First suppose that X and Y are related. None of the operations (a), (b), (c) change the parity of the number of

Since x > y, x − 20 > 400 = 20 and y − 20 < 20.
A triangle inequality gives x < 20 + y.
Thus 20 < x − 20 < y < 40. Thursday 8
So x − 20 = 25 and y − 20 = 16. September 2022
Hence x = 45, y = 36, and the perimeter is 45 + 36 + 20 = 101.

8. Method 1
A word that has its letters in alphabetical order and no repeated letter is called a standard word. Applying
operations (a) and (b) shows that any word relates to a standard word. We claim that two words are related if
and only if they relate to the same standard word.
First suppose that words X and Y relate to the same standard word S. Let Q denote a sequence of operations
(a) and (b) that converts Y to S. Let Q′ be the sequence Q with each operation (b) replaced with (c). Then the
sequence of operations that convert X to S followed by Q′ , converts X to Y . Thus X and Y are related.
Conversely, suppose two words X and Y are related. None of the operations (a), (b), (c) change the parity of the
number of times a letter L appears from one word to the next. So L appears an odd number of times in X if and
only if it appears an odd number of times in Y . Also, a standard form that relates to X has exactly those letters in
X that appear an odd number of times in X. The same applies to Y . Hence X and Y relate to the same standard
form.
So the claim is true. In particular no two standard words are related. Hence the maximum number of unrelated
words in a set is the number of standard words. We count the number of standard words by considering each of
the nine letters in turn: it may be either included or excluded. So the maximum number of unrelated words is
29 = 512.

Method 2
We show that two words X and Y are related if and only if, for each letter L, the parity of the number of times L
appears in X is the same as the parity of the number of times L appears in Y .
First suppose that X and Y are related. None of the operations (a), (b), (c) change the parity of the number of
times L appears from one word to the next. So the parities of L agree.
Conversely, suppose the parities agree. Operation (b) decreases the number of times a letter appears by 2. Operation
(c) increases the number of times a letter appears by 2. So operations (a), (b), (c) can change X to a word that has
each letter appearing the same number of times that it does in Y , and then operation (a) can arrange the letters
so they are in the same order as in Y . So X and Y are related.
Define the signature of a word to be the binary sequence of 0s and 1s whose kth term has the same parity as the
parity of the number of times the kth letter in the sequence A, B, C, D, E, F, G, H, I appears in the word. So two
words relate if and only if they have the same signature. Hence the maximum number of unrelated words in a set
is the number of signatures. We count the number of signatures by considering each of the terms in turn: it may
be either 0 or 1. So the maximum number of unrelated words is 29 = 512.

9. Method 1
First observe that, modulo 100, we have:
91 = 9, 92 = 81, 93 ≡ 29, 94 ≡ 61, 95 ≡ 49, 96 ≡ 41, 97 ≡ 69, 98 ≡ 21, 99 ≡ 89, 910 ≡ 1
and
911 = 91, 912 = 81, 913 ≡ 71, 914 ≡ 61, 915 ≡ 51, 916 ≡ 41, 917 ≡ 31, 918 ≡ 21, 919 ≡ 11, 9110 ≡ 1.
Hence, for all integers k ≥ 0, we have:
910k ≡ 1, 910k+1 = 9, 910k+2 = 81, 910k+3 ≡ 29, 910k+4 ≡ 61,
910k+5 ≡ 49, 910k+6 ≡ 41, 910k+7 ≡ 69, 910k+8 ≡ 21, 910k+9 ≡ 89
and
9110k ≡ 1, 9110k+1 = 91, 9110k+2 = 81, 9110k+3 ≡ 71, 9110k+4 ≡ 61,
9110k+5 ≡ 51, 9110k+6 ≡ 41, 9110k+7 ≡ 31, 9110k+8 ≡ 21, 9110k+9 ≡ 11.
Thus, if n is even, then 9n and 91n are both congruent to 1 modulo 10, hence
n n
99 + 9191 ≡ 9 + 91 mod 100 ≡ 0 mod 100.

The Olympiad program is supported by the Australian Government Department of Industry, Science, Energy
and Resources through the Science Competitions: Mathematics and Informatics Olympiads grant opportunity.

© 2022 Australian Mathematics Trust

Mathematics Contests The Australian Scene 2022 | Australian Intermediate Mathematics Olympiad Paper | 60
Thursday 8
September 2022

If n is odd, then 9n is congruent to 9 modulo 10 and 91n is congruent to 1 modulo 10, hence
n n
99 + 9191 ≡ 89 + 91 mod 100 = 80 mod 100.
n n
Thus 99 + 9191 is divisible by 100 if and only if n is even.

Method 2
We work modulo 100 throughout the following.
First observe that:
91 = 9, 92 = 81, 93 ≡ 29, 94 ≡ 61, 95 ≡ 49, 96 ≡ 41, 97 ≡ 69, 98 ≡ 21, 99 ≡ 89, 910 ≡ 1
and
911 = 91, 912 = 81, 913 ≡ 71, 914 ≡ 61, 915 ≡ 51, 916 ≡ 41, 917 ≡ 31, 918 ≡ 21, 919 ≡ 11, 9110 ≡ 1.
n 2 n−2 n−2 n−2 n−2 n−2
99 = (99 )9 = (981 )9 = (980 × 9)9 ≡ (1 × 9)9 = 99 .
0
n 99 = 9 if n is even
So 99 ≡ 1
99 ≡ 89 if n is odd
n 2 n−2 n−2 n−2 n−2 n−2
9191 = (9191 91
 ) 0 = (91
8281 91
) = (918280 × 91)91 ≡ (1 × 91)91 = 9191 .
n 9191 = 91 if n is even
So 9191 ≡ 1
9191 ≡ 91 if n is odd

n n 9 + 91 = 100 if n is even
Hence 99 + 9191 ≡
89 + 91 ≡ 80 if n is odd
n n
Thus 99 + 9191 is divisible by 100 if and only if n is even.

Method 3
We work modulo 100 throughout the following.
First observe that:
91 = 9, 92 = 81, 93 ≡ 29, 94 ≡ 61, 95 ≡ 49, 96 ≡ 41, 97 ≡ 69, 98 ≡ 21, 99 ≡ 89, 910 ≡ 1
and
911 = 91, 912 = 81, 913 ≡ 71, 914 ≡ 61, 915 ≡ 51, 916 ≡ 41, 917 ≡ 31, 918 ≡ 21, 919 ≡ 11, 9110 ≡ 1.
n n n n
99 + 9191 ≡ 99 + (−9)91
n n n
= 99 + (−1)91 991
n n
= 99 − 991
n n
−9n
= 99 (1 − 991 )

Now 9100q+r = (9100 )q × 9r ≡ 1q × 9r = 9r .


n
−9n
If n is even, then 91n − 9n ≡ 0, hence 991 ≡ 90 .
n n n
Therefore 99 + 9191 ≡ 99 (1 − 1) = 0.
If n is odd, then 91n − 9n is congruent to one of
91 − 9 = 82, 71 − 29 = 42, 51 − 49 = 2, 31 − 69 = −38 ≡ 62, 11 − 89 = −78 ≡ 22.
n n
Hence 991 −9 is congruent to one of 92 , 922 , 942 , 962 , 982 , all of which are congruent to 92 = 81.
n n n n
So 99 + 9191 ≡ 99 (1 − 81) ≡ 20 × 99 .
Now 9n is equivalent to one of 99 , 929 , 949 , 969 , 989 , all of which are congruent to 99 ≡ 89.
n n
Hence 99 + 9191 ≡ 20 × 89 = 1780 ≡ 80.
n n
Thus 99 + 9191 is divisible by 100 if and only if n is even.

Method 4
n n
We first show that 99 + 9191 is divisible by 4 for all positive integers n.
Modulo 4, 9k ≡ 1 and 91k ≡ 3 for all odd k.
n n
Since 9n and 91n are odd for all n, 99 + 9191 ≡ 1 + 3 ≡ 0 for all n.
n n
Next we show that 99 + 91
The91Olympiad program is supported by the Australian Government Department of Industry, Science, Energy
is divisible by 25 if and only if n is even.
and Resources through the Science Competitions: Mathematics and Informatics Olympiads grant opportunity.
Modulo 25, we have:
© 2022 Australian Mathematics Trust
91 = 9, 92 ≡ 6, 93 ≡ 4, 94 ≡ 11, 95 ≡ 24, 96 ≡ 16, 97 ≡ 19, 98 ≡ 21, 99 ≡ 14, 910 ≡ 1
and
Mathematics Contests The Australian Scene 2022 | Australian Intermediate Mathematics Olympiad Paper | 61
911 = 16, 912 = 6, 913 ≡ 21, 914 ≡ 11, 915 ≡ 1.
If n is odd, then 91 − 9 is congruent to one of
91 − 9 = 82, 71 − 29 = 42, 51 − 49 = 2, 31 − 69 = −38 ≡ 62, 11 − 89 = −78 ≡ 22.
n n
Hence 991 −9 is congruent to one of 92 , 922 , 942 , 962 , 982 , all of which are congruent to 92 = 81.
Thursday 8
n n n n
So 99 + 9191 ≡ 99 (1 − 81) ≡ 20 × 99 .
September
Now 9n is equivalent to one of 99 , 929 , 949 , 969 , 989 , all of which are congruent to 99 ≡ 89.
n n 2022
Hence 99 + 9191 ≡ 20 × 89 = 1780 ≡ 80.
n n
Thus 99 + 9191 is divisible by 100 if and only if n is even.

Method 4
n n
We first show that 99 + 9191 is divisible by 4 for all positive integers n.
Modulo 4, 9k ≡ 1 and 91k ≡ 3 for all odd k.
n n
Since 9n and 91n are odd for all n, 99 + 9191 ≡ 1 + 3 ≡ 0 for all n.
n n
Next we show that 99 + 9191 is divisible by 25 if and only if n is even.
Modulo 25, we have:
91 = 9, 92 ≡ 6, 93 ≡ 4, 94 ≡ 11, 95 ≡ 24, 96 ≡ 16, 97 ≡ 19, 98 ≡ 21, 99 ≡ 14, 910 ≡ 1
and
911 = 16, 912 = 6, 913 ≡ 21, 914 ≡ 11, 915 ≡ 1.
Hence 920 ≡ 1 ≡ 9120 mod 25.

Modulo 20, we have 91 = 9, 92 ≡ 1, and 911 = 11, 91210≡ 1.


Hence, for even n, we have:
n n
99 = 920r+1 = (920 )r × 9 ≡ 9 mod 25 and 9191 = 9120r+1 = (9120 )r × 91 ≡ 91 ≡ 16 mod 25.
n n
Hence 99 + 9191 ≡ 9 + 16 ≡ 0 mod 25.
For odd n, we have:
n n
99 = 920r+9 = (920 )r × 99 ≡ 14 mod 25 and 9191 = 9120r+11 = (9120 )r × 9111 ≡ 16 mod 25.
n n
Hence 99 + 9191 ≡ 14 + 16 ≡ 5 mod 25.
n n
Thus 99 + 9191 is divisible by 100 if and only if n is even.

10. Suppose a tower with centre C is completely visible to the observer but that tower obscures part of the next
tower away from O. By drawing lines through O tangential to the towers, we see that all towers from A to C are
completely visible from O but none of the towers to the right of C is visible. Number the tower centres from A to
C: 0, 1, 2, . . ., m. We first show that m ≤ 7.

Method 1
Line OZ is tangential at X to the (m − 1)th tower with centre B as shown. It intersects L at Y and CZ is
perpendicular to OZ.

(m − 1)d
Z
A B Y
• • • • • •
1 C
X

15


O

Triangles BXY and CY Z are similar, hence BY ≤ Y C. So 2BY ≤ BC = d.


Triangles BXY and OAY are similar, hence OY /15 = BY /1.
Since there is a tower centre at A, we have
AY = AB + BY = (m − 1)d + BY ≥ (2m − 1)BY = (2m − 1)OY /15.
Since AY /OY < 1, we have 15 > 2m − 1, hence m ≤ 7.

Method 2
Between the (m − 1)th tower, with centre
The Olympiad B,isand
program the mth
supported tower,
by the with
Australian centre C,Department
Government draw a line that isScience,
of Industry, tangential
Energyto
and Resources as
both towers at X and Z respectively through the Science
shown. Let O1Competitions:
be a pointMathematics and 15
on that line Informatics
metresOlympiads
from L. grant
Let opportunity.
A1 be the
point on L closest to O1 . Note
© 2022that O1 AMathematics
Australian 1 will be on the left of OA.
Trust

Mathematics Contests The Australian Scene 2022 | Australian Intermediate Mathematics Olympiad Paper | 62
(m − 1)d
O

Triangles BXY and CY Z are similar, hence BY ≤ Y C. So 2BY ≤ BC = d.


Triangles BXY and OAY are similar, hence OY /15 = BY /1. Thursday 8
Since there is a tower centre at A, we have September 2022
AY = AB + BY = (m − 1)d + BY ≥ (2m − 1)BY = (2m − 1)OY /15.
Since AY /OY < 1, we have 15 > 2m − 1, hence m ≤ 7.

Method 2
Between the (m − 1)th tower, with centre B, and the mth tower, with centre C, draw a line that is tangential to
both towers at X and Z respectively as shown. Let O1 be a point on that line 15 metres from L. Let A1 be the
point on L closest to O1 . Note that O1 A1 will be on the left of OA.

(m − 1)d
Z
A1 A B Y
• • • •
1 C
X

15 15


O1 O
Triangles BXY and CY Z are similar, hence BY = Y C. So 2BY = d.
Triangles BXY and O1 A1 Y are similar, hence A1 Y = 15XY and O1 Y = 15BY .

(m − 12 )d.
We have A1 Y ≥ AY = AB + BY = (m − 1)d + d/2 =11
So (m − 2 ) d ≤ A1 Y = 225XY = 225(BY − 1) = 225( 14 d2 − 1) < 225d2 /4.
1 2 2 2 2 2

Hence m < 15/2 + 1/2 = 8, so m ≤ 7.

Method 3
Between the (m − 1)th tower, with centre B, and the mth tower, with centre C, draw a line that is tangential to
both towers at X and Z respectively as shown. Let that line intersect OA at P .

(m − 1)d
Z
A B Y
• • • •
1 C
X

P

O

Triangles BXY and CY Z are similar, hence BY = Y C. So 2BY = d.


Triangles BXY and P AY are similar, hence AP = AY /XY .
So we have
AY (m − 1)d + d/2
15 ≥ = 
XY d2 /4 − 1

15 d2 − 4 ≥ 2(m − 1)d + d = (2m − 1)d

m ≤ (15 1 − 4/d2 + 1)/2

= 15 1/4 − 1/d2 + 1/2
< 15/2 + 1/2 = 8
The Olympiad program is supported by the Australian Government Department of Industry, Science, Energy
and Resources through the Science Competitions: Mathematics and Informatics Olympiads grant opportunity.
Hence m ≤ 7.
© 2022 Australian Mathematics Trust
Next we show that m = 7 is possible by finding an acceptable value for d that allows the following configuration.
Mathematics Contests The Australian Scene 2022 | Australian Intermediate Mathematics Olympiad Paper | 63
6d d
P

Thursday 8
O September 2022

Triangles BXY and CY Z are similar, hence BY = Y C. So 2BY = d.


Triangles BXY and P AY are similar, hence AP = AY /XY .
So we have
AY (m − 1)d + d/2
15 ≥ = 
XY d2 /4 − 1

15 d2 − 4 ≥ 2(m − 1)d + d = (2m − 1)d

m ≤ (15 1 − 4/d2 + 1)/2

= 15 1/4 − 1/d2 + 1/2
< 15/2 + 1/2 = 8

Hence m ≤ 7.
Next we show that m = 7 is possible by finding an acceptable value for d that allows the following configuration.

6d d
Z
A B Y
• • •
1 C
X

15

As above, from similar triangles, d = 2BY and OY = 15BY . We have

OY 2 = OA2 + AY 2
225(d/2)2 = 225 + (13d/2)2
225d2 = 900 + 169d2

Since d > 2, it is possible to have m = 7. d2 = 900/56 > 4

Thus the maximum number of towers on the right of A and completely visible at O is 7. By symmetry, the
maximum number of towers on the left of A and completely visible at O is 7. Including the tower at A, the
maximum number of towers completely visible at O is 2 × 7 + 1 = 15.

12

The Olympiad program is supported by the Australian Government Department of Industry, Science, Energy
and Resources through the Science Competitions: Mathematics and Informatics Olympiads grant opportunity.

© 2022 Australian Mathematics Trust

Mathematics Contests The Australian Scene 2022 | Australian Intermediate Mathematics Olympiad Paper | 64
Thursday 8
September 2022

Investigation
If there is no tower centre at A, let the distance between A and the closest tower centre on the right of A be αd
where 0 < α < 1. Suppose the observer can completely see exactly m towers to the right of A and m′ towers to
the left of A.

αd (m − 2)d
Z
A B Y
• • • •
1 C
X

16


O

As above, from similar triangles, d ≥ 2BY and OY = 16BY .


Now AY = αd + (m − 2)d + BY ≥ (2α + 2(m − 2) + 1)BY = (2α + 2m − 3)OY /16.
Since AY /OY < 1, we have m < 9.5 − α. Similarly m′ < 9.5 − (1 − α) = 8.5 + α.
If α = 1/2, then m ≤ 8 and m′ ≤ 8.
If α < 1/2, then m ≤ 9 and m′ ≤ 8.
If α > 1/2, then m ≤ 8 and m′ ≤ 9.
Next we show that these upper bounds on m are attainable by finding an acceptable value for d that allows the
following configurations.

αd 6d or 7d d
Z
A B Y
• • •
1 C
X

16

As above, from similar triangles, d = 2BY and OY = 16BY . We have

OY 2 = OA2 + AY 2
256(d/2)2 = 256 + d2 (α + (6 or 7) + 12 )2
256d2 = 1024 + d2 (2α + (13 or 15))2

With 1 > α ≥ 1/2, we have 256d2 = 1024 + d2 (2α + 13)2 . So d2 = 1024/(256 − (2α + 13)2 ).
With 0 < α < 1/2, we have 256d2 = 1024 + d2 (2α + 15)2 . So d2 = 1024/(256 − (2α + 15)2 ).
In each case there is a value for d > 2. So it is possible for m to attain its upper bounds. Similarly, the upper
bounds for m′ are attainable.
So the maximum number of completely visible towers is 8 + 8 = 16 if α = 1/2 and 8 + 9 = 17 if α ̸= 1/2.

The Olympiad program is supported by the Australian Government Department of Industry, Science, Energy
and Resources through the Science Competitions: Mathematics and Informatics Olympiads grant opportunity.

© 2022 Australian Mathematics Trust

Mathematics Contests The Australian Scene 2022 | Australian Intermediate Mathematics Olympiad Paper | 65
2022 Australian Intermediate Mathematics Olympiad
Statistics

Distribution of Awards/School Year

Number of Awards
Number of
Year
Students High
Prize Distinction Credit Participation
Distinction

8 476 2 19 40 125 290

9 539 6 51 53 172 257

10 493 16 71 83 153 170

Other 455 5 11 28 92 319

All Years 1963 29 152 204 542 1036

The award distribution is based on approximately the top 10% for High Distinction, next 15% for
Distinction and the following 25% for Credit.

Number of Correct Answers Questions 1–8

Number Correct / Question


Year
1 2 3 4 5 6 7 8

8 451 174 253 161 25 37 32 32

9 520 224 291 230 69 63 45 52

10 478 220 299 288 90 61 66 97

Other 430 109 202 110 29 41 21 26

All Years 1879 727 1045 789 213 202 164 207

Mean Score/Question/School Year

Question
Number of
School Year Overall Mean
Students
1–8 9 10

8 476 6.9 0.4 0.0 7.3

9 539 8.3 0.6 0.1 9.0

10 493 10.1 1.0 0.2 11.3

Other 455 6.0 0.3 0.0 6.4

All Years 1963 7.9 0.6 0.1 8.6

Mathematics Contests The Australian Scene 2022 | Australian Intermediate Mathematics Olympiad Statistics | 66
Australian Mathematical
Olympiad Training Program
About the Program

The Australian Mathematical Olympiad Committee


In 1980, a group of distinguished mathematicians formed the Australian Mathematical Olympiad
Committee (AMOC) to coordinate an Australian entry in the International Mathematical Olympiad (IMO).

Since then, AMOC has developed a comprehensive program to enable all students (not only the few
who aspire to national selection) to enrich and extend their knowledge of mathematics. The activities
in this program are not designed to accelerate students. Rather, the aim is to enable students to
broaden their mathematical experience and knowledge.

The largest of these activities is the Mathematics for Young Australians (MYA) program. The Maths
Challenge is a problem-solving event held in the first or second term, in which approximately 15,000
young Australians explore carefully developed mathematical problems. Students who wish to continue
to extend their mathematical experience can then participate in the Maths Enrichment and after that,
they may attempt the Australian Intermediate Mathematics Olympiad (AIMO), which is the third stage
of this program.

Originally AMOC was a subcommittee of the Australian Academy of Science. In 1992 it collaborated
with the Australian Mathematics Foundation (which organises the Australian Mathematics
Competition) to form the Australian Mathematics Trust. The Trust is based in Canberra with
representatives on our board from a range of mathematical organisations. We are supported by a vast
network of passionate volunteers from Australia and around the world, including academics, members
of professional mathematics societies, educators and working mathematicians.

The AMOC has responsibility for the MYA program, and many open and invitational mathematics
competitions, culminating in the selection of a team of students who represent Australia internationally
each year.

2022 Australian Mathematical Olympiad Committee


Chair
Norman Do, Monash University, VIC
Deputy Chair
Ms Thanom Shaw, SCEGGS Darlinghurst, NSW
Chief Executive Officer
Mr Nathan Ford, Australian Maths Trust, ACT
Mathematician
Mr Mike Clapper, Australian Maths Trust, ACT
Treasurer
Dr P Swedosh, The King David School, VIC
Chair, Senior Problems Committee
Dr Ivan Guo, Monash University, VIC
Chair, Challenge
Dr Kevin McAvaney, VIC
Director of Training and IMO Team Leader
Dr Angelo Di Pasquale, University of Melbourne, VIC
IMO Deputy Team Leader
Mr Sampson Wong, University of Sydney, NSW
EGMO Team Leader
Ms Thanom Shaw, SCEGGS Darlinghurst, NSW

Mathematics Contests The Australian Scene 2022 | About the Program | 68


State Directors
Dr G Gamble, University of Western Australia
Mr Nhat Anh Hoang, WA Deputy
Dr I Roberts, Northern Territory
Dr D Badziahin, University of Sydney, NSW
Mr M Bammann, South Australia
Dr B Gray, Queensland
Dr P Swedosh, The King David School, VIC
Dr Chris Wetherell, AMT Senior Mathematician, ACT
Mr Andy Tran – NSW Deputy

The AMOC Senior Problems Committee


This committee has been in existence for many years and carries out a number of roles. A central role
is the collection and moderation of problems for senior and exceptionally gifted intermediate and
junior secondary school students. Each year the Problems Committee provides examination papers
for the AMOC Senior Contest and the Australian Mathematical Olympiad. In addition, problems
are submitted for consideration to the Problem Selection Committees of the annual Asian Pacific
Mathematics Olympiad, the European Girls’ Mathematical Olympiad and the International Mathematical
Olympiad.

2022 AMOC Senior Problems Committee


Mr Mike Clapper, Mathematician, Australian Maths Trust, Victoria
Miss Michelle Chen, Melbourne University, Victoria
Dr Alice Devillers, University of Western Australia Western Australia
Dr Angelo Di Pasquale, University of Melbourne, Victoria
Dr Norman Do, Monash University, Victoria
Dr Ivan Guo, Monash University, Victoria
Dr Kevin McAvaney, Deakin University, Victoria
Dr Daniel Mathews, Monash University, Victoria
Mr Andrew Elvey Price, Melbourne University, Victoria
Dr Chaitanya Rao, Victoria
Ms Thanom Shaw, SCEGGS Darlinghurst, New South Wales
Dr Ian Wanless, Monash University, Victoria
Dr Chris Wetherell, Australian Maths Trust, Australian Capital Territory
Mr Sampson Wong, University of Sydney, New South Wales
Dr Yiying (Sally) Victoria

Activities of the Australian Mathematical Olympiad Training Program

2022 Australian Mathematical Olympiad


The Australian Mathematical Olympiad (AMO) consists of two papers of four questions each and was
sat on 2–3 February. There were 193 participants, including 13 from New Zealand. A total of 19 students
obtained Gold certificates, 29 students obtained Silver certificates and 50 students obtained Bronze
certificates. Of the remaining students 52 received an Honourable Mention.

XXXIV Asian Pacific Mathematics Olympiad


In March students from nations around the Asia-Pacific region were invited to sit the Asian Pacific
Mathematics Olympiad (APMO). Of the top ten Australian students who participated, there were three
Silver, four Bronze and three Honourable Mention certificates awarded.

Mathematics Contests The Australian Scene 2022 | About the Program | 69


11th European Girls' Mathematical Olympiad, Eger, Hungary
The 11th EGMO was hosted by Eger, Hungary. The EGMO consists of two papers of three questions
worth seven points each. They were attempted by teams of four students from 56 countries on 8 and 9
April, with Australia placing 3rd. The results for Australia were four Gold medals.

63rd International Mathematical Olympiad, Oslo, Norway


The 63rd IMO was hosted by Oslo, Norway. The IMO consists of two papers of three questions worth
seven points each. They were attempted by teams of six students from 104 countries on 11 and 12 July,
with Australia placing 29th. The results for Australia were one Gold and four Silver medals, and one
Honourable Mention.

2022 AMOC Senior Contest


Held on 16 August, the Senior Contest was sat by 130 students. A total of 15 students obtained Gold
certificates including one perfect score by William Cheah, 18 students obtained Silver certificates
and 33 students obtained Bronze certificates. Of the remaining students 33 received an Honourable
Mention.

Mathematics Contests The Australian Scene 2022 | About the Program | 70


Timetable for Selection of the Teams to the
2022 EGMO and IMO

Each year many hundreds of gifted young Australian school students are identified using the results
from the Australian Mathematics Competition, the Maths for Young Australians program and other
smaller mathematics competitions. A network of 30 or so dedicated mathematicians and teachers has
been organised to give these students support during the year, either by correspondence programs or
by special teaching sessions run in each state. These programs are known to the respective AMOC State
Directors.

These students are among others who sit the AIMO paper in September, or who are invited to sit the
AMOC Senior Contest each August. The outstanding students in these contests, programs and other
mathematical competitions are then identified. About 90 of these are then invited to attend the AMOC
School of Excellence (operating virtually during COVID-19), which is held in November/December each
year. At this School, four students plus one reserve are selected for the European Girls’ Mathematical
Olympiad, held in April annually.

In February, approximately 140 students are invited to attempt the Australian Mathematical Olympiad.
The best 20 or so of these students are then invited to represent Australia in the correspondence Asian
Pacific Mathematics Olympiad in March. About 80 students are chosen for the AMOC Selection School
(operating virtually during COVID-19) which is held in March/April each year just prior to EGMO. At this
school six students plus one reserve are selected for the International Mathematical Olympiad, held in
July annually. A mentor program for the Australian teams operates prior to EGMO and the IMO.

The AMOC program is not meant to develop only future mathematicians. Overseas experience has
shown that many choose to work in the fields of engineering, computing, the physical and life sciences
while others will study law or go into the business world. We hope that the AMOC Mathematics Problem-
Solving Program will help the students to think logically, creatively, deeply and with dedication and
perseverance; that is, it will prepare these talented students to be future leaders of Australia.

Month Activity
August • Outstanding students are identified from AMC results, MYA, other
competitions and recommendations; and eligible students from previous
training programs
• AMOC state organisers invite students to participate in AMOC programs
• Various state-based programs
• AMOC Senior Contest
September • Australian Intermediate Mathematics Olympiad
November/December • AMOC School of Excellence
January • Summer Correspondence Program for those who attended the School
of Excellence
February • Australian Mathematical Olympiad
March • Asian Pacific Mathematics Olympiad
• AMOC Selection School
March/April
• Team preparation for 2022 EGMO in Eger, Hungary
May–June • Mentor program for those who attended the 2022 Selection School
July • IMO team preparation school for 2022 IMO in St Petersberg, Russia

Mathematics Contests The Australian Scene 2022 | Timetable for Selection of the Teams to the 2022 EGMO and IMO | 71
Australia's Achievements at EGMO
Year City Country Gold Silver Bronze HM Rank

2018 Florence Italy 1 2 1 20 out of 52 teams

2019 Kyiv Ukraine 2 2 14 out of 49 teams

2020 Egmond aan Zee Netherlands 1 1 1 12 out of 53 teams

2021 Kutaisi Georgia 1 2 12 out of 54 teams

2022 Edger (hyprid) Hungary 4 3 out of 57 teams

Australia's Achievements at IMO


Year City Country Gold Silver Bronze HM Rank

1981 Washington DC USA 1 23 out of 27 teams

1982 Budapest Hungary 1 21 out of 30 teams

1983 Paris France 1 2 19 out of 32 teams

1984 Prague Czechoslovakia 1 2 15 out of 34 teams

1985 Joutsa Finland 1 1 2 11 out of 38 teams

1986 Warsaw Poland 5 15 out of 37 teams

1987 Havana Cuba 3 15 out of 42 teams

1988 Canberra Australia 1 1 1 17 out of 49 teams

1989 Braunschweig Germany 2 2 22 out of 50 teams


People's Republic
1990 Beijing 2 4 15 out of 54 teams
of China
1991 Sigtuna Sweden 3 2 20 out of 56 teams

1992 Moscow Russia 1 1 2 1 19 out of 56 teams

1993 Istanbul Turkey 1 2 3 13 out of 73 teams

1994 Hong Kong Hong Kong 2 3 3 12 out of 69 teams

1995 Toronto Canada 1 4 1 21 out of 73 teams

1996 Mumbai India 2 3 23 out of 75 teams

1997 Mar del Plata Argentina 2 3 1 9 out of 82 teams

1998 Taipei Taiwan 4 2 13 out of 76 teams

1999 Bucharest Romania 1 1 3 1 15 out of 81 teams

2000 Taejon Republic of Korea 1 3 1 16 out of 82 teams

2001 Washington DC USA 1 4 25 out of 83 teams

2002 Glasgow Scotland 1 2 1 1 26 out of 84 teams

2003 Tokyo Japan 2 2 2 26 out of 82 teams

2004 Athens Greece 1 1 2 1 27 out of 85 teams

2005 Merida Mexico 6 25 out of 91 teams

2006 Ljubljana Slovenia 3 2 1 26 out of 90 teams

Mathematics Contests The Australian Scene 2022 | Timetable for Selection of the Teams to the 2022 EGMO and IMO | 72
Year City Country Gold Silver Bronze HM Rank

2007 Hanoi Vietnam 1 4 1 22 out of 93 teams

2008 Madrid Spain 5 1 19 out of 97 teams

2009 Bremen Germany 2 1 2 1 23 out of 104 teams

2010 Astana Kazakhstan 1 3 1 1 15 out of 96 teams

2011 Amsterdam Netherlands 3 3 25 out of 101 teams

2012 Mar del Plata Argentina 2 4 27 out of 100 teams

2013 Santa Marta Colombia 1 2 3 15 out of 97 teams

2014 Cape Town South Africa 1* 3 2 11 out of 101 teams

2015 Chiang Mai Thailand 2 4 6 out of 104 teams

2016 Hong Kong Hong Kong 2 4 25 out of 109 teams

2017 Rio de Janeiro Brazil 3 2 1 34 out of 111 teams

2018 Cluj–Napoca Romania 2 3 1 11 out of 109 teams

2019 Bath United Kingdom 2 1 3 18 out of 112 teams

2020 St Petersburg Russia 2 3 1 8 out of 105 teams

2021 St Petersburg Russia 2 2 1 1 18 out of 107 teams

2022 Olslo Norway 1 4 1 19 out of 107 teams

* Perfect Score by Alexander Gunning

Mathematics Contests The Australian Scene 2022 | Timetable for Selection of the Teams to the 2022 EGMO and IMO | 73
Support for the Australian Mathematical Olympiad
Training Program

The Australian Mathematical Olympiad Training Program is an activity of the Australian Maths Trust
supported by the Australian Mathematical Olympiad Committee. It aims to identify, train and support
young mathematicians and to support the Australian International Mathematical Olympiad and
European Girls' Mathematical Olympiad teams.

Sponsors

AMT’s Olympiad programs are supported by the Australian Government Department of Industry,
Science, Energy and Resources through the Science Competitions: Mathematics and Informatics
Olympiads grant opportunity. They are also supported by Optiver, the Trust's National Sponsor of the

Australian Informatics and Mathematical Olympiads.

The AMT's EGMO initiative also receives grant funding from the Australian Government through
the Department of Industry, Science, Energy and Resources under the Inspiring Australia – Science
Engagement Programme.

The views expressed here are those of the authors and do not necessarily represent the views of the
government.

Special thanks

With special thanks to the Australian Mathematical Society, the Australian Association of Mathematics
Teachers and all those schools, societies, families and friends who have supported the 2021 EGMO and
IMO teams.

Mathematics Contests The Australian Scene 2022 | Support for the Australian Mathematical Olympiad Training Program | 74
2021 AMOC School of Excellence

The 2021 AMOC School of Excellence was held online 5–10 December. The main qualifying exams for
this School are the AIMO and the AMOC Senior Contest.

A total of 90 students from around Australia plus one student from New Zealand attended the School.
The breakdown of the Australian students into the three streams were as follows.

Stream Female Male Total


Senior 7 13 20
Intermediate 7 13 20
Junior 16 34 50
Total 30 60 90

The main purposes of the School are to:

• select the team of four for the following year’s EGMO from among the Senior group
• train students in all groups for possible future selection in an EGMO or IMO team.

In addition to preparing students for possible selection in an Australian mathematical Olympiad team,
the program helps all to develop their mathematical abilities, make new friends and enjoy the company
of other like-minded people.

The program covered the four major areas of number theory, geometry, combinatorics and algebra.
The academic program on non-exam days comprised of three 1.5 hour interactive lectures starting at
11am, 1:15pm and 3:45pm (AEDT), followed by a 1.5 hour problem session starting at 6:30pm. The two
exam days were a bit freer and included teams-based mathematical games in the evening.

Three highly experienced Senior students were invited to give one Senior lecture each. Not only does
the entire class benefit, but the process of preparing to teach also benefits the teacher. The student
lecture assignments were as follows.

William Cheah Constructions: Reverse and Inspired

Zian Shang Functional Equations

James Xu (from New Zealand) Combinatorial Number Theory

They all did an excellent job! I note that William and Zian went on to win Bronze medals and James
won a Silver medal at IMO 2022.

Many thanks to Michelle Chen, Yifan Guo, Grace He, Preet Patel, Thanom Shaw, Hadyn Tang, Andy Tran
and Sampson Wong, who served as virtual live-in staff and Zoom hosts.

Thanks also to Adrian Agisilaou, Gareth Ainsworth, Ross Atkins, James Bang, Andres Buritica, Matthew
Cheah, Jason Chi, Yong See Foo, Ivan Guo, Claire Huang, Yasiru Jayasooriya, Victor Khou, Charles Li,
Dana Ma, Kim Ramchen, Chaitanya Rao, Micah Sinclair, Ethan Tan, Ruiqian Tong, Haibing Wang, Rachel
Wong, Ruiying Wu, Elizabeth Yevdokimov, Fredy Yip and Ellen Zheng, who assisted in lecturing and
marking.

Angelo Di Pasquale
Director of Training, AMOC

Mathematics Contests The Australian Scene 2022 | 2021 AMOC School of Excellence | 75
Participants at the 2021 AMOC School of Excellence
Name m/f Year School State
Senior
William Cheah m 7 Scotch College VIC
Xiaoyu Chen m 11 All Saints’ College WA
Jordan Chi m 10 Scotch College VIC
Eunsu Choi f 10 Brisbane State High School QLD
Michael (Zixiang) Jiang m 10 Melbourne Grammar School VIC
Sherry (Yaohong) Jiang f 10 North Sydney Girls High School NSW
Crystal (Yunxin) Jin f 10 James Ruse Agricultural High School NSW
David Lumsden m 11 Scotch College VIC
Michael (Quang Minh) Nguyen m 11 Melbourne High School VIC
Sizhe Pan m 10 James Ruse Agricultural High School NSW
Daniel Seo m 10 James Ruse Agricultural High School NSW
Zian Shang m 10 Scotch College VIC
Catherine (Jiaqi) Song f 11 North Sydney Girls High School NSW
Christopher Tran m 10 The University High School VIC
Leonardo Wang m 10 Scotch College VIC
Tony (Yuzheng) Wu m 11 Killara High School NSW
Cloris Xu f 9 Baulkham Hills High School NSW
Iris Xu f 9 Baulkham Hills High School NSW
James Xu m 12 Kristin School NZ
Eason Yang m 8 Trinity Grammar VIC
Grace Chang Yuan f 11 Methodist Ladies’ College VIC
Intermediate
Joel Bariss m 9 Perth Modern School WA
Roger Cao m 10 Camberwell Grammar School VIC
Inez Crosby f 9 Santa Sabina College NSW
Joshua (Kezhuo) Deng m 7 Scotch College VIC
Evan Hee m 10 Rossmoyne Senior High School WA
Max Judd m 10 Perth Modern School WA
Elizabeth (Xuan Xian) Lee f 9 Queensland Academy for Science, QLD
Mathematics and Technology
William Liu m 10 The Scots College NSW
Laura (Xiangyue) Nan f 8 Fintona Girls’ School VIC
Jayden Pan m 7 Shore School NSW
Kathlin (Si Lin) Qiu f 9 Lauriston Girls’ School VIC
Miranda Sui f 9 Prebyterian Ladies’ College VIC
Ben Szabo-Virag m 9 Matthew Flinders Anglican College QLD
Justin Tran m 8 Sydney Grammar School NSW
Andrew (Yang) Wang m 9 Melbourne High School VIC
Liah (Zizhen) Wu f 9 Prebyterian Ladies’ College VIC
Ryan (Dongqiang) Yang m 10 John Monash Science School VIC
Ethan Yap m 9 Perth Modern School WA

Mathematics Contests The Australian Scene 2022 | 2021 AMOC School of Excellence | 76
Name m/f Year School State
Elaine (Jing Ru) Zhang f 9 Haileybury VIC
Yitong Zheng m 9 Indooroopilly State High School QLD
Junior
Alexander Artemov m 8 Scotch College VIC
Jeremy Arulampalam m 7 Walkerville Primary School SA
Sophia Boyce f 10 Canberra Girls Grammar School ACT
Albert Chen m 9 Melbourne Grammar School VIC
Sunguo (Jerry) Chen m 9 Kelvin Grove State College QLD
Yvonne Cui f 10 Presbyterian Ladies’ College WA
Matilda Grant f 8 A. B. Paterson College QLD
Anoushka Gupta f 8 Perth Modern School WA
Michelle (Xueyao) Han f 8 Caulfield Grammar School VIC
Ian Lai m 8 Queensland Academy for Science, Mathe- QLD
matics and Technology
Amber (Yanfei) Li f 7 Pymble Ladies’ College NSW
Annabel Li f 6 Artamon Public School NSW
Dora (Jiayi) Li f 10 Fintona Girls’ School VIC
Jiamu Li m 9 Scotch College VIC
Rhianna Li f 7 James Ruse Agricultural High School NSW
Jingni Liao m 9 Knox Grammar School NSW
Tyson Lieu m 8 Sydney Grammar School NSW
Aidan Lim m 8 Christ Church Grammar School WA
Kevin (Hangzhi) Liu m 5 Haileybury VIC
Matthew Lo m 8 Trinity Grammar School VIC
Jason Miao m 9 Knox Grammar School NSW
Yangyang (Qiyang) Ning m 8 James Ruse Agricultural High School NSW
Witter Onggara m 8 Scotch College VIC
Dylan Phung m 9 Yarra Valley Grammar School VIC
Willa Ran f 7 Melbourne Girls Grammar School VIC
Alaina Ray f 7 Presbyterian Ladies’ College NSW
Chrysolite Salla f 7 Presbyterian Ladies’ College NSW
Atharva Sathe m 9 Perth Modern School WA
Ee Lin Saw f 8 Somerville House QLD
Mohijit Singh m 8 Baulkham Hills High School NSW
Harrison Su m 9 James Ruse Agricultural High School NSW
Candy Tang f 7 Baulkham Hills High School NSW
Frank Tang m 9 Scotch College VIC
Johnson Tang m 9 The King’s School NSW
Oliver Taylor m 9 Caulfield Grammar School VIC
Luke Wang m 8 James Ruse Agricultural High School NSW
Matthew (Cunmeng) Wang m 6 Integrated Education Centre of Excellence VIC
Stanley Wu m 8 James Ruse Agricultural High School NSW
Jerry Qihao Xia m 7 Scotch College VIC
Tina Xiao f 8 Presbyterian Ladies’ College VIC

Mathematics Contests The Australian Scene 2022 | 2021 AMOC School of Excellence | 77
Name m/f Year School State
Echo Xu f 7 James Ruse Agricultural High School NSW
Andrew Yan m 10 Smith’s Hill High School NSW
Max (Beikun) Yan m 9 Anglican Church Grammar School QLD
Ryan Yang m 6 Qs School VIC
Victor Yuan m 8 James Ruse Agricultural High School NSW
Ben Zhang m 9 St Joseph’s College Gregory Terrace QLD
Jayee Zhang m 6 Qs School VIC
Daniel Zhou m 6 Qs School VIC
Dennis Zhou m 8 Brisbane Grammar School QLD
Leon Zhou m 7 James Ruse Agricultural High School NSW

Mathematics Contests The Australian Scene 2022 | 2021 AMOC School of Excellence | 78
AMO.
022 Australian Mathematical Olympiad
2022

AMO.
AUSTRALIAN
2022
MATHEMATICAL OLYMPIAD OLYMPIAD
AUSTRALIAN MATHEMATICAL
AUSTRALIAN
AUSTRALIAN 2022OLYMPIAD OLYMPIAD
MATHEMATICAL
MATHEMATICAL
2022
DAY 1
Wednesday, 2 February 2022
DAY allowed:
Time 1 4 hours
Wednesday,
No calculators are to be2022
2 February used.
Time allowed: is
Each question 4 hours
worth seven points.
No calculators are to be used.
Each question is worth seven points.
1. Prove that a convex pentagon with integer side lengths and an odd perimeter can
have two right angles, but cannot have more than two right angles.
1. (A
Prove that ais convex
polygon convex pentagon
if all of itswith integer
interior sideare
angles lengths and180
less than an ◦odd
.) perimeter can
have two right angles, but cannot have more than two right angles.
Suppose
2. (A that
polygon P is a ifpoint
is convex all ofinside a convex
its interior hexagon
angles than 180◦ .)
are lessABCDEF such that P ABC,
P CDE and P EF A are parallelograms of equal area.
Suppose
2. Prove that
that P is aPpoint
P BCD, DEF inside
and P a convex
F AB are hexagon ABCDEF ofsuch
also parallelograms that
equal P ABC,
area.
P CDE and P EF A are parallelograms of equal area.
3. Prove
Determine
that all polynomials
P BCD, P DEFp(x)
andwith realare
P F AB coefficients such that: of equal area.
also parallelograms

ˆ p(x) >all
3. Determine 0 for all positive
polynomials realwith
p(x) numbers x
real coefficients such that:
1 1 1
ˆ + + = 1 for all positive real numbers x, y, z satisfying xyz = 1.
ˆ p(x)
p(x) > 0p(y)
for all p(z)
positive real numbers x
1 1 1
4. Letˆ Sp(x) + set of
be the + points = 1 for all positive real numbers x, y, z satisfying xyz = 1.
p(y) p(z) (i, j) in the plane with i, j ∈ {1, 2, . . . , 2022} and i < j.
We would like to colour each point in S either red or blue, so that whenever points
Letj)Sand
4. (i, be (j,
thek)set of points
have the same(i, j) in thethe
colour, plane
pointwith ∈ {1,
i, jalso
(i, k) has2,that
. . . , 2022}
colour.and i < j.
We would like to colour each point
How many such colourings of S are there? in S either red or blue, so that whenever points
(i, j) and (j, k) have the same colour, the point (i, k) also has that colour.
How many such colourings of S are there?

The Olympiad program is supported by the Australian Government


Department of Industry, Science, Energy and Resources through the Science
Competitions: Mathematics and Informatics Olympiads grant opportunity.
The Olympiad
© 2022 program
Australian is supported
Mathematics Trust by the Australian Government
Department of Industry, Science, Energy and Resources through the Science
Competitions: Mathematics and Informatics Olympiads grant opportunity.

© 2022 Australian Mathematics Trust


Mathematics Contests The Australian Scene 2022 | Australian Mathematical Olympiad Paper | 79
AMO.
2022

AMO.
AUSTRALIAN
2022
MATHEMATICAL OLYMPIAD OLYMPIAD
AUSTRALIAN MATHEMATICAL
AUSTRALIAN
AUSTRALIAN 2022OLYMPIAD OLYMPIAD
MATHEMATICAL
MATHEMATICAL
2022
DAY 1
Wednesday, 2 February 2022
DAY allowed:
Time 2 4 hours
Thursday, 3 February 2022
No calculators are to be used.
Time allowed: is
Each question 4 hours
worth seven points.
No calculators are to be used.
Each question is worth seven points.
1. Prove that a convex pentagon with integer side lengths and an odd perimeter can
have two right angles, but cannot have more than two right angles.
5. (A
Letpolygon
x1 , x2 , . .is. ,convex
x2022 be
if aallsequence of real angles
of its interior numbersaresuch
less that
thanx180 2 + · · · + x2022 > 0.
1 + ◦x.)
Suppose that there exists a positive integer k less than 2022 such that:
2. Suppose that P is a point inside a convex hexagon ABCDEF such that P ABC,
ˆ the first k terms of the sequence are less than or equal to zero
P CDE and P EF A are parallelograms of equal area.
ˆ the remaining terms of the sequence are greater than or equal to zero.
Prove that P BCD, P DEF and P F AB are also parallelograms of equal area.
Prove that x1 + 2x2 + 3x3 + · · · + 2022x2022 > 0.
3. Determine all polynomials p(x) with real coefficients such that:
6. In the country of Biplania there are two airline companies: Redwing and Blueways.
ˆ p(x) > 0 for all positive real numbers x
There are 20 cities in Biplania, including Abadu and Ethora. Between any pair of
1 1 1
ˆ exactly
cities, + one+of the = two airlines
1 for has direct
all positive realflights andx,that
numbers y, z airline fliesxyz
satisfying in both
= 1.
p(x) p(y) p(z)
directions between the two cities.
Let S be
4. Arthur theflies
only set on
of points (i, j)Heincan
Redwing. thetravel
planefrom i, j ∈ {1,
with Abadu to 2, . . . , 2022}
Ethora and i four
in exactly < j.
We would
flights but like
not to colour each point in S either red or blue, so that whenever points
fewer.
(i, j) and (j, k) have the same colour, the point (i, k) also has that colour.
Martha only flies on Blueways.
How many such colourings of S are there?
Show that Martha can travel between any two cities in Biplania in at most two
flights.

7. For any positive integer n, let f (n) denote the absolute value of the difference
between n and its nearest power of 2. We define the stopping time of n to be the
smallest positive integer k such that f (f (· · · f (n) · · · )) = 0, where f is applied k
times. For example, f (54) = 10, f (10) = 2, and f (2) = 0. So f (f (f (54))) = 0 and
the stopping time of 54 is 3.

(a) What is the smallest positive integer whose stopping time is 2022?

(b) What is the maximum number of consecutive positive integers that have the
same stopping time?

8. Let ABC be an acute scalene triangle with orthocentre H. The altitudes from B
and C intersect the angle bisector of ∠BAC at D and E respectively. The tangents
to the circumcircle of triangle DEH at D and E intersect at X.
Prove that BX = CX.

The Olympiad program is supported by the Australian Government


Department of Industry, Science, Energy and Resources through the Science
Competitions: Mathematics and Informatics Olympiads grant opportunity.
The Olympiad
© 2022 program
Australian is supported
Mathematics Trust by the Australian Government
Department of Industry, Science, Energy and Resources through the Science
Competitions: Mathematics and Informatics Olympiads grant opportunity.

© 2022 Australian Mathematics Trust


Mathematics Contests The Australian Scene 2022 | Australian Mathematical Olympiad Paper | 80
AUSTRALIAN MATHEMATICAL OLYMPIAD 2022
Australian
SolutionsMathematical Olympiad
Solutions
© 2022 Australian Mathematics Trust

1. Prove that a convex pentagon with integer side lengths and an odd perimeter can have
two right angles, but cannot have more than two right angles.
(A polygon is convex if all of its interior angles are less than 180◦ .)

Solution (Mike Clapper)


Firstly we establish that such a pentagon with two right angles is possible. A unit
equilateral triangle placed on top of a unit square forms a pentagon with perimeter 5.
From the angle sum formula, we see that if a pentagon has four right angles, the fifth
angle would be 180◦ which means that the pentagon actually degenerates into a rectangle
(and would have an even perimeter anyway).
So it remains to show that a pentagon with integer side lengths and three right angles
must have an even perimeter. If three out of five angles are right angles, then at least
two of them must be adjacent. This gives two cases, all three right angles are adjacent or
only two are adjacent, as shown below:

Case 1 Case 2

Firstly consider Case 1:

a c

The bounding rectangle has integer sides and hence its perimeter has even parity. Further-
more, the dotted section forms a right-angled triangle with integer sides. The perimeter
of the pentagon is found by replacing the two dotted sides with the hypotenuse. But since

Mathematics Contests The Australian Scene 2022 | Australian Mathematical Olympiad Solutions | 81
a2 + b2 = c2 implies that a + b has the same parity as c, the perimeter of the pentagon is
also even.
Algebraically, using the labelling in the diagram, the perimeter of the bounding rectangle
is 2(x + y) and the perimeter of the pentagon is 2(x + y) + (c − a − b).
Now consider Case 2:

c
e
a

b
y

Draw a line parallel to the base as in the diagram to form two right-angled triangles, both
with hypotenuse e. We see that a2 + b2 = c2 + d2 . This means that a + b has the same
parity as c+d and hence a+b+c+d is even. As the perimeter is given by a+b+c+d+2y
it follows that the pentagon has even perimeter.
Hence a pentagon with integer sides and an odd perimeter can have at most two right
angles.

2
Mathematics Contests The Australian Scene 2022 | Australian Mathematical Olympiad Solutions | 82
2. Suppose that P is a point inside a convex hexagon ABCDEF such that P ABC, P CDE
and P EF A are parallelograms of equal area.
Prove that P BCD, P DEF and P F AB are also parallelograms of equal area.

Solution 1 (Angelo Di Pasquale)


A diagonal of a parallelogram divides it into two triangles of equal area. Hence the lines
connecting P to the vertices of the hexagon partition into six triangles of equal area.
From this it follows that P BCD, P DEF and P F AB are quadrilaterals of equal area.
We also have
1 1
|AP C| = |P ABC| = |P EF A| = |AP E|.
2 2
Let line Q be the intersection of lines AP and CE.

F E

A P Q D

B C

We have
|CP Q| CQ |CAQ|
= = .
|EP Q| QE |EAQ|
Using addendo this implies

CQ |CAQ| − |CP Q| |AP C|


= = = 1.
QE |EAQ| − |EP Q| |AP E|

Thus Q is the midpoint of CE.


Since P CDE is a parallelogram, its diagonals bisect each other. In particular P QD is
a straight line. Hence points A, P , Q, and D are collinear. Thus P D ∥ F E. A similar
argument shows that P F ∥ DE. Hence P DEF is a parallelogram. Similarly P F AB and
P BCD are also parallelograms, as required.

Solution 2 (Thanom Shaw)


Consider the dilation by a factor of 2 with centre B and let BA′ P ′ C ′ be the image of
parallelogram BAP C.

3
Mathematics Contests The Australian Scene 2022 | Australian Mathematical Olympiad Solutions | 83
B

A
C′

P
A′ D

P′ = E

Since P ABC and P EF A are parallelograms of equal area, side EF lies on the line P ′ A′ .
Since P ABC and P CDE are parallelograms of equal area, side ED lies on the line P ′ C ′ .
Since E lies on both P ′ A′ and P ′ C ′ , E is the point of intersection of these lines. That is,
E = P ′ and P is the midpoint of BE. Similarly, P is also the midpoint of AD and CF .
Now P B, P D, and P F divide parallelograms P ABC, P CDE, and P EF A into congruent
triangles P AB, BCP , DP C, P DE, EF P , AP F , of equal area. Note that corresponding
sides of these triangles are equal given opposite sides of parallelograms are equal and P
is the midpoint of BE, AD and CF .
Finally, we have that each of P BCD, P DEF and P F AB are parallelograms as their op-
posite sides are equal. Furthermore, each parallelogram consists of the pairs of congruent
triangles of equal area, BCP & DP C, P DE & EF P , AP F & P AB. Hence P BCD,
P DEF and P F AB are parallelograms of equal area.

Solution 3 (Angelo Di Pasquale)


The problem is invariant under an affine transformation. Hence we may assume without
loss of generality that P ABC is a unit square, with B = (0, 0), C = (1, 0), P = (1, 1),
and A = (0, 1). From |P ABC| = |P EF A|, point E lies on the line y = 2. From
−→ −→
|P ABC| = |P CDE|, point E lies on the line x = 2. Hence E = (2, 2). Since AP = F E,
we have F = (1, 2), and similarly D = (2, 1). From these, the conclusion easily follows.

Solution 4 (Chaitanya Rao)


Establish that the quadrilaterals have equal area as in Solution 1. To show they are
parallelograms, we start by writing the given equal area condition as equal vector cross
−→ −→ −→ −→ −→ −→
products: P A × P C = P C × P E = P E × P A. From the first equality,
−→ −→ −→ −→ −→ −→ −→ − →
P A × P C = −P E × P C =⇒ (P A + P E) × P C = 0 .
−→ −→ −→ −→ −→ −→ −→ − →
Since P EF A is a parallelogram, P A + P E = P A + AF = P F so P F × P C = 0 . As the
two vectors are each non-zero, we conclude that F P ∥ P C ∥ AB. By a similar argument
we can show that BP ∥ P E ∥ AF . Hence P F AB is a parallelogram. Similarly P BCD
and P DEF are parallelograms.

4
Mathematics Contests The Australian Scene 2022 | Australian Mathematical Olympiad Solutions | 84
3. Determine all polynomials p(x) with real coefficients such that:

ˆ p(x) > 0 for all positive real numbers x


1 1 1
ˆ + + = 1 for all positive real numbers x, y, z satisfying xyz = 1.
p(x) p(y) p(z)

Solution 1 (Norman Do)


Substituting x = y = z = 1 yields P (1) = 3. Now using the given relation, we obtain
1 1 1 1 1 1
+ + −2
= + + .
P (x) P (x) P (x ) P (x ) P (1) P (x−2 )
2

This implies that


2 1 1
= 2
+ .
P (x) P (x ) 3
Now multiply out to obtain the equation

6P (x2 ) = 3P (x) + P (x2 )P (x).

This must be true for all x > 0, which then implies that the equation is true at the level
of polynomials. If the degree of P (x) is d > 0, then the left side of this equation has
degree 2d, while the right side has degree 3d. This yields a contradiction, so it must be
the case that P (x) is a constant polynomial. Since P (1) = 3, the only possible solution
is P (x) = 3, which does indeed satisfy the given conditions.

Solution 2 (William Steinberg)


We first show that if P is a polynomial satisfying similar conditions in two variables for
xy = 1 then P (x) = xn + 1 for some n. This means that for xy = 1 we have
1 1
+ =1
P (x) P (y)

which we can rewrite as (P (x) − 1)(P (x−1 ) − 1) = 1. Define Q(x) = P (x) − 1 and R(x) =
xd Q(x−1 ) where d is the degree of Q. In other words, R is the same polynomial as Q but
with coefficients reversed. Thus for each positive real x we have that Q(x)R(x) = xd ,
so both Q and R must be monomials and hence P (x) = Q(x) + 1 = cxn + 1 for some c
and n. Substituting this back into the original condition and expanding we find that this
works when c = ±1, but c = −1 violates the condition that P (x) > 0 for x > 0.
Now we go back to the original problem. Substitute x = y = z = 1 to obtain P (1) = 3.
Substituting z = 1 gives us, for xy = 1,
1 1 1 2
+ =1− = .
P (x) P (y) P (1) 3

Observe that P ′ (x) = 23 P (x) satisfies the same conditions as in the two-variable version
described so P ′ (x) = xn + 1 for some n and hence P (x) = 32 xn + 32 . If n > 0 then taking
1 1
x = y = 3− n and z = 9 n yields a contradiction. Thus P (x) = 3 and this clearly works.

5
Mathematics Contests The Australian Scene 2022 | Australian Mathematical Olympiad Solutions | 85
Solution 3 (Ian Wanless)
Suppose first that P (x) is not constant. Rewriting the equation we have
1 1 1
1− = +
P (x) P (y) P (1/(xy))

whenever x, y > 0. Fixing x but taking the limit as y → ∞ we see the right-hand side
approaches 1/P (0). But this is independent of x, so the left-hand side must be 1/P (0)
too. Since P (x) is constant on x > 0 it must be constant. It’s clear that if P (x) is
constant then it must be that P (x) = 3, and that this is a solution.

6
Mathematics Contests The Australian Scene 2022 | Australian Mathematical Olympiad Solutions | 86
4. Let S be the set of points (i, j) in the plane with i, j ∈ {1, 2, . . . , 2022} and i < j. We
would like to colour each point in S either red or blue, so that whenever points (i, j) and
(j, k) have the same colour, the point (i, k) also has that colour.
How many such colourings of S are there?

Solution 1 (Ivan Guo)


The answer is 2022!. We will work with the general case of S = {(i, j) : 1 ≤ i < j ≤ N }
and present a bijection between valid colourings and permutations on 1, . . . , N .
First, construct a map from permutations to colourings as follows. Given a permutation
(a1 , a2 , . . . , aN ), colour (i, j) red if ai < aj and blue otherwise. The colouring satisfies the
required conditions since:

ˆ If (i, j) and (j, k) are both red, then ai < aj < ak . Hence (i, k) is also red.
ˆ If (i, j) and (j, k) are both blue, then ai > aj > ak . Hence (i, k) is also blue.

Next, we show that this map is injective. If two permutations (ai )1≤i≤N , (bi )1≤i≤N map to
the same colouring, then for every i < j, we have ai < aj if and only if bi < bj . Hence, for
every i, the sets {j : aj > ai } and {j : bj > bi } are identical. Thus the two permutations
must be the same.
It remains to check that the map is surjective. We will prove that every colouring is
obtainable from this mapping via an induction on N . The idea is to identifying a number
p such that (k, p) is red for every k < p, and (p, k) is blue for every k > p. If this can be
done then ap must be N in the permutation, while the other elements can be reduced to
the N − 1 case. The base case of N = 1 is trivially true.
Define the sequence c1 , c2 , . . . , cm as follows. Let c1 = 1. For each i > 1, let ci be the
smallest integer such that ci > ci−1 and (ci−1 , ci ) is red. The construction continues for
as long as possible, and terminates at cm = p. Consequently, (p, k) must be blue for all
k > p.
We claim that for all k < p, (k, p) is red. Since the colouring is valid, (ci , cj ) must be
red for all i < j. Hence (ci , p) is red for all ci < p. Suppose there is some q such that
ci < q < ci+1 and (q, p) is blue. By the minimality of ci+1 , (ci , q) must be blue. This
implies that (ci , p) is blue, which is a contradiction. So the required property on p is
proven.
To complete the induction, we remove all points in S which have coordinates involving p,
and consider the colouring of the remaining points {(i, j) : i ̸= p, j ̸= p, 1 ≤ i < j ≤ N }.
This is equivalent to a colouring of the N − 1 case. By the inductive hypothesis, there
is permutation (b1 , . . . , bN −1 ) of 1, . . . , N − 1 which maps to this colouring. It is then
straightforward to check that the permutation (b1 , . . . , bp−1 , N, bp , . . . , bN −1 ) maps to our
original colouring. This establishes the bijection and completes the proof.

Solution 2 (Angelo Di Pasquale)


Let Gn be the directed graph on n vertices V1 , . . . Vn such that Vi → Vj if (i, j) is red and
Vj → Vi if (i, j) is blue. Note:

(i) For each P, Q ∈ Gn with P ̸= Q, we have exactly one of P → Q or Q → P .


(ii) If A → B and B → C, then A → C.

7
Mathematics Contests The Australian Scene 2022 | Australian Mathematical Olympiad Solutions | 87
A complete directed graph satisfying (i) and (ii) shall be called nice. There’s clearly
a bijection between valid colourings and nice complete directed graphs with labelled
vertices.
We wish to count the number of nice graphs on n labelled vertices. We prove by induction
that the answer is n! for each n ≥ 2.
The base case is immediate.
Assume the answer is n! for all nice graphs on n labelled vertices. Let Gn+1 be a nice
graph on n + 1 labelled vertices.
Pick any point P in Gn+1 , and starting from this point walk around following the arrows.
If we get back to P , then let Q be the last point on the walk just before P so that Q → P .
But P → Q due to multiple applications of (ii). This contradicts (i). Hence we never get
back to P . Since the graph is finite, the walk must terminate at some vertex, Z say. Note
that Z has only in-edges. This implies that no other vertex Z ′ has only in-edges because
otherwise both Z → Z ′ and Z ′ → Z would both occur, which contradicts (i).
Hence we have shown that there is exactly one vertex, Z say, in Gn+1 having only in-edges.
If we remove Z from Gn+1 , we get a nice graph on n labelled vertices. Thus every nice
graph on n + 1 labelled vertices is obtainable from a nice graph on n vertices by adding a
single vertex Z which has only in-edges. Conversely, every such extension of a nice graph
on n vertices results in a nice graph on n + 1 vertices.
Since there are n + 1 ways to choose Z in Gn+1 and n! ways of completing a nice graph
on the remaining n labelled vertices, it follows that there are (n + 1)n! = (n + 1)! nice
graphs on n + 1 labelled vertices.

Solution 3 (Alan Offer)


For a colouring of S, define the relation ‘≺’ on the set N = {1, 2, . . . , 2022} by:

ˆ i ≺ i (so ≺ is reflexive)
ˆ for i < j, i ≺ j if (i, j) is blue, while j ≺ i if (i, j) is red (so ≺ is antisymmetric and,
further, every pair of points is comparable).

We will show that the given condition on the colouring of S is then equivalent to the
condition that ≺ is transitive. Suppose that i ≺ j and j ≺ k. There are six cases:

(a) If i < j < k, (i, j) and (j, k) are both blue. Then (i, k) is coloured validly (blue) if
and only if i ≺ k.
(b) If i < k < j, (i, j) is blue and (j, k) is red. Then (i, k) is coloured validly (blue) if
and only if i ≺ k.
(c) If j < i < k, (i, j) is red and (j, k) is blue. Then (i, k) is coloured validly (blue) if
and only if i ≺ k.
(d) If j < k < i, (i, j) is red and (j, k) is blue. Then (i, k) is coloured validly (red) if
and only if i ≺ k.
(e) If k < i < j, (i, j) is blue and (j, k) is red. Then (i, k) is coloured validly (red) if
and only if i ≺ k.
(f) If k < j < i, (i, j) and (j, k) are both red. Then (i, k) is coloured validly (red) if and
only if i ≺ k.

8
Mathematics Contests The Australian Scene 2022 | Australian Mathematical Olympiad Solutions | 88
Therefore ≺ is transitive if and only if all such triple of points have valid colourings. Thus
≺ is a total order.
Similarly, notice that a total order ≺ in turn determines an allowable colouring of S.
Thus the allowable colourings are in one-to-one correspondence with the total orders of N ,
which in turn correspond to the ways of writing the 2022 elements of N in a row, or in
other words, the permutations of N . Thus the number of colourings is 2022!.

9
Mathematics Contests The Australian Scene 2022 | Australian Mathematical Olympiad Solutions | 89
5. Let x1 , x2 , . . . , x2022 be a sequence of real numbers such that x1 + x2 + · · · + x2022 > 0.
Suppose that there exists a positive integer k less than 2022 such that:

ˆ the first k terms of the sequence are less than or equal to zero
ˆ the remaining terms of the sequence are greater than or equal to zero.

Prove that x1 + 2x2 + 3x3 + · · · + 2022x2022 > 0.

Solution 1 (Kevin McAvaney)


We prove the result for n terms. If all terms are non-negative then the result clearly
holds. If there exists at least one negative term, then we may choose 1 ≤ k < n. Hence

x1 + · · · + (k − 1)xk−1 + kxk + (k + 1)xk+1 + · · · + nxn


> x1 + · · · + (k − 1)xk−1 − k(x1 + · · · + xk−1 + xk+1 + · · · + xn ) + (k + 1)xk+1 + · · · + nxn
= (1 − k)x1 + · · · + (−1)xk−1 + xk+1 + · · · + (n − k)xn
> 0.

Solution 2 (Ivan Guo)


For each i = 1, . . . , n, define the partial sum Si = xi + · · · + xn . Note that the sequence
S1 , . . . , Sn is non-decreasing up to Sk+1 and then non-increasing. Since S1 is positive and
Sn = xn is non-negative, Si must be non-negative for all i. Thus

x1 + 2x2 + · · · + nxn = S1 + · · · + Sn > 0.

Solution 3 (Angelo Di Pasquale)


Let S = x1 + 2x2 + · · · + 2022x2022 .
Suppose that xi < 0 < xj . Then i ≤ k ≤ j. Replace xi and xj by xi + ε and xj − ε where
ε = min{|xi |, |xj |} in S. The change in S (new minus old) is given by

−ixi + i(xi + ε) − jxj + j(xj − ε) = −(j − i)ε < 0.

So the new value of S is less than the old value.


By using a sequence of such replacements we can make 2021 of x1 , . . . , xn equal to 0. At
this point, the final value of S is positive. And since it is less than the initial value of S,
we conclude that the initial value of S was also positive.

10
Mathematics Contests The Australian Scene 2022 | Australian Mathematical Olympiad Solutions | 90
6. In the country of Biplania there are two airline companies: Redwing and Blueways. There
are 20 cities in Biplania, including Abadu and Ethora. Between any pair of cities, exactly
one of the two airlines has direct flights and that airline flies in both directions between
the two cities.
Arthur only flies on Redwing. He can travel from Abadu to Ethora in exactly four flights
but not fewer.
Martha only flies on Blueways.
Show that Martha can travel between any two cities in Biplania in at most two flights.

Solution 1 (Ian Wanless)


We are dealing with a complete graph on 20 vertices (cities) in which every edge is
coloured either red or blue, indicating which airline flies that route. We are told that the
shortest red path from Abadu to Ethora has four edges. Let one such path involve cities
A (Abadu), B, C, D and E (Ethora) in that order. The fact that this is a shortest path
indicates that the edges AC, AD, AE, BD, BE and CE are all blue.
First notice that Martha can fly between any two cities in {A, B, C, D, E} in at most two
steps. If there is not a direct flight on Blueways then she must be trying to fly between
consecutive cities on the path ABCDE. She can do this by flying via the end of the path
furthest from the edge between her two cities.
So we can without loss of generality assume that Martha is trying to fly from city X
which is not one of {A, B, C, D, E}. In order to not provide a shorter path for Arthur,
we know that XA and XE cannot both be red. For the remainder of the proof, suppose
without loss of generality that XA is blue. Hence Martha can reach C, D or E in two
flights by flying via A. Suppose Martha can’t reach B from X in at most two flights.
Then XB must be red and XE must be red (otherwise XE followed by EB works). But
now Arthur can fly AB, BX, XE which is a contradiction. It follows that Martha can
reach any city in {A, B, C, D, E} from X in at most 2 flights.
We may thus assume that Martha’s destination is Y , where Y is not one of {A, B, C, D, E}
and XY is red. If AY is blue then Martha can fly via A, so we may assume that AY is
red. To prevent Arthur from flying AY, Y E we must have that Y E is blue. But now if
XE is blue then Martha can fly via E, so the only problem is if XE is red. But if XE
is red then Arthur can fly AY, Y X, XE. This contradiction finishes the proof.

Solution 2 (Michelle Chen)


Suppose Martha is trying to fly from city X to city Y . First, notice that AE must be
blue since Arthur cannot fly directly from A to E. Now, there are three cases.
If {X, Y } = {A, E}, then Martha can fly directly from X to Y since AE is blue.
Suppose one of X or Y is in {A, E}. Without loss of generality, assume X = A and
Y ̸= E. Since Arthur cannot fly AY , Y E, at least one of AY and Y E is blue. If AY is
blue, then Martha can fly directly from X to Y . Otherwise, Y E is blue and Martha can
fly XE, EY .
Finally, we have the case where neither X nor Y is in {A, E}. If XY is blue, then Martha
can fly directly from X to Y . Otherwise, suppose XY is red. Since Arthur cannot fly
AX, XE, at least one of AX and XE is blue. Without loss of generality, say AX is
blue. Now, if AY is blue, then Martha can fly XA, AY . Otherwise, if AY is red, then
Y E must be blue since Arthur cannot fly AY , Y E. Also, XE must be blue since Arthur
cannot fly AY , Y X, XE. Hence, Martha can fly XE, EY .

11
Mathematics Contests The Australian Scene 2022 | Australian Mathematical Olympiad Solutions | 91
7. For any positive integer n, let f (n) denote the absolute value of the difference between n
and its nearest power of 2. We define the stopping time of n to be the smallest positive
integer k such that f (f (· · · f (n) · · · )) = 0, where f is applied k times. For example,
f (54) = 10, f (10) = 2, and f (2) = 0. So f (f (f (54))) = 0 and the stopping time of 54 is
3.

(a) What is the smallest positive integer whose stopping time is 2022?
(b) What is the maximum number of consecutive positive integers that have the same
stopping time?

Solution 1 (Angelo Di Pasquale)


(a) Answer: 31 (24043 + 1).
For each positive integer i, let ai denote the smallest positive integer with stopping time i.
Observe that for any integer n satisfying 2k−1 ≤ n ≤ 2k we have f (n) ≤ 2k−2 . It follows
that
1 ≤ n ≤ 2k =⇒ f (n) ≤ 2k−2 . (1)
We shall prove ai = 13 (22i−1 + 1) by induction. The base cases i = 1, 2 are easily verified.
Suppose that ai = 13 (22i−1 + 1) for i ≥ 2. It is straightforward to verify that

22i−3 < ai < 22i−2 .

Let b be any positive integer satisfying b < 22i−1 + ai . If b ≤ 22i−1 , then from (1) we have
f (b) ≤ 22i−3 < ai . If 22i−1 < b < 22i−1 + ai , then again f (b) < ai . Either way, f (b) has
stopping time strictly less than i. Thus b has stopping time at most i. It follows that

22i−1 + 1 22i+1 + 1
ai+1 ≥ 22i−1 + ai = 22i−1 + = .
3 3

However, since
22i+1 + 1
22i−1 < ≤ 22i−1 + 22i−2 ,
3
we have  
22i+1 + 1 22i+1 + 1 22i−1 + 1
f = − 22i−1 = = ai .
3 3 3
Hence 13 (22i+1 +1) has stopping time i+1. From this we conclude that ai+1 = 13 (22i+1 +1),
which finishes the induction.
(b) Answer: 3
For each positive integer n, let S(n) denote its stopping time. Note S(5) = S(6) = S(7) =
2. So it is possible that three consecutive positive integers have the same stopping time.
It remains to show that four consecutive positive integers cannot have the same stopping
time.
Suppose, for the sake of contradiction, that some four consecutive positive integers have
the same stopping time. Let n be the smallest integer such that S(n) = S(n + 1) =
S(n + 2) = S(n + 3). Observe that S(n) = 1 if and only if n is a power of 2. Since no
four consecutive positive integers can be powers of 2, there exists an integer k ≥ 3 such
that
2k < n, n + 1, n + 2, n + 3 < 2k+1 .

12
Mathematics Contests The Australian Scene 2022 | Australian Mathematical Olympiad Solutions | 92
Furthermore we calculate that S(2k + 2k−1 ) = 2 and S(2k + 2k−1 ± 1) = 3. Hence none of
the four consecutive integers can be 2k + 2k−1 . Thus it leaves us with two cases.
Case 1. 2k < n, n + 1, n + 2, n + 3 < 2k + 2k−1
Then f (n) = n−2k , f (n+1) = n+1−2k , f (n+2) = n+2−2k , and f (n+3) = n+3−2k are
four smaller consecutive positive integers with the same stopping time. This contradicts
the minimality of n.
Case 2. 2k + 2k−1 < n, n + 1, n + 2, n + 3 < 2k+1
This implies f (n + 3) = 2k+1 − n − 3, f (n + 2) = 2k+1 − n − 2, f (n + 1) = 2k+1 − n − 1,
and f (n) = 2k+1 − n are four smaller consecutive positive integers with the same stopping
time. This contradicts the minimality of n.
Hence no four consecutive positive integers can have the same stopping time.

Solution 2 (Ian Wanless)


(a) Write all numbers in binary. If n starts with 10 then f (n) is obtained by removing
the leading 1 from n. Otherwise n starts with 11, and in that case we complement every
bit from the first 1 up to but not including the last 1 (we leave the last 1 as is). It follows
that f (n) is at least two bits shorter than n unless n is precisely of the form 11 followed
by an arbitrary number of zeroes. It also follows that trailing zeros do not affect the
stopping time.
Let am be the least integer with stopping time equal to m. It is simple to check that
a1 = 1 and a2 = 11. Let bm = 1010 · · · 1011 (where there are m ones) for all m > 1. Since
bm has stopping time m, the definition of am implies am ≤ bm . We will show by induction
that am = bm for all m. The base cases of m = 1, 2 clearly hold.
Suppose not and consider the smallest m > 2 for which am < bm . By definition f (bm ) =
bm−1 = am−1 ≤ f (am ). We will show this is incompatible with the characterisation
above. If am has fewer bits than bm then f (am ) will have fewer bits than f (bm ) = bm−1 ,
contradicting f (bm ) ≤ f (am ). So am and bm must have the same number of bits. But then
am < bm means that am must begin with 10 and so f (am ) would remove only the leading
1. Then am − f (am ) = bm − f (bm ), which contradicts am − bm < 0 ≤ f (am ) − f (bm ).

It follows that a2022 = b2022 = 1 + 2021
i=1 2
2i−1
= (24043 + 1)/3.
(b) Denote the stopping time of n by S(n). Note that three consecutive numbers can
have the same stopping time, S(5) = S(6) = S(7) = 2. To show that four consecutive
numbers cannot have the same stopping time, it suffices to show that S(4k) < S(4k ± 1)
for all k ≥ 1.
We do this by induction. If k is a power of 2 then S(4k) = 1 < 2 = S(4k ± 1) as required.
If k = 3 × 2a for a ≥ 0 then S(4k) = 2 < 3 = S(4k ± 1). For all other values of k, the
numbers f (4k − 1), f (4k), f (4k + 1) are three consecutive integers (in either increasing or
decreasing order) and f (4k) is divisible by 4 and less than 4k. This reduces to an earlier
case, so the claim follows by induction.

13
Mathematics Contests The Australian Scene 2022 | Australian Mathematical Olympiad Solutions | 93
8. Let ABC be an acute scalene triangle with orthocentre H. The altitudes from B and
C intersect the angle bisector of ∠BAC at D and E respectively. The tangents to the
circumcircle of triangle DEH at D and E intersect at X.
Prove that BX = CX.

Solution 1 (Ivan Guo)

D
H X

B E

Let ∠BAC = α. Then

∠HED = ∠HCA + ∠CAE = 90◦ − α/2 = ∠HBA + ∠BAD = ∠HDE.

Hence HE = HD and HEXD is a kite with ∠XDE = ∠XED = ∠DHE = α.


Next, reflect A about CH to obtain the point P . Then ∠BP C = α = 180◦ − ∠BHC,
so BHCP is cyclic. Since E lies on CH, the perpendicular bisector of AP , we have
∠P EH = ∠AEH = ∠EDH. So EP is tangent to the circumcircle of triangle DEH,
and thus X, E and P are collinear.
Since HEXD is a kite, we have

∠HXP = 90◦ − ∠DEX = 90◦ − α = ∠ACH = ∠P CH.

Hence X also lies on the circumcircle of BHCP . Finally, since X lies on the external
angle bisector of ∠BHC, it is the midpoint of the arc BC that contains H. Therefore
XB = XC.

Solution 2 (William Steinberg)

14
Mathematics Contests The Australian Scene 2022 | Australian Mathematical Olympiad Solutions | 94
Let P be the second intersection of circle BHC with AB and M be the midpoint of arc
BHC. Note that,
∠BP C = ∠EHD = ∠BAC.

Since
∠ACH = ∠HBA = ∠HCP,
CE bisects ∠ACP . Also AE bisects ∠P AC. So E is the incentre of triangle AP C and
P E bisects ∠CP A.
Hence P, M, E are collinear. Thus

∠M ED = ∠P AE + ∠EP A = ∠BAC = ∠EHD

and M E is tangent to circle HDE. Similarly M D is tangent to circle HDE. Therefore


X = M and hence BX = BM = CM = CX as desired.

15
Mathematics Contests The Australian Scene 2022 | Australian Mathematical Olympiad Solutions | 95
Australian Mathematical Olympiad
Statistics

Score Distribution/Problem

Mark/Problem Q1 Q2 Q3 Q4 Q5 Q6 Q7 Q8
0 10 35 59 186 26 31 57 120

1 13 31 37 0 14 16 22 46

2 6 7 35 3 2 9 22 3

3 15 34 11 1 0 7 21 4

4 18 29 4 0 1 6 11 0

5 17 4 3 0 8 33 24 0

6 26 5 10 1 20 12 7 1

7 88 48 34 2 122 79 29 19

Average 5.2 3.4 2.4 0.2 5.4 4.5 2.8 1.1

Mathematics Contests The Australian Scene 2022 | Australian Mathematical Olympiad Statistics | 96
XXXIV Asian Pacific Mathematics Olympiad
XXXIV Asian Pacific Mathematics Olympiad

March, 2022

Time allowed: 4 hours Each problem is worth 7 points

The contest problems are to be kept confidential until they are posted on the official
APMO website http://apmo-official.org.
Please do not disclose nor discuss the problems online until that date. The use of
calculators is not allowed.

Problem 1. Find all pairs (a, b) of positive integers such that a3 is a multiple of b2
and b − 1 is a multiple of a − 1.
Note: An integer n is said to be a multiple of an integer m if there is an integer k
such that n = km.

Problem 2. Let ABC be a right triangle with ∠B = 90◦ . Point D lies on the line
CB such that B is between D and C. Let E be the midpoint of AD and let F be
the second intersection point of the circumcircle of ACD and the circumcircle of
BDE. Prove that as D varies, the line EF passes through a fixed point.

Problem 3. Find all positive integers k < 202 for which there exists a positive
integer n such that
 n   2n  
kn

k
+ + ··· + = ,
202 202 202 2

where {x} denote the fractional part of x.


Note: The fractional part of a real number x is defined as the real number k with
0 ≤ k < 1 such that x − k is an integer.

Problem 4. Let n and k be positive integers. Cathy is playing the following game.
There are n marbles and k boxes, with the marbles labelled 1 to n. Initially, all
marbles are placed inside one box. Each turn, Cathy chooses a box and then moves the
marbles with the smallest label, say i, to either any empty box or the box containing
marble i + 1. Cathy wins if at any point there is a box containing only marble n.
Determine all pairs of integers (n, k) such that Cathy can win this game.

Problem 5. Let a, b, c, d be real numbers such that a2 + b2 + c2 + d2 = 1. Determine


the minimum value of (a − b)(b − c)(c − d)(d − a) and determine all values of (a, b, c, d)
such that the minimum value is achieved.

Mathematics Contests The Australian Scene 2022 | XXXIV Asian Pacific Mathematics Olympiad Paper | 97
XXXIV Asian Pacific Mathematics
APMO Olympiad
2022 Solution
Solutions

1. Find all pairs (a, b) of positive integers such that a3 is a multiple of b2 and b − 1 is a multiple of a − 1.
Note: An integer n is said to be a multiple of an integer m if there is an integer k such that n = km.
Solution
Solution 1.1
By inspection, we see that the pairs (a, b) with a = b are solutions, and so too are the pairs (a, 1). We
will see that these are the only solutions.
• Case 1. Consider the case b < a. Since b − 1 is a multiple of a − 1, it follows that b = 1. This
yields the second set of solutions described above.
• Case 2. This leaves the case b ≥ a. Since the positive integer a3 is a multiple of b2 , there is a
positive integer c such that a3 = b2 c.
Note that a ≡ b ≡ 1 modulo a − 1. So we have

1 ≡ a3 = b2 c ≡ c (mod a − 1).

If c < a, then we must have c = 1, hence, a3 = b2 . So there is a positive integer d such that
a = d2 and b = d3 . Now a − 1 | b − 1 yields d2 − 1 | d3 − 1. This implies that d + 1 | d(d + 1) + 1,
which is impossible.
If c ≥ a, then b2 c ≥ b2 a ≥ a3 = b2 c. So there’s equality throughout, implying a = c = b. This
yields the first set of solutions described above.
Therefore, the solutions described above are the only solutions.

Solution 1.2
We will start by showing that there are positive integers x, c, d such that a = x2 cd and b = x3 c. Let
g = gcd(a, b) so that a = gd and b = gx for some coprime d and x. Then, b2 | a3 is equivalent to
g 2 x2 | g 3 d3 , which is equivalent to x2 | gd3 . Since x and d are coprime, this implies x2 | g. Hence,
g = x2 c for some c, giving a = x2 cd and b = x3 c as required.
Now, it remains to find all positive integers x, c, d satisfying

x2 cd − 1 | x3 c − 1.

That is, x3 c ≡ 1 (mod x2 cd − 1). Assuming that this congruence holds, it follows that d ≡ x3 cd ≡ x
(mod x2 cd − 1). Then, either x = d or x − d ≥ x2 cd − 1 or d − x ≥ x2 cd − 1.
• If x = d then b = a.
• If x − d ≥ x2 cd − 1, then x − d ≥ x2 cd − 1 ≥ x − 1 ≥ x − d. Hence, each of these inequalities must
in fact be an equality. This implies that x = c = d = 1, which implies that a = b = 1.
• If d − x ≥ x2 cd − 1, then d − x ≥ x2 cd − 1 ≥ d − 1 ≥ d − x. Hence, each of these inequalities must
in fact be an equality. This implies that x = c = 1, which implies that b = 1.
Hence the only solutions are the pairs (a, b) such that a = b or b = 1. These pairs can be checked to
satisfy the given conditions.

Official solutions reproduced from the Asian Pacific Mathematics Olympiad website: https://www.apmo-official.org

Mathematics Contests The Australian Scene 2022 | XXXIV Asian Pacific Mathematics Olympiad Solutions | 98


Solution 1.3
All answers are (n, n) and (n, 1) where n is any positive integer. They all clearly work.
To show that these are all solutions, note that we can easily eliminate the case a = 1 or b = 1. Thus,
assume that a, b = 1 and a = b. By the second divisibility, we see that a − 1 | b − a. However,
gcd(a, b) | b − a and a − 1 is relatively prime to gcd(a, b). This implies that (a − 1) gcd(a, b) | b − a,
 b−1

which implies gcd(a, b)  − 1.
a−1
b−1
The last relation implies that gcd(a, b) < , since the right-hand side are positive. However, due
a−1
to the first divisibility,
gcd(a, b)3 = gcd(a3 , b3 ) ≥ gcd(b2 , b3 ) = b2 .
Combining these two inequalities, we get that
2 b−1 b
b3 < <2 .
a−1 a
1 3
This implies a < 2b 3 . However, b2 | a3 gives b ≤ a 2 . This forces
3 1 √
a < 2(a 2 ) 3 = 2 a =⇒ a < 4.

Extracting a = 2, 3 by hand yields no additional solution.

2. Let ABC be a right triangle with ∠B = 90◦ . Point D lies on the line CB such that B is between D
and C. Let E be the midpoint of AD and let F be the second intersection point of the circumcircle of
ACD and the circumcircle of BDE. Prove that as D varies, the line EF passes through a fixed
point.

Mathematics Contests The Australian Scene 2022 | XXXIV Asian Pacific Mathematics Olympiad Solutions | 99
Solution
Solution 2.1
Let the line EF intersect the line BC at P and the circumcircle of ACD at G distinct from F . We
will prove that P is the fixed point.
First, notice that BED is isosceles with EB = ED. This implies ∠EBC = ∠EDP .
Then, ∠DAG = ∠DF G = ∠EBC = ∠EDP which implies AG  DC. Hence, AGCD is an isosceles
trapezoid.
∼ DEP and AG = DP .
Also, AG  DC and AE = ED. This implies AEG =
Since B is the foot of the perpendicular from A onto the side CD of the isosceles trapezoid AGCD,
we have P B = P D + DB = AG + DB = BC, which does not depend on the choice of D. Hence, the
initial statement is proven.
Solution 2.2
Set up a coordinate system where BC is along the positive x-axis, BA is along the positive y-axis,
and B is the origin. Take A = (0, a), B = (0, 0), C = (c, 0), D = (−d, 0) where a, b, c, d > 0. Then
E = (− d2 , a2 ). The general equation of a circle is

x2 + y 2 + 2f x + 2gy + h = 0 (1)

Substituting the coordinates of A, D, C into (1) and solving for f, g, h, we find that the equation of the
circumcircle of ADC is
cd
x2 + y 2 + (d − c)x + ( − a)y − cd = 0. (2)
a
Similarly, the equation of the circumcircle of BDE is
d2 a
x2 + y 2 + dx + ( − )y = 0. (3)
2a 2

Then (3)−(2) gives the equation of the line DF which is

a2 + d2 − 2cd
cx + y + cd = 0. (4)
2a
Solving (3) and (4) simultaneously, we get
 
c(d2 − a2 − 2cd) 2ac(c − d)
F = 2 2
, 2 ,
a + (d − 2c) a + (d − 2c)2
and the other solution D = (−d, 0).
From this we obtain the equation of the line EF which is ax + (d − 2c)y + ac = 0. It passes through
P (−c, 0) which is independent of d.
3. Find all positive integers k < 202 for which there exists a positive integer n such that
 n   2n  
kn

k
+ + ··· + = ,
202 202 202 2
where {x} denote the fractional part of x.
Note: {x} denotes the real number k with 0 ≤ k < 1 such that x − k is an integer.
Solution
Denote the equation in the problem statement as (*), and note that it is equivalent  to 
the condition
in
that the average of the remainders when dividing n, 2n, . . . , kn by 202 is 101. Since is invariant
202
in each residue class modulo 202 for each 1 ≤ i ≤ k, it suffices to consider 0 ≤ n < 202.

Mathematics Contests The Australian Scene 2022 | XXXIV Asian Pacific Mathematics Olympiad Solutions | 100
 
in
If n = 0, so is , meaning that (*) does not hold for any k. If n = 101, then it can be checked
202
that (*) is satisfied if and only if k = 1. From now on, we will assume that 101  n.
   
in in in
For each 1 ≤ i ≤ k, let ai = = − . Rewriting (*) and multiplying the equation by
202 202 202
202, we find that

n(1 + 2 + . . . + k) − 202(a1 + a2 + . . . + ak ) = 101k.

Equivalently, letting z = a1 + a2 + . . . + ak ,

nk(k + 1) − 404z = 202k.

Since n is not divisible by 101, which is prime, it follows that 101 | k(k + 1). In particular, 101 | k or
101 | k + 1. This means that k ∈ {100, 101, 201}. We claim that all these values of k work.

• If k = 201, we may choose n = 1. The remainders when dividing n, 2n, . . . , kn by 202 are 1, 2,
. . . , 201, which have an average of 101.
• If k = 100, we may choose n = 2. The remainders when dividing n, 2n, . . . , kn by 202 are 2, 4,
. . . , 200, which have an average of 101.
• If k = 101, we may choose n = 51. To see this, note that the first four remainders are 51, 102, 153,
2, which have an average of 77. The next four remainders (53, 104, 155, 4) are shifted upwards
from the first four remainders by 2 each, and so on, until the 25th set of the remainders (99,
150, 201, 50) which have an average of 125. Hence, the first 100 remainders have an average of
77 + 125
= 101. The 101th remainder is also 101, meaning that the average of all 101 remainders
2
is 101.

In conclusion, all values k ∈ {1, 100, 101, 201} satisfy the initial condition.

4. Let n and k be positive integers. Cathy is playing the following game. There are n marbles and k
boxes, with the marbles labelled 1 to n. Initially, all marbles are placed inside one box. Each turn,
Cathy chooses a box and then moves the marbles with the smallest label, say i, to either any empty
box or the box containing marble i + 1. Cathy wins if at any point there is a box containing only
marble n.
Determine all pairs of integers (n, k) such that Cathy can win this game.
Solution
We claim Cathy can win if and only if n ≤ 2k−1 .

First, note that each non-empty box always contains a consecutive sequence of labeled marbles. This
is true since Cathy is always either removing from or placing in the lowest marble in a box. As a
consequence, every move made is reversible.

Next, we prove by induction that Cathy can win if n = 2k−1 . The base case of n = k = 1 is trivial.
Assume a victory can be obtained for m boxes and 2m−1 marbles. Consider the case of m + 1 boxes
and 2m marbles. Cathy can first perform a sequence of moves so that only marbles 2m−1 , . . . , 2m are
left in the starting box, while keeping one box, say B, empty. Now move the marble 2m−1 to box B,
then reverse all of the initial moves while treating B as the starting box. At the end of that, we will
have marbles 2m−1 + 1, . . . , 2m in the starting box, marbles 1, 2, . . . , 2m−1 in box B, and m − 1 empty
boxes. By repeating the original sequence of moves on marbles 2m−1 + 1, . . . , 2m , using the m boxes
that are not box B, we can reach a state where only marble 2m remains in the starting box. Therefore

Mathematics Contests The Australian Scene 2022 | XXXIV Asian Pacific Mathematics Olympiad Solutions | 101
a victory is possible if n = 2k−1 or smaller.

We now prove by induction that Cathy loses if n = 2k−1 + 1. The base case of n = 2 and k = 1 is
trivial. Assume a victory is impossible for m boxes and 2m−1 +1 marbles. For the sake of contradiction,
suppose that victory is possible for m + 1 boxes and 2m + 1 marbles. In a winning sequence of moves,
consider the last time a marble 2m−1 + 1 leaves the starting box, call this move X. After X, there
cannot be a time when marbles 1, . . . , 2m−1 + 1 are all in the same box. Otherwise, by reversing these
moves after X and deleting marbles greater than 2m−1 + 1, it gives us a winning sequence of moves
for 2m−1 + 1 marbles and m boxes (as the original starting box is not used here), contradicting the
inductive hypothesis. Hence starting from X, marbles 1 will never be in the same box as any marbles
greater than or equal to 2m−1 + 1.

Now delete marbles 2, . . . , 2m−1 and consider the winning moves starting from X. Marble 1 would only
move from one empty box to another, while blocking other marbles from entering its box. Thus we
effectively have a sequence of moves for 2m−1 + 1 marbles, while only able to use m boxes. This again
contradicts the inductive hypothesis. Therefore, a victory is not possible if n = 2k−1 + 1 or greater.

5. Let a, b, c, d be real numbers such that a2 + b2 + c2 + d2 = 1. Determine the minimum value of


(a − b)(b − c)(c − d)(d − a) and determine all values of (a, b, c, d) such that the minimum value is
achieved.
Solution
1
The minimum value is − . There are eight equality cases in total. The first one is
8
 √ √ √ √ 
1 3 1 3 1 3 1 3
+ ,− − , − ,− + .
4 4 4 4 4 4 4 4

Cyclic shifting all the entries give three more quadruples. Moreover, flipping the sign ((a, b, c, d) →
(−a, −b, −c, −d)) all four entries in each of the four quadruples give four more equality cases.
Solution 5.1
Since the expression is cyclic, we could WLOG a = max{a, b, c, d}. Let

S(a, b, c, d) = (a − b)(b − c)(c − d)(d − a)

Note that we have given (a, b, c, d) such that S(a, b, c, d) = − 81 . Therefore, to prove that S(a, b, c, d) ≥
− 18 , we just need to consider the case where S(a, b, c, d) < 0.

• Exactly 1 of a − b, b − c, c − d, d − a is negative.
Since a = max{a, b, c, d}, then we must have d − a < 0. This forces a > b > c > d. Now, let us
write
S(a, b, c, d) = −(a − b)(b − c)(c − d)(a − d)
Write a − b = y, b − c = x, c − d = w for some positive reals w, x, y > 0. Plugging to the original
condition, we have

(d + w + x + y)2 + (d + w + x)2 + (d + w)2 + d2 − 1 = 0 (∗)

and we want to prove that wxy(w + x + y) ≤ 81 . Consider the expression (∗) as a quadratic in d:

4d2 + d(6w + 4x + 2y) + ((w + x + y)2 + (w + x)2 + w2 − 1) = 0

Mathematics Contests The Australian Scene 2022 | XXXIV Asian Pacific Mathematics Olympiad Solutions | 102
Since d is a real number, then the discriminant of the given equation has to be non-negative, i.e.
we must have

4 ≥ 4((w + x + y)2 + (w + x)2 + w2 ) − (3w + 2x + y)2


= (3w2 + 2wy + 3y 2 ) + 4x(w + x + y)
≥ 8wy + 4x(w + x + y)
= 4(x(w + x + y) + 2wy)

However, AM-GM gives us


 2
1 x(w + x + y) + 2wy 1
wxy(w + x + y) ≤ ≤
2 2 8

This proves S(a, b, c, d) ≥ − 18 for any a, b, c, d ∈ R such that a > b > c > d. Equality holds if and
only if w = y, x(w + x + y) = 2wy and wxy(w + x + y) = 18 . Solving these equations gives us
1
w4 = 16 which forces w = 12 since w > 0. Solving for x gives us x(x + 1) = 12 , and we will get
√ √
x = − 2 + 23 as x > 0. Plugging back gives us d = − 14 − 43 , and this gives us
1

 √ √ √ √ 
1 3 1 3 1 3 1 3
(a, b, c, d) = + ,− + , − ,− −
4 4 4 4 4 4 4 4

Thus, any cyclic permutation of the above solution will achieve the minimum equality.
• Exactly 3 of a − b, b − c, c − d, d − a are negative Since a = max{a, b, c, d}, then a − b has to
be positive. So we must have b < c < d < a. Now, note that

S(a, b, c, d) = (a − b)(b − c)(c − d)(d − a)


= (a − d)(d − c)(c − b)(b − a)
= S(a, d, c, b)

Now, note that a > d > c > b. By the previous case, S(a, d, c, b) ≥ − 81 , which implies that

1
S(a, b, c, d) = S(a, d, c, b) ≥ −
8
as well. Equality holds if and only if
 √ √ √ √ 
1 3 1 3 1 3 1 3
(a, b, c, d) = + ,− − , − ,− +
4 4 4 4 4 4 4 4

and its cyclic permutation.


Solution 5.2
1
The minimum value is − . There are eight equality cases in total. The first one is
8
 √ √ √ √ 
1 3 1 3 1 3 1 3
+ ,− − , − ,− + .
4 4 4 4 4 4 4 4

Cyclic shifting all the entries give three more quadruples. Moreover, flipping the sign ((a, b, c, d) →
(−a, −b, −c, −d)) all four entries in each of the four quadruples give four more equality cases. We then
begin the proof by the following optimization:

Claim 1. In order to get the minimum value, we must have a + b + c + d = 0.

Mathematics Contests The Australian Scene 2022 | XXXIV Asian Pacific Mathematics Olympiad Solutions | 103
a+b+c+d
Proof. Assume not, let δ = 4 and note that

(a − δ)2 + (b − δ)2 + (c − δ)2 + (d − δ)2 < a2 + b2 + c2 + d2 ,

so by shifting by δ and scaling, we get an even smaller value of (a − b)(b − c)(c − d)(d − a).

The key idea is to substitute the variables

x = ac + bd
y = ab + cd
z = ad + bc,

so that the original expression is just (x − y)(x − z). We also have the conditions x, y, z ≥ −0.5 because
of:

2x + (a2 + b2 + c2 + d2 ) = (a + c)2 + (b + d)2 ≥ 0.

Moreover, notice that


−1
0 = (a + b + c + d)2 = a2 + b2 + c2 + d2 + 2(x + y + z) =⇒ x + y + z = .
2
Now, we reduce to the following optimization problem.
Claim 2. Let x, y, z ≥ −0.5 such that x + y + z = −0.5. Then, the minimum value of

(x − y)(x − z)

is −1/8. Moreover, the equality case occurs when x = −1/4 and {y, z} = {1/4, −1/2}.

Proof. We notice that


  
1 1 1 1
(x − y)(x − z) + = 2y + z + 2z + y + +
8 2 2 8
  
1 2 1 1
= (4y + 4z + 1) + y + z+ ≥ 0.
8 2 2
1 1
The last inequality is true since both y + and z + are not less than zero.
2 2
1 1
The equality in the last inequality is attained when either y + = 0 or z + = 0, and 4y + 4z + 1 = 0.
2 2
This system of equations give (y, z) = (1/4, −1/2) or (y, z) = (−1/2, 1/4) as the desired equality cases.

Note: We can also prove (the weakened) Claim 2 by using Lagrange Multiplier, as follows. We first
prove that, in fact, x, y, z ∈ [−0.5, 0.5]. This can be proved by considering that

−2x + (a2 + b2 + c2 + d2 ) = (a − c)2 + (b − d)2 ≥ 0.

We will prove the Claim 2, only that in this case, x, y, z ∈ [−0.5, 0.5]. This is already sufficient to prove
the original question. We already have the bounded domain [−0.5, 0.5]3 , so the global minimum must
occur somewhere. Thus, it suffices to consider two cases:
• If the global minimum lies on the boundary of [−0.5, 0.5]3 . Then, one of x, y, z must be −0.5 or
0.5. By symmetry between y and z, we split to a few more cases.

Mathematics Contests The Australian Scene 2022 | XXXIV Asian Pacific Mathematics Olympiad Solutions | 104
– If x = 0.5, then y = z = −0.5, so (x − y)(x − z) = 1, not the minimum.
– If x = −0.5, then both y and z must be greater or equal to x, so (x − y)(x − z) ≥ 0, not the
minimum.
– If y = 0.5, then x = z = −0.5, so (x − y)(x − z) = 0, not the minimum.
– If y = −0.5, then z = −x, so

(x − y)(x − z) = 2x(x + 0.5),

which obtain the minimum at x = −1/4. This gives the desired equality case.
• If the global minimum lies in the interior (−0.5, 0.5)3 , then we apply Lagrange multiplier:

∂ ∂
(x − y)(x − z) = λ (x + y + z) =⇒ 2x − y − z = λ.
∂x ∂x
∂ ∂
(x − y)(x − z) = λ (x + y + z) =⇒ z − x = λ.
∂y ∂y
∂ ∂
(x − y)(x − z) = λ (x + y + z) =⇒ y − x = λ.
∂z ∂z
Adding the last two equations gives λ = 0, or x = y = z. This gives (x − y)(x − z) = 0, not the
minimum.
Having exhausted all cases, we are done.

Mathematics Contests The Australian Scene 2022 | XXXIV Asian Pacific Mathematics Olympiad Solutions | 105
2022 APMO Awards

See www.apmo-official.org/country_report/AUS/2022 for Australian awards and www.apmo-official.


org/year_report/2022 for country ranks and award distributions.

Mathematics Contests The Australian Scene 2022 | 2022 APMO Awards | 106
2022 AMOC Selection School

The 2022 AMOC Selection School was held online 28 March–2 April. The qualifying exam was the 2022
AMO.

A total of 83 students from around Australia attended the School. The breakdowns of the students into
the three streams were as follows.

Stream Female Male Total

Senior 6 15 21

Intermediate 6 14 21

Junior 11 30 41

Total 24 59 83

The main purposes of the Selection School are to:

• select the team of six for this year’s IMO from among the Senior group
• complete the final training of the already selected team of four for this year’s EGMO
• train students in all groups for possible future selection in an EGMO or IMO team.

The routine is similar to that for the AMOC School of Excellence.

Many thanks to Yifan Guo, Grace He, Claire Huang, Jongmin Lim, Dana Ma, Preet Patel, Thanom Shaw,
Andy Tran, Sally Tsang and Sampson Wong, who served as virtual live-in staff and Zoom hosts.

Thanks also to Adrian Agisilaou, Ross Atkins, Andres Buritica, Matthew Cheah, Jason Chi, Mikaela Gray,
Yasiru Jayasooriya, Charles Li, Micah Sinclair, Hadyn Tang, Ruiying Wu, Eliz- abeth Yevdokimov, Fredy
Yip and Jonathan Zheng, who assisted in lecturing and marking.

Angelo Di Pasquale
Director of Training, AMOC

Mathematics Contests The Australian Scene 2022 | AMOC Selection School | 107
2022 Australian EGMO team
Name Year School State
Eunsu Choi 11 Brisbane State High School QLD
Cloris Xu 10 Baulkham Hills High School NSW
Iris Xu 10 Baulkham Hills High School NSW
Grace Chang Yuan 12 Methodist Ladies’ College VIC
Reserve
Crystal Jin 11 James Ruse Agricultural High School NSW

2022 Australian IMO team


Name Year School State
William Cheah 8 Scotch College VIC
Sizhe Pan 11 James Ruse Agricultural High School NSW
Zian Shang 11 Scotch College VIC
Christopher Tran 11 The University High School VIC
Tony (Yuzheng) Wu 12 Killara High School NSW
Grace Chang Yuan 12 Methodist Ladies’ College VIC
Reserve
Cloris Xu 10 Baulkham Hills High School NSW

Mathematics Contests The Australian Scene 2022 | AMOC Selection School | 108
Participants at the 2022 AMOC Selection School
Name m/f Year School State
Senior
Joel Bariss m 10 Perth Modern School WA
William Cheah m 8 Scotch College VIC
Xiaohan (Taiga) Chen m 11 Yarra Valley Grammar School VIC
Jordan Chi m 11 Scotch College VIC
Eunsu Choi f 11 Brisbane State High QLD
Michael (Zixiang) Jiang m 11 Melbourne Grammar School VIC
Crystal (Yunxin) Jin f 11 James Ruse Agricultural High School NSW
Leo Li m 11 James Ruse Agricultural High School NSW
David Lumsden m 12 Scotch College VIC
Jayden Pan m 8 Shore School NSW
Sizhe Pan m 11 James Ruse Agricultural High School NSW
Zian Shang m 11 Scotch College VIC
Christopher Tran m 11 The University High School VIC
Leonardo Wang m 11 Scotch College VIC
Liah (Zizhen) Wu f 10 Prebyterian Ladies’ College VIC
Tony (Yuzheng) Wu m 12 Killara High School NSW
Cloris Xu f 10 Baulkham Hills High School NSW
Iris Xu f 10 Baulkham Hills High School NSW
Ryan (Dongqiang) Yang m 11 John Monash Science School VIC
Ethan Yap m 10 Perth Modern School WA
Grace Chang Yuan f 12 Methodist Ladies’ College VIC
Intermediate
Sophia Boyce f 11 Canberra Girls Grammar School ACT
Sunguo (Jerry) Chen m 10 Kelvin Grove State College QLD
Inez Crosby f 10 North Sydney Girls High School NSW
Joshua (Kezhuo) Deng m 8 Scotch College VIC
Matilda Grant f 9 A. B. Paterson College QLD
Jerry Jin m 10 Camberwell Grammar School VIC
Queensland Academy for Science, Mathematics and
Ian Lai m 9 QLD
Technology
Elizabeth (Xuan Xian) Lee f 10 Queensland Academy for Health Sciences QLD
Jingni Liao m 10 Knox Grammar School NSW
Jason Miao m 10 Knox Grammar School NSW
Laura (Xiangyue) Nan f 9 Fintona Girls’ School VIC
Levi Pesin m 8 Pesin Home School SA
Atharva Sathe m 10 Perth Modern School WA
Harrison Su m 10 James Ruse Agricultural High School NSW
Miranda Sui f 10 Prebyterian Ladies’ College VIC
Ben Szabo-Virag m 10 Matthew Flinders Anglican College QLD
Oliver Taylor m 10 Caulfield Grammar School VIC
Justin Tran m 9 Sydney Grammar School NSW
Alysa Xu f 10 Shanghai Weiyu High School China
Eason Yang m 9 Trinity Grammar VIC
Yitong Zheng m 10 Indooroopilly State High School QLD

Mathematics Contests The Australian Scene 2022 | AMOC Selection School | 109
Name m/f Year School State
Junior
Alexander Artemov m 9 Scotch College VIC
Max Beckmann m 10 St Joseph’s College Gregory Terrace QLD
Casper Cai m 10 St Peter’s College SA
Albert Chen m 10 Melbourne Grammar School VIC
Garry Chen m 10 Anglican Church Grammar School QLD
George Chen m 9 James Ruse Agricultural High School NSW
Bartholomew Cheng m 10 Cairns State High School QLD
Beatrice Chong f 9 Perth Modern School WA
Anoushka Gupta f 9 Perth Modern School WA
Michelle (Xueyao) Han f 9 Caulfield Grammar School VIC
Ella Job f 10 Somerville House QLD
Amber (Yanfei) Li f 8 Pymble Ladies’ College NSW
Jiamu Li m 10 Scotch College VIC
Rhianna Li f 8 James Ruse Agricultural High School NSW
Aidan Lim m 9 Christ Church Grammar School WA
Adrian Lin m 10 Scotch College VIC
Kevin (Hangzhi) Liu m 6 Haileybury VIC
Matthew Lo m 9 Trinity Grammar VIC
Quan Nguyen m 10 Anglican Church Grammar School QLD
Yangyang (Qiyang) Ning m 9 James Ruse Agricultural High School NSW
Witter Onggara m 9 Scotch College VIC
Kathryn Petersen f 10 Brisbane Girls Grammar School QLD
Brendan Plover m 8 Penleigh and Essendon Grammar School VIC
Ee Lin Saw f 9 Somerville House QLD
Mohijit Singh m 9 Baulkham Hills High School NSW
Johnson Tang m 10 The King’s School NSW
Jeremiah Wang m 10 Christ Church Grammar School WA
Luke Wang m 9 James Ruse Agricultural High School NSW
Matthew (Cunmeng) Wang m 7 Scotch College VIC
Jerry Qihao Xia m 8 Scotch College VIC
Max (Beikun) Yan m 10 Anglican Church Grammar School QLD
Ryan Yang m 7 Scotch College VIC
Yiheng You m 10 James Ruse Agricultural High School NSW
Victor Yuan m 9 James Ruse Agricultural High School NSW
Ben Zhang m 10 St Joseph’s College Gregory Terrace QLD
Ivy Zhang f 10 Penleigh and Essendon Grammar School VIC
Jayee Zhang m 7 Scotch College VIC
Daniel Zhou m 7 Haileybury VIC
Ella Liu Zhou f 10 Brisbane State High School QLD
Leon Zhou m 8 James Ruse Agricultural High School NSW
Louisa Zhu f 10 Penleigh and Essendon Grammar School VIC

Mathematics Contests The Australian Scene 2022 | AMOC Selection School | 110
The 11th European Girls' Mathematical Olympiad

The 11th EGMO, Eger, Hungary (hybrid)


The 11th European Girls’ Mathematical Olympiad (EGMO) was held 6–12 April 2022 as a hybrid event —
in the city of Eger, Hungary, for the 38 teams able to participate on site (31 of which were considered
official European teams) and online for the 19 teams unable to travel. With 222 contestants from a total
of 57 teams, this was the highest participation at an EGMO to date.

The four members of the 2022 Australian EGMO team were Iris Xu, Grace Chang Yuan, Cloris Xu and
Eunsu Choi pictured below (clockwise from the top left). Also part of the Australian contingent were
Thanom Shaw (Team Leader), Yiying (Sally) Tsang (Deputy Team Leader), and Grace He and Claire
Huang (both Observers with the Deputy). Crystal Jin was the reserve team member.

Dream team

For this hybrid EGMO, contestants on site in Eger sat contest papers 1 and 2 from 08:30 to 13:00 CEST
on April 8th and 9th. The Australian team sat both papers in the same time frame (which translated to
16:30 to 21:00 AEST) but from home and supervised on Zoom.

Mathematics Contests The Australian Scene 2022 | The 11th European Girls' Mathematical Olympiad | 111
Australian team sitting the exam

EGMO contestants sitting exam

At the EGMO there are six contest problems selected by a Problems Selection Committee from those
submitted by participating countries (and finally approved by the jury). The six problems on the 2022
EGMO contest papers may be described as follows.

1. A classical geometry problem, proposed by Netherlands

2. A number theory problem involving finding all possible functions on the natural numbers satisfying
particular conditions, proposed by Ukraine

3. A number theory problem involving an infinite sequence that eventually becomes constant, proposed
by United Kingdom

4. An algebra problem involving determining the largest possible set of real numbers given particular
conditions, proposed by Romania

5. A combinatorics problem involving domino tilings and parity, proposed by United States of America

6. A geometry problem involving up to 6 concyclic points, proposed by Australia.

Mathematics Contests The Australian Scene 2022 | The 11th European Girls' Mathematical Olympiad | 112
The score distributions by problem number were as follows.

Mark P1 P2 P3 P4 P5 P6
0 34 131 153 33 75 91
1 32 16 39 16 57 90
2 13 17 9 11 17 18
3 2 16 6 10 17 10
4 2 10 1 1 10 3
5 2 8 1 4 8 2
6 4 3 1 30 4 1
7 133 21 12 117 34 7
Mean 4.67 1.55 0.78 4.91 2.18 1.05

Overall, contestants found the problems of the 11th EGMO more accessible than in the previous two years
with an average score of 15.14 (out of 42) compared to 9.77 in 2021, 13.01 in 2020. For the 11th EGMO the
medal cut-offs were set at 29 points for Gold, 22 for Silver and 15 for Bronze. The medal distributions1
were as follows.

Gold Silver Bronze Total


All teams (222 contestants) 23 32 62 117
Official European teams (123 contestants) 10 22 33 65
Proportion from official European teams 8.1% 17.9% 26.8% 52.8%

Of those who did not get a medal, a further 47 contestants (27 from official European teams) re-
ceived an Honourable Mention for scoring full marks on at least one problem. The highest score
achieved was 42 out of 42 and was achieved by Jessica Wan from the United States of America and
Taisiia Korotchenko and Galiia Sharafetdinova participating as individual contestants.

Congratulations to the Australian team on a phenomenal performance at their fifth appearance at an


EGMO. The team finished an historic 3rd out of 57 teams in the rankings 2, building on its strong past
performances of 20th in 2018, 14th in 2019, equal 12th in 2020, and 12th in 2021.

Furthermore, each of Australia’s four team members achieved Gold medals with scores placing them in
the top 23 of the 222 contestants of the 11th EGMO, 2022.

• Grace Chang Yuan, Year 12, Methodist Ladies’ College, VIC (Gold, ranked equal 6th)
• Cloris Xu, Year 10, Baulkham Hills High School, NSW (Gold, ranked equal 12th)
• Eunsu Choi, Year 11, Brisbane State High School, QLD (Gold, ranked equal 15th)
• Iris Xu, Year 10, Baulkham Hills High School, NSW (Gold, ranked equal 20th)

1The total number of medals is approximately half the number of contestants. The numbers of Gold,
Silver, and Bronze medals are in the approximate ratio 1:2:3 and are chosen on the basis of the
performances of members of official European teams (which in 2022 were those European teams
participating on site). Medals are awarded to participants not on official European teams on the basis
of the boundaries set.

2Countries are ranked each year on the EGMO’s official website according to the sum of the individual
student scores from each country.

Mathematics Contests The Australian Scene 2022 | The 11th European Girls' Mathematical Olympiad | 113
The following table shows the Australian EGMO team’s scores in detail.

Name P1 P2 P3 P4 P5 P6 Score Award


Grace Chang Yuan 7 7 6 6 7 1 34 Gold Medal

Cloris Xu 7 7 2 7 7 2 32 Gold Medal

Eunsu Choi 7 4 7 7 5 1 31 Gold Medal

Iris Xu 7 7 0 7 7 1 29 Gold Medal

Australian Average 7 6.25 3.75 6.75 6.5 1.25 31.5

EGMO average 4.67 1.55 0.78 4.91 2.18 1.05 15.14

The table below shows the distribution of awards for each country at the 11th EGMO, 2022. This and other
statistical information can also be found on the official website of the EGMO, https://www.egmo.org.

Country Size Total Gold Silver Bronze HM Rank


Albania 4 10 0 0 0 0 54
Australia 4 126 4 0 0 0 3
Austria 2 25 0 0 1 0 46
Azerbaijan 4 69 1 0 2 1 23
Bangladesh 4 57 0 0 2 2 29
Belarus 4 76 1 0 3 0 20
Belgium 3 33 0 1 0 0 41
Bolivia 4 7 0 0 0 1 55
Bosnia and Herzegovina 4 83 1 0 3 0 16
Brazil 4 68 0 0 3 0 24
Bulgaria 4 102 1 2 1 0 5
Canada 4 89 0 3 1 0 11
Costa Rica 4 31 0 0 1 1 43
Croatia 4 60 0 1 0 3 26
Cyprus 4 32 0 0 1 1 42
Czech Republic 4 46 0 0 2 0 36
Denmark 4 25 0 0 0 2 46
Ecuador 4 3 0 0 0 0 57
Finland 4 20 0 0 0 0 50
France 4 79 0 1 2 1 17
Georgia 4 55 0 1 1 1 33
Germany 4 90 1 1 2 0 10
Greece 4 24 0 0 0 2 48
Hungary 4 96 1 1 2 0 8
Hungary B 4 77 0 2 1 0 19
India 4 78 0 0 4 0 18
Ireland 4 14 0 0 0 0 51
Israel 4 91 1 0 3 0 9
Italy 4 88 0 2 2 0 13
Japan 4 74 1 0 1 1 21
Kazakhstan 4 62 0 0 2 2 25

Mathematics Contests The Australian Scene 2022 | The 11th European Girls' Mathematical Olympiad | 114
Country Size Total Gold Silver Bronze HM Rank
Kyrgyzstan 4 7 0 0 0 0 55

Kosovo 4 11 0 0 0 1 53

Lithuania 4 26 0 0 0 2 45

Latvia 4 37 0 0 1 2 39

Mexico 4 85 0 2 2 0 15

Mongolia 4 58 0 0 2 2 28

Netherlands 4 47 0 1 0 1 35

North Macedonia 4 56 0 0 2 2 32

Norway 4 23 0 0 0 1 49

Peru 4 130 3 1 0 0 2

Poland 4 89 1 2 0 1 11

Republic of Moldova 4 57 0 1 2 0 29

Romania 4 112 1 3 0 0 4

Saudi Arabia 4 53 0 0 1 3 34

Serbia 4 57 0 0 1 3 29

Spain 4 43 0 0 1 2 38

Switzerland 4 60 0 1 1 1 26

Slovakia 4 70 0 1 2 1 22

Slovenia 4 45 0 0 1 2 37

Syria 2 29 0 0 1 1 44

Tajikistan 4 37 0 0 0 3 39

Tunisia 3 12 0 0 0 1 52

Turkey 4 102 1 2 1 0 5

Ukraine 4 100 1 2 1 0 7

United Kingdom 4 86 1 0 3 0 14

United States of America 4 140 3 1 0 0 1

Total (57 teams, 222 contestants) 23 32 62 47

While we were unable to make it to Europe this year, we did our best to get in the spirit of an event
hosted by Hungary, enjoying watching live the opening and closing ceremonies, complete with Hungarian
music and dancing, and with some Hungarian treats on the side too.

Mathematics Contests The Australian Scene 2022 | The 11th European Girls' Mathematical Olympiad | 115
FLT and Kurtosh Kurtosh being made Hungarian treats

Closing ceremony
There is no doubt that the 2022 Australian EGMO team dearly missed the usual face-to-face training
throughout the year and the adventures that come with travelling abroad. Indeed, throughout their
several years of training, the four team members have only met at online AMOC schools and online
training sessions, but never in person (except for sisters Cloris and Iris, of course). Still, inspired by
each other and working independently with such grit and determination, each team member took
their problem-solving skills to the highest of levels and performed incredibly on the international EGMO
stage. Placing 3rd overall and taking home a clean sweep of Gold medals, the performance of Australia’s
fifth ever EGMO team has certainly been one for the history books. EGMO 2023 is to be held in April in
Portorož, Slovenia and we very much look forward to making it back to Europe in person and spreading
some much-missed Aussie cheer.

Thanom Shaw
EGMO Team Leader, Australia

Mathematics Contests The Australian Scene 2022 | The 11th European Girls' Mathematical Olympiad | 116
Paper

Language: English
EGMO 2022 Day: 1
Hungary Eger
European Girls’ Mathematical Olympiad

Friday, April 8, 2022

Problem 1. Let ABC be an acute-angled triangle in which BC < AB and BC < CA. Let point P
lie on segment AB and point Q lie on segment AC such that P = B, Q = C and BQ = BC = CP .
Let T be the circumcentre of triangle AP Q, H the orthocentre of triangle ABC, and S the point of
intersection of the lines BQ and CP . Prove that T , H and S are collinear.

Problem 2. Let N = {1, 2, 3, . . .} be the set of all positive integers. Find all functions f : N → N
such that for any positive integers a and b, the following two conditions hold:

(1) f (ab) = f (a)f (b), and

(2) at least two of the numbers f (a), f (b) and f (a + b) are equal.

Problem 3. An infinite sequence of positive integers a1 , a2 , . . . is called good if

(1) a1 is a perfect square, and

(2) for any integer n ≥ 2, an is the smallest positive integer such that

na1 + (n − 1)a2 + . . . + 2an−1 + an

is a perfect square.

Prove that for any good sequence a1 , a2 , . . ., there exists a positive integer k such that an = ak for all
integers n ≥ k.

Language: English Time: 4 hours and 30 minutes


Each problem is worth 7 points

To make this a fair and enjoyable contest for everyone, please do not mention or refer
to the problems on the internet or on social media until Saturday 9 April, 22:00 UTC
(15:00 Pacific Daylight Time, 00:00 (Sunday) Central European Summer Time, 08:00
(Sunday) Australian Eastern Standard Time).

Mathematics Contests The Australian Scene 2022 | 11th European Girls' Mathematical Olympiad Paper | 117
Language: English
EGMO 2022 Day: 2
Hungary Eger
European Girls’ Mathematical Olympiad

Saturday, April 9, 2022

Problem 4. Given a positive integer n ≥ 2, determine the largest positive integer N for which there
exist N + 1 real numbers a0 , a1 , . . . , aN such that
1
(1) a0 + a1 = − , and
n
(2) (ak + ak−1 )(ak + ak+1 ) = ak−1 − ak+1 for 1 ≤ k ≤ N − 1.

Problem 5. For all positive integers n, k, let f (n, 2k) be the number of ways an n × 2k board can
be fully covered by nk dominoes of size 2 × 1. (For example, f (2, 2) = 2 and f (3, 2) = 3.)
Find all positive integers n such that for every positive integer k, the number f (n, 2k) is odd.

Problem 6. Let ABCD be a cyclic quadrilateral with circumcentre O. Let the internal angle
bisectors at A and B meet at X, the internal angle bisectors at B and C meet at Y , the internal angle
bisectors at C and D meet at Z, and the internal angle bisectors at D and A meet at W . Further, let
AC and BD meet at P . Suppose that the points X, Y, Z, W, O and P are distinct.
Prove that O, X, Y, Z and W lie on the same circle if and only if P, X, Y, Z and W lie on the same
circle.

Language: English Time: 4 hours and 30 minutes


Each problem is worth 7 points

To make this a fair and enjoyable contest for everyone, please do not mention or refer
to the problems on the internet or on social media until Saturday 9 April, 22:00 UTC
(15:00 Pacific Daylight Time, 00:00 (Sunday) Central European Summer Time, 08:00
(Sunday) Australian Eastern Standard Time).

Mathematics Contests The Australian Scene 2022 | 11th European Girls' Mathematical Olympiad Paper | 118
European Girls' Mathematical Olympiad
Solutions
Solutions to the 2022 European Girls’ Mathematical Olympiad

1. Solution (Grace Chang Yuan, year 12, Methodist Ladies’ College, VIC. Grace was a
Gold medallist with the 2022 Australian EGMO team.)
To prove that T , S, H are collinear, it suffices to prove that ∠BSH = ∠QST .

Q
T

P S

B C

Since BQ = BC and BH ⊥ CQ (H is the orthocentre of ABC), BH is the angle


bisector of ∠CBQ. Similiarly, CH is the angle bisector of ∠P CB. Hence H is the
incentre of SBC and SH bisects ∠BSC.
Now let ∠BAC = ∠P AC = ∠P AQ = α, ∠CBA = ∠CBP = ∠BP C = β (since
BC = CP ), and ∠ACB = ∠QCB = ∠BQC = γ (since BQ = BC). Also note that
α + β + γ = 180◦ .
We have that ∠QCS = ∠ACP = ∠BP C − ∠P AC = β − α and ∠SQC = ∠BQC = γ.
Since the angle sum of QSC is 180◦ = α + β + γ, ∠CSQ = 2α, ∠BSC = 180◦ − 2α,
and ∠BSH = 12 ∠BSC = 90◦ − α.
Now since T is the circumcentre of AP Q, ∠P T Q = 2∠P AQ = 2α. Hence ∠P T Q =
∠CSQ and SP T Q is cyclic. Furthermore, since T P and T Q are equal radii, ∠QP T =
∠T QP = 90◦ − α and since SP T Q is cyclic, ∠QST = ∠QP T = 90◦ − α.
Thus ∠BSH = ∠QST and T , S, H are collinear, as required. 

Mathematics Contests The Australian Scene 2022 | 11th European Girls' Mathematical Olympiad Solutions | 119
2. Solution (Based on the solution by Iris Xu, year 10, Baulkham Hills High School, NSW.
Iris was a Gold medallist with the 2022 Australian EGMO team.)
Answer For any fixed prime p and any fixed positive integer A, a solution is f (x) = Ak
where pk  x. (Recall that pk  x means that pk is the highest power of p that divides x.)
We shall prove that the answer stated above constitutes all solutions to the problem.
For each x ∈ N, we can write out its prime factorisation
x = pa11 pa22 . . . pakk .
Using repeated applications of condition (1), we get
f (x) = f (p1 )a1 f (p2 )a2 . . . f (pk )ak . (*)
We shall refer to result (*) quite a bit in the proof.
Put a = 1 into condition (1) to find f (b) = f (1)f (b). Since f (b) ≥ 1, this implies f (1) = 1.
Case 1 f (p) = 1 for all primes p
For each x ∈ N, write out its prime factorisation x = pa11 pa22 . . . pakk . From (*), we get
f (x) = f (p1 )a1 f (p2 )a2 . . . f (pk )ak = 1a1 1a2 . . . 1ak = 1.
Therefore f (x) = 1, for all x ∈ N, which is covered by A = 1 in the answer stated above.
A simple substitution shows that it works.
Case 2 There are two primes p < q such that f (p) = 1 and f (q) = 1.
Without loss of generality, we may assume that p and q are the smallest two primes with
the above property. Thus f (r) = 1, for all primes r < q with r = p. It follows from (*)
that f (x) = 1 for all positive integers x < q that are relatively prime to p.
Consider q − p. Note that 1 ≤ q − p < q, and q − p is relatively prime to both p and q.
Thus f (q − p) = 1.
From condition (2), two of f (p), f (q − p), f (q) are the same. Since f (p), f (q) > 1 and
f (q − p) = 1, we get that f (p) = f (q).
Let c be the unique non-negative integer such that cq < p2 < (c + 1)q. Since q > p, this
implies c < p, and so f (c) = 1.
Consider p2 − cq. We have gcd(p, p2 − cq) = gcd(p, cq) = gcd(p, c) = 1. Note that we
chose c so that 0 < p2 − cq < q. Thus f (p2 − cq) = 1. But we also have f (p2 ) = f (p)2
and f (cq) = f (c)f (q) = 1 · f (q) = f (p).
From condition (2), two of 1, f (p) and f (p)2 are equal. Regardless of which two are equal,
we get f (p) = 1. This contradiction shows that there are no solutions for this case.
Case 3 There is exactly one prime p such that f (p) = 1.
Let p = pi be the ith prime, and let f (pi ) = A, where A is a positive integer.
For each x ∈ N, write out its prime factorisation x = pa11 pa22 . . . pakk . From (*), we get
f (x) = f (p1 )a1 f (p2 )a2 . . . f (pk )ak = 1a1 1a2 . . . f (p)ai . . . 1ak = Aai .

Therefore f (x) = Ak where pk  x.


It remains to check that this really works. Let pα  a and pβ  b.
Note that f (a)f (b) = pα pβ = pα+β = f (ab), and so condition (1) holds.
To check condition (2), if α = β then f (a) = f (b). And if α = β, then pmin(α,β)  a + b,
and so f (a + b) is equal to one of f (a) or f (b). So condition 2 also holds. 

Mathematics Contests The Australian Scene 2022 | 11th European Girls' Mathematical Olympiad Solutions | 120
3. Solution (Eunsu Choi, year 11, Brisbane State High School, QLD. Eunsu was a Gold
medallist with the 2022 Australian EGMO team.)
For all positive integers n, let
Sn2 = na1 + (n − 1)a2 + · · · + 2an−1 + an .

Note that
Sn2 − Sn−1
2
= a1 + a2 + · · · + an
and therefore    2 
an = Sn2 − Sn−1
2 2
− Sn−1 − Sn−2 .
Lemma: Sn+2 − Sn+1 ≤ Sn+1 − Sn , for all integers n ≥ 1.
Proof: Assume for the sake of contradiction that Sn+2 − Sn+1 > Sn+1 − Sn and so
Sn+2 = 2Sn+1 − Sn + x,
for some positive integer x.
We then have that
 2 2
  2 
an+2 = Sn+2 − Sn+1 − Sn+1 − Sn2
= (2Sn+1 − Sn + x)2 − 2Sn+1
2
+ Sn2
2
= 2Sn+1 + 2Sn2 − 4Sn Sn+1 + x (x − 2Sn + 4Sn+1 )
and now show that this value for an+2 is not minimal as required.
Let
2
bn+2 = 2Sn+1 + 2Sn2 − 4Sn Sn+1 < an+2
since x(x − 2Sn + 4Sn+1 ) > 0 (as Sn < Sn+1 and x > 0). Also note that bn+2 > 0 as
bn+2 = 2 (Sn+1 − Sn )2 > 0.
Now
(n + 2)a1 + (n + 1)a2 + · · · + 2an+1 + bn+2 = (n + 2)a1 + (n + 1)a2 + · · · + 2an+1 + an+2
− x(x − 2Sn + 4Sn+1 )
2
= Sn+2 − x(x − 2Sn + 4Sn+1 )
= (2Sn+1 − Sn + x)2 − x2 + 2xSn − 4xSn+1
2
= 4Sn+1 + Sn2 − 4Sn+1 Sn
= (2Sn+1 − Sn )2
which is a perfect square. Hence an+2 is not minimal and we have a contradiction.
Therefore Sn+2 − Sn+1 ≤ Sn+1 − Sn for all integers n ≥ 1.
Since Sn < Sn+1 , we have that the difference Sn+1 − Sn is always positive and so bounded
below. From the lemma above, this difference is also decreasing and so after a certain
point, Sn+1 − Sn will be constant.
Let k be the integer such that Sn − Sn−1 = Sn−1 − Sn−2 = c for some constant c, for all
n ≥ k. Then for all such n,
   2 
an = Sn2 − Sn−1
2
− Sn−1 − Sn−22

= c × (Sn + Sn−1 ) − c × (Sn−1 + Sn−2 )


= c × ((Sn − Sn−1 ) + (Sn−1 − Sn−2 ))
= c × (c + c)
= 2c2 .

So there exists a positive integer k such that an = ak for all integers n ≥ k. 

Mathematics Contests The Australian Scene 2022 | 11th European Girls' Mathematical Olympiad Solutions | 121
4. Solution (Eunsu Choi, year 11, Brisbane State High School, QLD. Eunsu was a Gold
medallist with the 2022 Australian EGMO team.)
Answer N = n.
First, we will prove that N ≤ n.
We prove by induction that for k = 0, 1, 2, . . . , n − 1,
1
ak + ak+1 = − .
n−k

The base case, k = 0, is true from (1).


1
For the inductive step, suppose that ak−1 + ak = − . Then from (2), we get
n−k+1

(ak + ak−1 )(ak + ak+1 ) = (ak−1 + ak ) − (ak + ak+1 )


1 1
− (ak + ak+1 ) = − − (ak + ak+1 )
 n − k +1 n − k + 1
1 1
1− (ak + ak+1 ) = −
n−k+1 n−k+1
n−k+1−1 1
(ak + ak+1 ) = −
n−k+1 n−k+1
1
ak + ak+1 = − .
n−k
So the inductive step holds and our claim is true by induction.
It follows that an−1 + an = −1.
Now if N > n, then (2) would hold for k = n and we would have

−(an + an+1 ) = an−1 − an+1 =⇒ −1 = an−1 + an = 0.

Clearly this is a contradiction, so N ≤ n.


We now prove that N = n by constructing a0 , . . . , an satisfying (1) and (2).
Define
1
a0 = 0 and ak+1 = − − ak for 0 ≤ k ≤ n − 1.
n−k
This gives a1 = − n1 and so (1) is clearly satisfied.
We now check that (2) is satisfied. For 1 ≤ k ≤ n − 1,
1 1
LHS = (− − ak−1 + ak−1 )(− − ak + ak )
n−k+1 n−k
1
=
(n − k + 1)(n − k)
1 1
RHS = (− − ak ) − (− − ak )
n−k+1 n−k
1 1
=− +
n−k+1 n−k
1
=
(n − k + 1)(n − k)

and so LHS = RHS and (2) is satisfied. It follows that N = n. 

Mathematics Contests The Australian Scene 2022 | 11th European Girls' Mathematical Olympiad Solutions | 122
5. Solution (Based on solutions by Cloris Xu and Iris Xu, Year 10, Baulkham Hills High
School, NSW. Cloris and Iris were Gold Medalists with the 2022 Australian EGMO team.)

We note that there is only one way to tile a 1 × 2k board, so f (1, 2k) = 1. Hence n = 1
satisfies the condition.
Now we will show that n even does not satisfy the condition. Label each small square of
an n × n board (i, j) if it is in the ith row and jth column. Let An be the set of tilings of
the n × n board where a domino covers (1, 1) and (1, 2), and Bn the set of tilings where
a domino covers (1, 1) and (2, 1). Clearly,

f (n, n) = |An | + |Bn |.

We claim there is a bijection between tilings in An and Bn as follows: Take any tiling in
An and reflect it along the diagonal of the n × n board. This takes the domino on (1, 1)
and (1, 2) to (1, 1) and (2, 1), so we obtain a tiling in Bn . As every tiling in An maps
to a different tiling in Bn , and all tilings in Bn are achievable, this is a bijection. Hence
|An | = |Bn |, so f (n, n) is even.
Now assume n is odd; let n = 2m + 1. Consider the board with 2k rows and 2m + 1
columns. Construct a vertical line l through the centre of the (m + 1)th column. Let
Cn be the set of tilings of this board which are symmetric about l and Dn be the set of
tilings which are not symmetric about l, so that

f (n, 2k) = |Cn | + |Dn |. (1)

For each tiling in Dn , a reflection about l generates a different tiling in Dn . Hence |Dn | is
even. Consider a tiling in Cn . Since it is symmetric about l, the (m + 1)th column must
be tiled entirely by vertical dominoes. Thus, a tiling of the left m × 2k squares determines
the tiling of the whole board:
|Cn | = f (m, 2k). (2)

From 1 and 2, we have

f (2m + 1, 2k) is odd ⇐⇒ f (m, 2k) is odd. (3)

Now we show by induction on positive integers q that for 2q−1 ≤ n < 2q , f (n, 2k) is odd
if and only if n = 2q − 1. Clearly this is true for q = 1. Assume true for q = r, a positive
integer. Then if f (n, 2k) is odd for 2r ≤ n < 2r+1 , from 3, f ( n−1
2
, 2k) is also odd. Since
r r+1 r−1 n−1 r
n must be odd, 2 < n < 2 and so 2 ≤ 2 < 2 . Hence, by the assumption,
n−1 r r+1
2
= 2 − 1, so n = 2 − 1, completing the induction.
Based on all the above, the only n which meet the specified criteria are of the form
n = 2q − 1, for some positive integer q. 
[Note: The final result can also be shown by noticing that for every positive integer m,
the sequence 2m + 1, m,  m2 , . . . consisting of positive integers is strictly decreasing. For
f (n, 2k) to be odd, this sequence must never contain an even number, so must terminate
at 1. Therefore the sequence 1, 3, 7, . . . must comprise exactly those n that meet the
criteria, and it is easy to show that these are all those of the form 2q − 1.]

Mathematics Contests The Australian Scene 2022 | 11th European Girls' Mathematical Olympiad Solutions | 123
6. Solution (Offical Solution)
Let Ω be the circumcircle of the quadrilateral ABCD and let r be its radius.
First note that since
   
∠B + ∠C ∠A + ∠D
∠XY Z + ∠ZW X = ◦
180 − ◦
+ 180 − = 180◦ ,
2 2

XY ZW is cyclic.
Let ω be the circle passing through these four points. Our goal is to prove that O ∈ ω if
and only if P ∈ ω.
Now if ABCD is an isosceles trapezium, without loss of generality say AB  CD, then
by symmetry, points X, Z, O and P lie on the axis of symmetry (that is, the common
perpendicular bisector of AB and CD) and hence are collinear. By the conditions of the
problem, these four points are distinct and X and Z are on ω, so neither O, nor P can
lie of ω and hence the problem statement holds true.
Henceforth, assume that AB ∦ CD and BC ∦ AD.

O
Y
ω
X

W
B Z
P
D
C

Q S R

Let AB and CD meet at Q, and let BC and AD meet at R. Without loss of generality,
suppose that B lies between A and Q, and D lies between A and R. We now show that
QR is the radical axis of circles Ω and ω.
Point W is the intersection point of the bisectors ∠A and ∠D, so in triangle ADQ, point
W is the incentre. Similarly, BY and CY are the external bisectors of ∠QBC and ∠BCQ,
so in triangle BCQ, point Y is the excentre opposite of Q. Hence, both Y and W lie on
the bisector of ∠DQA.

Mathematics Contests The Australian Scene 2022 | 11th European Girls' Mathematical Olympiad Solutions | 124
∠B−∠A
Since ∠DQA = 180◦ − ∠D − ∠A = ∠B − ∠A, ∠Y QA = 2
and we have

∠B ∠B − ∠A ∠A
∠BY W = ∠BY Q = ∠Y BA − ∠Y QA = − = = ∠BAW,
2 2 2
so the points A, B, Y, W are concyclic and therefore QA · QB = QY · QW . Hence Q has
equal power with respect to Ω and ω. Similarly, R has equal power with respect to Ω and
ω. Therefore, the radical axis of Ω and ω is line QR.
Let OP and QR intersect at point S. By the fundamental theorem of poles and polars,
P QR is autopolar and in particular, QR is the polar of P with respect to Ω. Conse-
quently, OP ⊥ QR and points S and P are inverses of eachother with respect to Ω, so
OS · OP = r2 .
Since P and O lie inside Ω, the polar line QR lies entirely outside, so S is distinct from
P and O. Moreover,

SO · SP = OS · (OS − OP ) = OS 2 − OS · OP = SO2 − r2 ,

so SO · SP is equal to the power of S with respect to Ω. Since S lies on the radical axis
QR, it has equal power with respect to Ω and ω; therefore, SO · SP is equal to the power
of S with respect to ω. From this, it follows that O ∈ ω if and only if P ∈ ω. 

Mathematics Contests The Australian Scene 2022 | 11th European Girls' Mathematical Olympiad Solutions | 125
2022 Mentor Program

Students who attended the AMOC Selection School are invited to participate in the mentor program
which runs from April to July.

For Seniors, the purpose of the mentor program is to prepare the IMO team for the IMO, and prepare
other non-year-12 Seniors to be serious IMO team contenders in future years. This is our top tier of
training. In 2022 it consisted of doing 15 trial IMO papers. Students completed one or two such training
papers each week.

For Juniors and Intermediates, the purpose of the mentor program is to help them train up for
the AMOC Senior Contest held in August. This contest is used to select the Seniors and some
Intermediates for the AMOC School of Excellence. In 2022 it consisted of 6 training papers sent out at
fortnightly intervals.

For all streams, their work was marked, feedback given, and solutions discussed with each group using
the Zoom platform.

I am pleased to report that most students who attended the 2022 AMOC Selection School accepted
the invitation to take part in the mentor program.

Angelo Di Pasquale
IMO Team Leader

Mathematics Contests The Australian Scene 2022 | Mentor Program | 126


2022 IMO Team Preparation School

This year's joint pre-IMO training camp with the UK IMO team was held in separate locations due to
both teams mitigating risks associated with COVID-19. While the UK team trained in Cambridge, we
trained in Oslo in the very same hotel that would later host our team for the IMO itself. As usual, both
teams sat the same practice exams agreed to beforehand by the trainers from both teams.

We arrived in Oslo on the afternoon of 3 July. On each of the next five days the Australian team sat a
4.5 hour trial IMO in the morning. The exams were marked so that later that afternoon there would be
a quick debrief and the students would get their results back.

The final exam was the Mathematical Ashes which is part of our ongoing friendly rivalry with the UK
team. The result was 8172 in the UK's favour. Our congratulations go to the UK! Maybe we can win
them back next year.

My thanks go to Sampson Wong and Hadyn Tang who cared for the bulk of academic matters in Oslo.
Also thanks to Ben Kirk and Louise Bilston who supported the team, in particular while Sampson and
Hadyn were busy marking.

Angelo Di Pasquale
IMO Team Leader

Mathematics Contests The Australian Scene 2022 | IMO Team Preparation School | 127
Mathematics Ashes

68 kykk AJP h`BMBM; 1tK


kykk Ji?2KiB+H b?2b la M/ lLE

aim`/v- CmHv N- kykk


ö ù
S`Q#H2K RX 6Q` 2p2`v BMi2;2` n ≥ 1 +QMbB/2` i?2 n ⇥ n i#H2 rBi? 2Mi`v n+1 ij
i i?2 BMi2`b2+iBQM
Q7 `Qr i M/ +QHmKM j- 7Q` 2p2`v i = 1, . . . , n M/ j = 1, . . . , nX .2i2`KBM2 HH BMi2;2`b n ≥ 1 7Q`
r?B+? i?2 bmK Q7 i?2 n2 2Mi`B2b BM i?2 i#H2 Bb 2[mH iQ 14 n2 (n − 1)X

S`Q#H2K kX G2i ABCD #2  [m/`BHi2`H BMb+`B#2/ BM  +B`+H2 ⌦X G2i i?2 iM;2Mi iQ ⌦ i D


BMi2`b2+i i?2 `vb BA M/ BC i TQBMib E M/ F - `2bT2+iBp2HvX  TQBMi T Bb +?Qb2M BMbB/2 i?2
i`BM;H2 ABC bQ i?i T EkCD M/ T F kADX G2i K 6= D #2  TQBMi QM i?2 b2;K2Mi DF bm+? i?i
T D = T KX S`Qp2 i?i i?2 HBM2b AC- DT M/ BK BMi2`b2+i i QM2 TQBMiX

S`Q#H2K jX *QMbB/2`  3m ⇥ 3m b[m`2 ;`B/- r?2`2 m Bb M BMi2;2` ;`2i2` i?M RX  7`Q; bBib QM
i?2 HQr2` H27i +Q`M2` +2HH S M/ rMib iQ ;2i iQ i?2 mTT2` `B;?i +Q`M2` +2HH F X h?2 7`Q; +M ?QT 7`QK
Mv +2HH iQ 2Bi?2` i?2 M2ti +2HH iQ i?2 `B;?i Q` i?2 M2ti +2HH mTr`/bX
aQK2 +2HHb +M #2 biB+Fv- M/ i?2 7`Q; ;2ib i`TT2/ QM+2 Bi ?QTb QM bm+?  +2HHX  b2i X Q7 +2HHb Bb
+HH2/ #HQ+FBM; B7 i?2 7`Q; +MMQi `2+? F 7`QK S r?2M HH i?2 +2HHb Q7 X `2 biB+FvX  #HQ+FBM; b2i
Bb KBMBKH B7 Bi /Q2b MQi +QMiBM  bKHH2` #HQ+FBM; b2iX

UV S`Qp2 i?i i?2`2 2tBbib  KBMBKH #HQ+FBM; b2i +QMiBMBM; i H2bi 3m2 − 3m +2HHbX
U#V S`Qp2 i?i 2p2`v KBMBKH #HQ+FBM; b2i +QMiBMb i KQbi 3m2 +2HHbX
LQi2X M 2tKTH2 Q7  KBMBKH #HQ+FBM; b2i 7Q` m = 2 Bb b?QrM #2HQrX *2HHb Q7 i?2 b2i X `2 K`F2/ #v H2ii2`b xX

KHKHKHKHKH F
x x KH
x
x
x
S x

GM;m;2, 1M;HBb? hBK2, 9 ?Qm`b M/ jy KBMmi2b


Zm2biBQMb Kmbi MQi #2 Tm#HBb?2/ 1+? T`Q#H2K Bb rQ`i? d TQBMib

Mathematics Contests The Australian Scene 2022 | Mathematics Ashes Paper | 128
Mathematics Ashes
Results

Australia P1 P2 P3 Total
William Cheah 7 2 0 9
Sizhe Pan 7 7 0 14
Zian Shang 7 7 1 15
Christopher Tran 6 7 1 14
Tony (Yuzheng) Wu 7 0 0 7
Grace Chang Yuan 6 7 0 13
Total 40 30 2 72

United Kingdom P1 P2 P3 Total


Alex Chui 7 7 2 16
Benjamin Gillott 6 7 2 15
Mohit Hulse 7 7 1 15
Isaac King 7 7 2 16
Sida Li 7 2 2 11
George Zhou 6 0 2 8
Total 40 30 11 81

Mathematics Contests The Australian Scene 2022 | Mathematics Ashes Results | 129
The 63rd International Mathematical Olympiad

The 63rd International Mathematical Olympiad (IMO) was held 6–16 July 2022 in the city of Oslo,
Norway. This was the first time that Norway has hosted the IMO.

A total of 589 high school students from 104 countries participated. Of these, 68 were female.

As per normal IMO rules, each participating country may enter a team of up to six students, a Team
Leader and a Deputy Team Leader.1

Participating countries also submit problem proposals for the IMO. This year there were 193 problem
proposals from 50 countries. The local Problem Selection Committee shortlisted 33 of these for
potential use on the IMO exams.

At the IMO the Team Leaders, as an international collective, form what is called the Jury. The Jury
makes the important decisions that shape each year’s IMO. Their first task is to set the two IMO
competition papers from the aforementioned shortlist and approve marking schemes. During this
period the Leaders and their observers are trusted to keep all information about the contest problems
completely confidential.

The six problems that ultimately appeared on the IMO exam papers may be described as follows.

1. An easy combinatorics problem proposed by France.


2. A functional inequality proposed by the Netherlands. Although intended to be a medium-level
problem, it turned out to be easier than the Jury anticipated.
3. A beautiful difficult number theory problem proposed by the United States of America.
4. An easy classical geometry problem proposed by Slovakia.
5. A medium to easy number theory problem proposed by Belgium.
6. A beautiful difficult combinatorics problem proposed by Serbia.

These six problems were posed in two exams held on Monday 11 July and Tuesday 12 July. Each exam
paper had three problems. The contestants worked individually. They were allowed four and a half hours
per paper to attempt the problems. Each problem was scored out of a maximum of seven points.

After the exams, the Leaders and their Deputies spent about two days assessing the work of the
students from their own countries, guided by the marking schemes which had been agreed to earlier.
A local team of markers called Coordinators also assessed the papers. They too were guided by the
marking schemes but are allowed some flexibility if, for example, a Leader brought something to their
attention in a contestant’s exam script that is not covered by the marking scheme. The Team Leader
and Coordinators must agree on scores for each student of the Leader’s country in order to finalise
scores. Any disagreements that cannot be resolved in this way are ultimately referred to the Jury. No
such referrals occurred this year.

1 The IMO regulations also permit countries to enter a small number of additional staff as Observers. These may fulfil various
roles such as meeting child safety obligations, assisting with marking and coordination, or learning about how to host an IMO.

Mathematics Contests The Australian Scene 2022 | The 63rd International Mathematical Olympiad | 130
The contestants found Problems 1 and 4 to be the easiest with average scores of 5.54 and 5.47,
respectively. Problem 6 was the hardest, averaging just 0.68. The score distributions by problem
number were as follows.

Mark P1 P2 P3 P4 P5 P6
0 34 104 407 74 112 434
1 56 89 68 31 115 80
2 19 23 69 18 44 22
3 21 41 6 17 48 7
4 10 7 3 10 15 19
5 13 3 4 5 35 3
6 51 19 4 1 49 2
7 385 303 28 433 171 22
Mean 5.54 4.31 0.81 5.47 3.52 0.68

The medal cuts were set at 34 points for Gold, 29 for Silver and 23 for Bronze. The medal distributions2
were as follows.

Gold Silver Bronze Total


Number 44 101 140 285
Proportion 7.5% 17.1% 23.8% 48.4%

These awards were presented at the closing ceremony. Of those who did not get a medal, a further 210
contestants received an Honourable Mention for scoring full marks on at least one problem.
Ten contestants achieved the most excellent feat of a perfect score of 42. This included all six team
members from the People’s Republic of China.3
Congratulations to the Australian IMO team on their performance this year! The team finished 29th in
the rankings4, winning one Gold medal5, four Bronze medals and one Honourable Mention.
The Gold medallist was:
• Tony (Yuzheng) Wu, year 12, Killara High School, NSW

The Bronze medallists were:


• William Cheah, year 8, Scotch College, VIC
• Sizhe Pan, year 11, James Ruse Agricultural High School, NSW
• Zian Shang, year 11, Scotch College, VIC
• Christopher Tran, year 11, The University High School, VIC

An Honourable Mention was awarded to


• Grace Chang Yuan, year 12, Methodist Ladies’ College, VIC for her complete solutions to problem
2 and 4. Grace was also one of five girls at the IMO who was a recipient of the Mirzakhani Award6,
being the outstanding female IMO contestant from Oceania.

2 The total number of medals is approved by the Jury and should not normally exceed half the total number of contestants.
The numbers of Gold, Silver and Bronze medals should be approximately in the ratio 1:2:3.
3 It is exceedingly difficult for an entire team of six to achieve a perfect score. The only other time this has occurred was when
the United States of America achieved this at the 1994 IMO.
4 The ranking of countries is not officially part of the IMO general regulations. However, countries are ranked each year on the
IMO’s official website according to the sum of the individual student scores from each country.
5 This is now the fifth year in a row that the Australian IMO team has won at least one Gold medal.
6 Named for Fields Medal winner Maryam Mirzakhani, these awards aim to encourage girls in mathematics.

Mathematics Contests The Australian Scene 2022 | The 63rd International Mathematical Olympiad | 131
The following table shows the Australian team’s scores in detail.

Name P1 P2 P3 P4 P5 P6 Total Award


William Cheah 7 7 0 7 7 0 28 Bronze medal
Sizhe Pan 7 7 1 7 5 0 27 Bronze medal
Zian Shang 7 7 0 7 7 0 28 Bronze medal
Christopher Tran 7 7 0 7 6 0 27 Bronze medal
Tony (Yuzheng) Wu 7 7 2 7 7 4 34 Gold medal
Grace Yuan 2 7 0 7 2 0 18 Honourable mention
Total 37 42 3 42 34 4 162 29th/104
Australian average 6.17 7.00 0.50 7.00 5.67 0.67 27.00
IMO average 5.54 4.31 0.81 5.47 3.52 0.68 20.32

Four members of the 2022 Australian IMO team are eligible for the 2023 IMO team.

The 2022 IMO was organised by Norway’s Ministry of Education and The University of Oslo. Hosts for
future IMOs have been secured as follows.
2–13 July, 2023 Chiba, Japan
2024 United Kingdom
2025 Melbourne, Australia
Much of the statistical information found in this report can also be found on the official website of the
IMO.
www.imo-official.org

Australian IMO team at opening ceremony. Left to right: Grace Chang Yuan, William Cheah, Christopher
Tran, Tony Wu, Sizhe Pan and Zian Shang.

Mathematics Contests The Australian Scene 2022 | The 63rd International Mathematical Olympiad | 132
Grace Chang Yuan receiving her Mirzakhani Tony Wu having just received his Gold medal at
Award at the closing ceremony. the closing ceremony.

Angelo Di Pasquale
IMO Team Leader, Australia

Mathematics Contests The Australian Scene 2022 | The 63nd International Mathematical Olympiad | 133
Some IMO Country Totals
Rank Country Total
1 People’s Republic of China 252
2 Republic of Korea 208
3 United States of America 207
4 Vietnam 196
5 Romania 194
6 Thailand 193
7 Germany 192
8 Islamic Republic of Iran 191
8 Japan 191
10 Israel 188
10 Italy 188
12 Poland 183
13 United Kingdom 179
14 Canada 178
14 Taiwan 178
16 Bulgaria 177
17 Kazakhstan 174
17 Ukraine 174
19 Brazil 173
19 Hong Kong 173
19 Peru 173
22 Saudi Arabia 168
23 Mexico 167
24 India 165
24 Singapore 165
26 Armenia 163
26 Greece 163
26 Turkey 163
29 Australia 162
29 Mongolia 162

Mathematics Contests The Australian Scene 2022 | The 63nd International Mathematical Olympiad | 134
Distribution of Awards at the 2022 IMO
Country Total Gold Silver Bronze HM
Albania 62 0 0 0 3
Algeria 75 0 0 1 3
Argentina 139 0 1 3 2
Armenia 163 0 2 4 0
Australia 162 1 0 4 1
Austria 118 0 0 2 4
Azerbaijan 157 0 1 4 1
Bangladesh 115 0 0 1 5
Belarus 160 0 2 3 1
Belgium 94 0 0 1 5
Bolivia 57 0 0 0 4
Bosnia and Herzegovina 154 0 3 2 1
Botswana 17 0 0 0 0
Brazil 173 2 1 2 1
Bulgaria 177 1 3 1 1
Canada 178 2 2 1 1
Chile 27 0 0 0 1
Colombia 74 0 0 1 2
Costa Rica 92 0 0 1 4
Croatia 157 0 1 4 1
Cuba 13 0 0 0 1
Cyprus 85 0 0 0 5
Czech Republic 122 0 1 2 3
Denmark 121 0 0 2 4
Dominican Republic 26 0 0 0 2
Ecuador 27 0 0 0 2
El Salvador 56 0 1 0 2
Estonia 124 0 1 2 3
Finland 101 0 0 1 4
France 158 0 3 2 1
Georgia 121 0 0 3 3
Germany 192 1 4 1 0
Ghana 18 0 0 0 0
Greece 163 0 2 3 1
Honduras 31 0 0 0 2
Hong Kong 173 1 3 1 1
Hungary 158 0 1 5 0
Iceland 51 0 0 0 3
India 165 1 0 5 0
Indonesia 151 0 1 4 1
Ireland 84 0 0 0 5
Islamic Republic of Iran 191 3 3 0 0

Mathematics Contests The Australian Scene 2022 | The 63nd International Mathematical Olympiad | 135
Country Total Gold Silver Bronze HM
Israel 188 1 4 1 0
Italy 188 2 2 2 0
Japan 191 1 4 1 0
Kazakhstan 174 0 3 3 0
Kosovo 55 0 0 0 4
Kyrgyzstan 73 0 0 0 4
Latvia 94 0 0 0 5
Liechtenstein 12 0 0 0 1
Lithuania 129 0 1 1 4
Luxembourg 26 0 0 0 1
Macau 127 0 0 3 3
Malaysia 137 1 1 1 3
Mauritania 42 0 0 0 4
Mexico 167 0 2 4 0
Mongolia 162 0 2 3 1
Montenegro 23 0 0 0 1
Morocco 82 0 0 0 4
Nepal 27 0 0 0 2
Netherlands 145 1 0 2 3
New Zealand 108 0 1 0 5
North Macedonia 148 0 2 2 2
Norway 103 0 0 1 4
Oman 19 0 0 0 0
Pakistan 54 0 0 0 3
Palestine 14 0 0 0 1
Panama 42 0 0 0 3
Paraguay 59 0 0 0 4
People’s Republic of China 252 6 0 0 0
Peru 173 0 3 3 0
Philippines 119 0 0 2 4
Poland 183 0 4 2 0
Portugal 137 0 1 3 2
Puerto Rico 40 0 0 0 1
Republic of Korea 208 3 3 0 0
Republic of Moldova 135 0 1 2 3
Romania 194 2 4 0 0
Rwanda 34 0 0 0 1
Saudi Arabia 168 0 2 4 0
Serbia 153 0 2 2 2
Singapore 165 0 4 1 1
Slovakia 136 0 1 4 1
Slovenia 79 0 0 1 3
South Africa 94 0 0 0 6
Spain 139 0 0 4 2

Mathematics Contests The Australian Scene 2022 | The 63nd International Mathematical Olympiad | 136
Country Total Gold Silver Bronze HM
SriLanka 77 0 0 0 5
Sweden 106 0 0 1 5
Switzerland 145 0 0 4 2
Syria 82 0 0 1 5
Taiwan 178 1 2 3 0
Tajikistan 107 0 0 1 5
Thailand 193 3 2 1 0
Tunisia 85 0 0 0 6
Turkey 163 0 4 1 1
Uganda 13 0 0 0 1
Ukraine 174 1 1 4 0
United Arab Emirates 10 0 0 0 1
United Kingdom 179 1 3 2 0
United States of America 207 4 1 1 0
Uruguay 27 0 0 0 2
Uzbekistan 124 0 0 1 5
Venezuela 7 0 0 0 0
Vietnam 196 2 2 2 0
Total (104 teams, 589 contestants) 44 101 140 210
N.B. Not all countries entered a full team of six students.
Angelo Di Pasquale
IMO Team Leader, Australia

Mathematics Contests The Australian Scene 2022 | The 63nd International Mathematical Olympiad | 137
Paper

Mathematics Contests The Australian Scene 2022 | 63nd International Mathematical Olympiad Paper | 138
Mathematics Contests The Australian Scene 2022 | 63nd International Mathematical Olympiad Paper | 139
International Mathematical Olympiad
Solutions

1. Solution (Based on the solution by Sizhe Pan, year 11, James Ruse Agricultural High
School, NSW. Sizhe was a Bronze medallist with the 2022 Australian IMO team.)
3n+1
Answer All (n, k) with n ≤ k ≤ 2
Step 1 Each k < n fails
This is because the following initial sequence is invariant under the operation.

AA . . . A BA BB . . . B
n−1 n−1

Step 2 Each k > 3n+1


2
fails
n n
Let x = 2 and y = 2 . Consider the following initial sequence.

AA . . . A BB . . . B AA . . . A BB . . . B
x x y y

The k th coin from the left is the k ′ th coin from the right where
  k + k ′ = 2n + 1. Since
k > 3n+1
2
, we have k ′ < n+1
2
. Therefore k ′ ≤ n2 , and so k ′ ≤ n2 = y. This implies that
the process results in the following 4-cycle.

AA . . . A BB . . . B AA . . . A BB . . . B −→ BB . . . B AA . . . A BB . . . B AA . . . A
x x y y y x x y

−→ AA . . . A BB . . . B AA . . . A BB . . . B
y y x x

−→ BB . . . B AA . . . A BB . . . B AA . . . A
x y y x

−→ AA . . . A BB . . . B AA . . . A BB . . . B
x x y y

3n+1
Step 3 Each k with n ≤ k ≤ 2
works
Define a block to be any maximal chain. Note that the number, b say, of blocks either
stays the same or decreases. So eventually we reach a situation where b no longer changes.
If at any subsequent stage, the leftmost block is moved, then since k ≥ n this block
contains the leftmost n coins, and we are done.
If at any subsequent stage, a block is moved from somewhere in the middle, then the
blocks either side of it coalesce into a single block, which reduces b. Contradiction.
Thus from here on we may assume that it is only the rightmost block that is moved.
If b = 3, then without loss of generality the situation looks like

AA . . . A BB . . . B AA . . . A
r n s

where r + s = n. Moving the rightmost block combines all the A’s into a single block and
so reduces b. Contradiction.
If b ≥ 4, let a be the length of the shortest block of A’s. There are at least 4 blocks, so at
least two of these must be A’s. The total number of A’s is n, and so a ≤ n2 . Since only the
rightmost block is moved, we will reach a stage where the rightmost block consists of the
a A’s. This implies that k ≥ 2n + 1 − a ≥ 2n + 1 − n2 = 3n+2 2
. This contradicts k ≤ 3n+12
.
Mathematics Contests The Australian Scene 2022 | 63nd International Mathematical Olympiad Solutions | 140
Thus we are left with b = 2. In this case we are done because each of the two blocks will
2. Solution 1 (Grace Chang Yuan, year 12, Methodist Ladies’ College, VIC. Grace was
awarded an Honourable Mention with the 2022 Australian IMO team. She was also one
of five girls at the 2022 IMO who were recipients of the Mirzakhani Award.)
Answer f (x) = x1 .
Recall that we seek all functions f : R+ → R+ such that for each x ∈ R+ , there is exactly
one y ∈ R+ satisfying
xf (y) + yf (x) ≤ 2. (1)

For each x > 0, let d(x) denote the unique y satisfying (1).
Claim 1 d(x) = y if and only if d(y) = x
Proof This follows immediately from (1). 
Claim 2 d(x) = x for all x > 0
Proof Suppose, for the sake of contradiction, that d(x) = y and d(y) = x for some
x = y. Therefore we have the following inequalities.

xf (y) + yf (x) ≤ 2 (since d(x) = y) (2)


xf (x) + xf (x) > 2 (since d(x) = x) (3)
 y)
yf (y) + yf (y) > 2 (since d(y) = (4)
1
Inequalities (3) and (4) yield f (x) > x
and f (y) ≥ y1 , respectively. It follows that

x y x y
xf (y) + yf (x) ≥ + > 2 · = 2.
y x y x

Here we have used the AM–GM inequality which is strict because x = y implies x
y
= xy .
But xf (y) + yf (x) > 2 contradicts (2). 
1
Claim 3 f (x) = x
for all x > 0
Proof From claim 2, we have d(x) = x. Putting this into (1) yields xf (x) + xf (x) ≤ 2
for all x. So we have
1
f (x) ≤ for all x > 0 (5)
x
and
xf (y) + yf (x) > 2 for all y = x. (6)

Suppose, for the sake of contradiction, that f (x) < x1 for some x > 0. Thus f (x) = 1
y
for
some y > x. Note also from (5) that f (y) ≤ y1 . It follows that
x
xf (y) + yf (x) = xf (y) + 1 ≤ +1<2
y

since x < y. This contradicts (6). 


1
Finally, it remains to verify that f (x) = x
is indeed a solution.
1 x y
Putting f (x) = x
into (1) yields y
+ x
≤ 2. However, by the AM–GM inequality, we have

x y x y
y
+ x
≥ 2 y
· x
= 2, with equality if and only if y = x. Thus (1) fails if y = x, and
succeeds if y = x. This shows the existence and uniqueness of y, as required. 

Mathematics Contests The Australian Scene 2022 | 63nd International Mathematical Olympiad Solutions | 141
Solution 2 (This alternative way to finish solution 1 is based on the solution by Zian
Shang, year 11, Scotch College, VIC. Zian was a Bronze medallist with the 2022 Australian
IMO team.)
We establish (5) and (6) in the same way as solution 1.
Hence for any y = x, we have

2 < xf (y) + yf (x) (from (6))


x
≤ + yf (x) (from (5))
y
2y − x
⇔ f (x) > . (7)
y2

For each fixed x, we see that RHS(7) is a continuous function of y. So although y = x is


not permitted in (7), we may take the limit as y approaches x in (7) to find
2x − x 1
f (x) ≥ = .
x2 x
1
But f (x) ≤ x
from (5). Hence f (x) = x1 .
The verification that f (x) = 1
x
is indeed a solution is as per solution 1. 

Mathematics Contests The Australian Scene 2022 | 63nd International Mathematical Olympiad Solutions | 142
3. Solution (Problem Selection Committee)
Let us allow the value x = 0 as well. We prove the same statement under this more general
constraint. This implies the statement with the original conditions.
Call a pair {p, q} of primes with p = q special if pq = x2 + x + k for some non-negative
integer x. The following two-part claim forms the key mechanism of the solution:

(a) For every prime r, there are at most two primes less than r forming a special pair
with r.
(b) If two such primes p and q exist, then {p, q} is itself special.

Proof of Claim
We are interested in integers 0 ≤ x < r satisfying

x2 + x + k ≡ 0 (mod r). (1)

Since there are at most two residues modulo r that can satisfy that quadratic congruence,
there are at most two possible values of x. That proves (a).
For part (b), suppose there are primes p, q with p < q < r and non-negative integers y, z
such that
y 2 + y + k = pr (2)
2
z + z + k = qr. (3)

From p < q < r, we can see that 0 ≤ y < z < r, so that y, z are the two solutions of (1)
modulo r. By Vieta’s formulas we have y + z ≡ −1 (mod r), and so

y + z = r − 1. (4)

Subtracting (3) from (2) yields

(y − z)(y + z + 1) = r(p − q)
⇔ y−z =p−q (using (4)) (5)

Adding (4) and (5) yields


y = 12 (r + p − q − 1).
Putting this into (2) yields

k = pr − y 2 − y
= 14 (4pr − (r + p − q − 1)2 − 2(r + p − q − 1))
= 14 (2pq + 2pr + 2qr − p2 − q 2 − r2 + 1)

which is symmetric in p, q, r.
Equation (2), now takes the form
 2
pr − 12 (r + p − q − 1) − 12 (r + p − q − 1) = 14 (2pq + 2pr + 2qr − p2 − q 2 − r2 + 1)

which is a polynomial identity in p, q, r. Interchanging r with q in the above yields


 2
pq − 12 (q + p − r − 1) − 12 (q + p − r − 1) = 14 (2pq + 2pr + 2qr − p2 − q 2 − r2 + 1).

Therefore pq − w2 − w = k, where w = 12 (q + p − r − 1). That is, w2 + w + k = pq, and


so {p, q} is special. This concludes the proof of the claim. 

Mathematics Contests The Australian Scene 2022 | 63nd International Mathematical Olympiad Solutions | 143
Now we settle the problem by induction on |S|.
The base cases with |S| ≤ 3 are clearly true because all configurations are related by
rotation and or reflection.
Suppose we have proven it for |S| = n and consider |S| = n + 1. Let r be the largest
prime in S. The claim tells us that in any valid circle of primes:

• the neighbours of r are uniquely determined, and


• removing r from the circle results in a smaller valid circle.

It follows that there is at most one valid circle (up to rotation and reflection). This
completes the inductive step, and the proof. 
Comment (Problem Selection Committee)
The statement is not as inapplicable as it might seem. For example, for k = 41, the
following 385 primes form a valid cycle of primes!
53, 4357, 104173, 65921, 36383, 99527, 193789, 2089123, 1010357, 2465263, 319169, 15559, 3448, 2647, 1951,
152297, 542189, 119773, 91151, 66431, 222137, 1336799, 469069, 45613, 1047941, 656291, 355867, 146669,
874879, 2213327, 305119, 3336209, 1623467, 520963, 794201, 1124833, 28697, 15683, 42557, 6571, 39607,
1238833, 835421, 2653681, 5494387, 9357539, 511223, 1515317, 8868173, 114079681, 59334071, 22324807,
3051889, 5120939, 7722467, 266239, 693809, 3931783, 1322317, 100469, 13913, 74419, 23977, 1361, 62983,
935021, 512657, 1394849, 216259, 45827, 31393, 100787, 1193989, 600979, 209543, 357661, 545141, 19681,
10691, 28867, 165089, 2118023, 6271891, 12626693, 21182429, 1100467, 413089, 772867, 1244423, 1827757,
55889, 1558873, 5110711, 1024427, 601759, 290869, 91757, 951109, 452033, 136471, 190031, 4423, 9239,
15809, 24133, 115811, 275911, 34211, 877, 6653, 88001, 46261, 317741, 121523, 232439, 379009, 17827, 2699,
15937, 497729, 335539, 205223, 106781, 1394413, 4140947, 8346383, 43984757, 14010721, 21133961, 729451,
4997297, 1908223, 278051, 529747, 40213, 768107, 456821, 1325351, 225961, 1501921, 562763, 75527, 5519,
9337, 14153, 499, 1399, 2753, 14401, 94583, 245107, 35171, 397093, 195907, 2505623, 34680911, 18542791,
7415917, 144797293, 455529251, 86675291, 252704911, 43385123, 109207907, 204884269, 330414209, 14926789,
1300289, 486769, 2723989, 907757, 1458871, 65063, 4561, 124427, 81343, 252887, 2980139, 1496779, 3779057,
519193, 47381, 135283, 268267, 446333, 669481, 22541, 54167, 99439, 158357, 6823, 32497, 1390709, 998029,
670343, 5180017, 13936673, 2123491, 4391941, 407651, 209953, 77249, 867653, 427117, 141079, 9539, 227, 1439,
18679, 9749, 25453, 3697, 42139, 122327, 712303, 244261, 20873, 52051, 589997, 4310569, 1711069, 291563,
3731527, 11045429, 129098443, 64620427, 162661963, 22233269, 37295047, 1936969, 5033449, 725537, 1353973,
6964457, 2176871, 97231, 7001, 11351, 55673, 16747, 169003, 1218571, 479957, 2779783, 949609, 4975787,
1577959, 2365007, 3310753, 79349, 23189, 107209, 688907, 252583, 30677, 523, 941, 25981, 205103, 85087,
1011233, 509659, 178259, 950479, 6262847, 2333693, 305497, 3199319, 9148267, 1527563, 466801, 17033, 9967,
323003, 4724099, 14278309, 2576557, 1075021, 6462593, 2266021, 63922471, 209814503, 42117791, 131659867,
270892249, 24845153, 12104557, 3896003, 219491, 135913, 406397, 72269, 191689, 2197697, 1091273, 2727311,
368227, 1911661, 601883, 892657, 28559, 4783, 60497, 31259, 80909, 457697, 153733, 11587, 1481, 26161, 15193,
7187, 2143, 21517, 10079, 207643, 1604381, 657661, 126227, 372313, 2176331, 748337, 64969, 844867, 2507291,
29317943, 14677801, 36952793, 69332267, 111816223, 5052241, 8479717, 441263, 3020431, 1152751, 13179611,
38280013, 6536771, 16319657, 91442699, 30501409, 49082027, 72061511, 2199433, 167597, 317963, 23869,
2927, 3833, 17327, 110879, 285517, 40543, 4861, 21683, 50527, 565319, 277829, 687917, 3846023, 25542677,
174261149, 66370753, 9565711, 1280791, 91393, 6011, 7283, 31859, 8677, 10193, 43987, 11831, 13591, 127843,
358229, 58067, 15473, 65839, 17477, 74099, 19603, 82847, 21851, 61.

We can find smaller valid circles from the above by deleting the largest primes one by
one. For example, if we delete all but the five smallest primes from the above, we obtain
the valid circle 53, 499, 227, 523, 61.

Mathematics Contests The Australian Scene 2022 | 63nd International Mathematical Olympiad Solutions | 144
4. Solution 1 (William Cheah, year 8, Scotch College, VIC. William was a Bronze medallist
with the 2022 Australian IMO team.)
Our proof uses directed angles throughout.
Let X = AB ∩ DT and Y = AE ∩ CT .
Q

S
A

X Y
E
B
T

P C D R

The data in the problem statement immediately implies T BC ≡ T DE (SSS). There-


fore ∠BT C = ∠DT E, from which it follows that

∠QT B = ∠ET S. (1)

We also have
∠T BQ = ∠T BA = ∠AET = ∠SET. (2)
From (1) and (2), it follows that T BQ ∼ T ES (AA). Therefore T B/T Q = T E/T S,
which implies
T B · T S = T E · T Q.
Since T B = T D and T E = T C, it follows that

TD · TS = TC · TQ

and so CDQS is cyclic.


Another consequence of T BQ ∼ T ES is that

∠XQY = ∠BQT = ∠T SE = ∠XSY

and so XY QS is cyclic.
From cyclic CDQS and cyclic XY QS, we deduce

∠XDP = ∠SDC = ∠SQC = ∠SQY = ∠DXY

and so XY  P R.
Using XY  P R and cyclic XY QS, we deduce

∠RP Q = ∠Y XQ = ∠Y SQ = ∠RSQ

and so P SQR is cyclic, as desired. 

Mathematics Contests The Australian Scene 2022 | 63nd International Mathematical Olympiad Solutions | 145
Solution 2 (Based on the solution by Christopher Tran, year 11, The University High
School, VIC. Christopher was a Bronze medallist with the 2022 Australian IMO team.)
This is an alternative way to finish solution 1 once it is established that XY QS is cyclic
and XY  P R.
From XY  P R, we have
AP AR
= . (3)
AX AY
From XY QS cyclic, we have
AX · AQ = AY · AS. (4)

Multiplying (3) and (4) yields

AP · AQ = AR · AS

and so P SQR is cyclic, as desired. 

Mathematics Contests The Australian Scene 2022 | 63nd International Mathematical Olympiad Solutions | 146
Solution 3 (Based on the solution by Sizhe Pan, year 11, James Ruse Agricultural High
School, NSW. Sizhe was a Bronze medallist with the 2022 Australian IMO team.)
This is an alternative way to establish that XY  P R.
We prove that XY QS is cyclic as in solution 1. It follows that

∠BXT = ∠QXS = ∠QY S = ∠T Y E

and so T XB ∼ T Y E (AA).
From this we have
TX TB TD
= =
TY TE TC
and so T XY ∼ T DC (PAP). It follows that

∠DXY = ∠T XY = ∠T DC = ∠XDC

and so XY  CD, which is the same as XY  P R, as desired. 

Mathematics Contests The Australian Scene 2022 | 63nd International Mathematical Olympiad Solutions | 147
Solution 4 (Based on the solution by Zian Shang, year 11, Scotch College, VIC. Zian
was a Bronze medallist with the 2022 Australian IMO team.)
This is an alternative way to finish solution 1 that bypasses proving XY  P R.
We establish that XY QS and CDQS are cyclic as in solution 1. Using these, we have

∠SRP = ∠SDP − ∠DSR


= ∠SDC − ∠XSY
= ∠SQC − ∠XQY (XY QS and CDQS cyclic)
= ∠SQP

and so P SQR is cyclic, as desired. 

Mathematics Contests The Australian Scene 2022 | 63nd International Mathematical Olympiad Solutions | 148
5. Solution 1 (Based on the solution by William Cheah, year 8, Scotch College, VIC.
William was a Bronze medallist with the 2022 Australian IMO team.)
Answer (a, b, p) = (2, 2, 2) and (3, 4, 3).
It is easy to verify that the above are solutions. We prove that there are no other solutions.
For reference we are looking for solutions to

ap = b! + p. (1)

Case 1 b < p
If a ≤ b, then a | b!. Also a | ap , and so from (1) we have a | p. Since p is prime and
a ≤ b < p, it follows that a = 1. But 1 = b! + p has no valid solutions. Contradiction.
If a ≥ b + 1, then
b! = ap − p
≥ (b + 1)p − p
p  
 p i
= b −p
i=0
i
≥ bp + pb + 1 − p
> bp
≥ bb
≥ b!

which is also contradiction.


Therefore case 1 yields no solutions.
Case 2 b ≥ 2p
Then p | b!, and so p | ap , which implies p | a. Since p ≥ 2, this implies p2 | ap . But since
p2 | p × 2p | b!, it follows that p2 | ap − b! = p. Contradiction.
Case 3 p ≤ b < 2p
This implies
ap ≤ (2p − 1)! + p
p−1

=p i(2p − i) + p
i=1
p−1  2
 i + 2p − i
<p + p (AM–GM)
i=1
2
2p−2
= p(p + 1)
< p2p

and so a < p2 .
But p | b! and so p | ap , which implies p | a. It follows that a = pc for some positive
integer c < p. But then c < p ≤ b. Therefore c | b!, and so c | p. Since c < p, this implies
c = 1, and so a = p.
So we have reduced the problem to finding all solutions to

pp = b! + p

where p ≤ b < 2p.

Mathematics Contests The Australian Scene 2022 | 63nd International Mathematical Olympiad Solutions | 149
Although this is still part of case 3, it is convenient to designate it as a separate case as
follows.
Case 4 a = p
We wish to find all solutions to
b! = pp − p (2)
where p ≤ b < 2p.
If p = 2, then equation (2) implies b = 2.
If p = 3, equation (2) implies b = 4.
We shall show that there are no solutions for p ≥ 5 by showing that equation (2) fails
modulo (p + 1)2 .
For p ≥ 5, we have

b! = pp − p > pp /2 > pp−1 > p × (p − 1) × · · · × 2 = p!.

Therefore b > p, and so b ≥ p + 1.


Observe that p ≥ 5 implies that 2, p+1 2
and p + 1 are distinct positive integers. Since
p + 1 ≤ b, we see that b! is divisible by 2 · p+1
2
· (p + 1) = (p + 1)2 . It follows from (2) that

pp ≡ p (mod (p + 1)2 ). (3)

However, using the binomial theorem, we have

pp = (p + 1 − 1)p
p  
 p
= (p + 1)i (−1)p−i
i=0
i
≡ −1 + p(p + 1) (mod (p + 1)2 )
≡ −p − 2 (mod (p + 1)2 ). (4)

Comparing (3) with (4) yields p ≡ −p − 2 (mod (p + 1)2 ), and so (p + 1)2 | 2p + 2. But
(p + 1)2 = p2 + 2p + 1 > 2p + 2, so this is impossible. 

Mathematics Contests The Australian Scene 2022 | 63nd International Mathematical Olympiad Solutions | 150
Solution 2 (This is an alternative way to deal with case 4 from solution 1. It is based on
the solution by Tony Wu, year 12, Killara High School, NSW. Tony was a Gold medallist
with the 2022 Australian IMO team.)
Recall, we wish to find all solutions to

b! = pp − p (2)

where p ≤ b < 2p.


The case b = p is dealt with as in solution 1. Thus we may assume that b ≥ p + 1.
The cases p = 2 and p = 3 are also dealt with as in solution 1.
We directly check that the case p = 5 yields b! = 3120, which has no solution for b.
Let ν2 denote the 2-adic valuation. That is, for each positive integer n, ν2 (n) denotes the
highest power of 2 that divides n. Recall that for odd integers x, and any positive integer
m, the lifting the exponent lemma1 yields

ν2 (xm − 1) = ν2 (x − 1) + ν2 (x + 1) + ν2 (m) − 1.

Applying this to b! = p(pp−1 − 1) yields

ν2 (b!) = ν2 (p(pp−1 − 1))


= ν2 (pp−1 − 1)
= ν2 (p − 1) + ν2 (p + 1) + ν2 (p − 1) − 1
 
p−1
= ν2 (p + 1) + ν2 (p − 1) + ν2
2
  
p−1
= ν2 (p + 1)(p − 1) (3)
2
≤ ν2 ((p + 1)!) (4)
≤ ν2 (b!).

Thus equality holds throughout. However in going from (3) to (4), the inequality is strict
whenever p−1
2
> 2. This implies that p ≤ 5, which has already been dealt with. 

1
See en.wikipedia.org/wiki/Lifting-the-exponent_lemma.

Mathematics Contests The Australian Scene 2022 | 63nd International Mathematical Olympiad Solutions | 151
6. Solution (Based on the presentation by Tony Wu, year 12, Killara High School, NSW.
Tony was a Gold medallist with the 2022 Australian IMO team.)
Answer 2n2 − 2n + 1
Let A denote the Nordic square. For each m, let Am denote the cell in A that contains
the number m.
Imagine placing the numbers from 1 to n2 , one by one, going from smallest to largest in
the Nordic square and labelling the numbers as we go inductively as follows.
• Each valley is labelled with 1.
• Each cell that is not a valley is labelled with the sum of the labels for the cells
adjacent to it so far.
For clarity, create a second n × n board B where we write the labels for the cells in A in
the corresponding positions. Also, when we write a label in a cell X of B, let us mark the
edges of X which are adjacent to cells already containing labels. Here is an example that
illustrates the process partway through.

A B
13 3 3 1
11 4 10 6 2 1 2 1
5 2 12 9 1 1 5 1
7 1 8 2 1 1

The label given to Am is equal to the number of uphill paths ending at Am . This is because
if Am is not a valley, then the uphill paths ending at Am are precisely the paths formed
by appending Am to uphill paths that end at the cells adjacent to Am which contain
numbers smaller than m. Therefore the total number of uphill paths is equal to the sum
of the numbers in B. It is this sum that we wish to minimise.
The numbers are put into B, one by one, according to the following rules.
B1. If the neighbours of a cell X are currently empty, we may write a 1 in X. (This
corresponds to a valley in A.)
B2. If a cell X has at least one nonempty neighbour, we may write in X the sum of the
numbers in its nonempty neighbours. We also mark the edges between X and its
nonempty neighbours.
Note that the newly written number in X is greater than or equal to the number of newly
marked edges on that turn. This inequality is strict if X corresponds to a valley or if X
is adjacent to any cells containing numbers greater than 1.
Every edge between neighbouring cells is eventually marked. There are 2n(n − 1) such
edges. And since there is at least one valley, it follows that the sum of the numbers in B
is greater than or equal to 2n2 − 2n + 1.
The bound 2n2 − 2n + 1 is sharp if and only if rule B1 is used only once, and no two
numbers in B greater than 1 are in adjacent cells.
It follows that the cells containing 1s form a tree2 , and that the complement of this tree
in B consists of isolated cells.
2
The graph of such cells is connected because rule B1 is used only once. Moreover it contains no cycles because
if a cycle were created, then according to rule B2 the cell creating the cycle would have a number greater than
or equal to 2 written in it, and so would not be part of the graph after all.

Mathematics Contests The Australian Scene 2022 | 63nd International Mathematical Olympiad Solutions | 152
Conversely, any tree containing k cells whose complement consists of isolated cells arises
from a Nordic square A. This can be seen by placing the numbers 1, 2, . . . , n2 one by one,
going from from smallest to largest, in the board as follows.
C1. Put 1 in any cell of the tree.
C2. For m = 2, 3, . . . , k, put m in any cell of the tree that is adjacent to a cell that
already contains a number.
C3. For m > k, put m in any isolated cell.
To finish the proof, we show that 2n2 − 2n + 1 is attainable by showing how to construct
a tree of cells on an n × n board whose complement consists of isolated cells.
Consider the following infinite repeating pattern.

.. .. .. .. ..
. . . . .

···

···

···

···

···
P Q

Then adjust the bottom two rows to connect everything up yielding an infinite blue tree.

.. .. .. .. ..
. . . . .

···

···

···

···

···
P Q

For n ≡ 1 (mod 3), position A so that it’s bottom-left corner is at P .


For n ≡ 1 (mod 3), position A so that it’s bottom-left corner is at Q.
Finally, number the the cells of the board according to rules C1-C3. 

Mathematics Contests The Australian Scene 2022 | 63nd International Mathematical Olympiad Solutions | 153
AMO.
2022
AUSTRALIAN
DATE
MATHEMATICAL
AUSTRALIAN OLYMPIAD OLYMPIAD
MATHEMATICAL
2022 AMOC Senior Contest Tuesday 16
AMOC SENIOR
2022CONTEST
AMOC SENIOR August 2022
CONTEST

2022
DAY 1
Wednesday, 2 February 2022
Tuesday, 16 August 2022
Time allowed: 4 hours
No calculators are to be used. Time allowed: 4 hours
Each question is worth seven points. No calculators are to be used.
Each question is worth seven points.

1. Prove
1. Find that aofconvex
all triples pentagon
positive with
integers (a, integer sidethat
b, n) such lengths and an odd perimeter can
have two right angles, but cannot have more than two right angles.
a 2
+ b =angles
(A polygon is convex if all of its2 interior are less than 180◦ .)
n! + 14.

(Note =1×
that n! that
2. Suppose P 2is×a 3point
× · · ·inside
× (n a−convex
1) × n.)
hexagon ABCDEF such that P ABC,
P CDE and P EF A are parallelograms of equal area.
1 , x2 , .that
2. Let xProve . . , xPnBCD,
be non-negative
P DEF and Preal numbers,
F AB are also where n ≥ 2. of equal area.
parallelograms
Prove that
3. Determine all polynomials p(x) with real coefficients such that:
(x1 + x2 + · · · + xn )2
x1 >
ˆ p(x) x20+for
x2all
x3 positive + · ·numbers
+ x3 x4 real · + xn−1xxn ≤ .
4
1 1 1
ˆ + + = 1 for all positive real numbers x, y, z satisfying xyz = 1.
(Note thatp(x)
each term the left-hand side is of the form xi xi+1 for i = 1, 2, . . . , n − 1.)
p(y) on p(z)

Let S be the set of points (i, j) in the plane with i, j ∈ {1, 2, . . . , 2022} and i < j.
3. Let4. ABC be a triangle with incentre I. Let K be the circle that passes through I
We would like to colour each point in S either red or blue, so that whenever points
and is tangent to the circumcircle of ABC at A. Suppose that K intersects AB and
(i, j) and (j, k) have the same colour, the point (i, k) also has that colour.
AC again at P and Q, respectively.
How many such colourings of S are there?
Prove that the angle bisectors of ∠BP Q and ∠P QC intersect on BC.

4. For a real number x, let ⌈x⌉ be the smallest integer greater than or equal to x.
Find all nonempty sets S of positive integers such that whenever a and b (not
 
necessarily distinct) are in S, then both ab and ab are in S.

5. Hugo and Maryna are playing a game with n buckets and an infinite pile of stones,
where n is a positive integer. Initially, all buckets are empty.
Hugo and Maryna alternate their turns, with Hugo going first. During Hugo’s turn,
he picks up two stones from the pile and either puts them into one bucket of his
choice or into two separate buckets of his choice. During Maryna’s turn, she chooses
one of the n buckets and empties it back onto the pile.
Hugo wins if, at any point, one of the buckets contains at least 50 stones.
For which values of n does Hugo have a winning strategy?

The Olympiad program is supported by the Australian Government


Department of Industry, Science, Energy and Resources through the Science
Competitions: Mathematics and Informatics Olympiads grant opportunity.

The Olympiad program is supported© 2022


by theAustralian
AustralianMathematics
GovernmentTrust
Department of Industry, Science, Energy
and Resources through the Science Competitions: Mathematics and Informatics Olympiads grant opportunity.

© 2022 Australian Mathematics Trust


Mathematics Contests The Australian Scene 2022 | AMOC Senior Contest Paper | 154
2022 AMOCSENIOR
AMOC Senior Contest
CONTEST 2022
Solutions
Solutions
© 2022 Australian Mathematics Trust
1. Find all triples of positive integers (a, b, n) such that

2a + b2 = n! + 14.

(Note that n! = 1 × 2 × 3 × · · · × (n − 1) × n.)

Solution (Mike Clapper)


Answer: (a, b, n) = (2, 4, 3), (4, 2, 3) and (1, 6, 4).
First consider the case of n ≥ 4, we have 2a + b2 = n! + 14 ≡ 2 (mod 4). If a ≥ 2, then
2a + b2 ≡ 0 or 1 (mod 4), which is a contradiction. So a = 1 and the required equation
becomes b2 = n! + 12. If n ≥ 5, then b2 = n! + 12 ≡ 2 (mod 5), but this is impossible
since b2 ≡ 0, 1 or 4 (mod 5). So the only possibility here is n = 4, which leads to the
solution (a, b, n) = (1, 6, 4).
For n ≤ 3 the problem reduces to a finite case check.

For n = 3, we have 2a + b2 = 20. It suffices to only check a ≤ 4, which yields the


solutions (a, b, n) = (2, 4, 3) and (4, 2, 3).
For n = 2 or 1, we have 2a + b2 = 16 or 15. It suffices to only check a ≤ 3, which
yields no solutions.

Remark. Other modulo arguments can also be used to narrow the problem down to a
finite case check. For example: b2 = n! + 12 and n ≥ 6 can be ruled out by using mod 9
instead of mod 5; the case n ≥ 7 can be ruled out by using mod 7.

1
Mathematics Contests The Australian Scene 2022 | AMOC Senior Contest Solutions | 155
2. Let x1 , x2 , . . . , xn be non-negative real numbers, where n ≥ 2.
Prove that
(x1 + x2 + · · · + xn )2
x1 x2 + x2 x3 + x3 x4 + · · · + xn−1 xn ≤ .
4
(Note that each term on the left-hand side is of the form xi xi+1 for i = 1, 2, . . . , n − 1.)

Solution 1 (Kevin McAvaney)


The required inequality is equivalent to
n
  
4(x1 x2 + x2 x3 + · · · + xn−1 xn ) ≤ x2i + 2xi (xi+1 + xi+2 + · · · + xn )
i=1
n
  
⇐⇒ 0≤ x2i − 2xi xi+1 + 2xi (xi+2 + xi+3 + · · · + xn ) .
i=1

This is true by the follow inequalities:

0 ≤ (x1 − x2 + x3 − x4 + · · · + (−1)n−1 xn )2
n
 2 
= xi + 2xi (−xi+1 + xi+2 − xi+3 + · · · + (−1)i+n xn )
i=1
n
 
= x2i − 2xi xi+1 + 2xi (xi+2 − xi+3 + · · · + (−1)i+n xn )
i=1
n
  
≤ x2i − 2xi xi+1 + 2xi (xi+2 + xi+3 + · · · + xn ) .
i=1

Hence the original inequality is valid.

Solution 2 (Ivan Guo)


We proceed by induction. The base case of n = 2 follows from
(x1 + x2 )2
x 1 x2 ≤ ⇐⇒ 0 ≤ (x1 − x2 )2 .
4
Next assume the result holds for n = k and consider n = k +1. Without loss of generality,
we may assume that x2 ≥ xk (if not we can reverse the sequence). Then we have

x1 x2 + x2 x3 + · · · + xk xk+1 ≤ (x1 + xk+1 )x2 + x2 x3 + · · · + xk−1 xk .

Applying the n = k case to the real numbers (x1 + xk+1 ), x2 , . . . , xk yields


(x1 + xk+1 + x2 + · · · + xk )2
(x1 + xk+1 )x2 + x2 x3 + · · · + xk−1 xk ≤ ,
4
completing the induction.

Solution 3 (Norman Do)


Use the AM-GM inequality as follows.
 (x1 + x3 + · · · ) + (x2 + x4 + · · · )
(x1 + x3 + · · · ) × (x2 + x4 + · · · ) ≤
2
2
Mathematics Contests The Australian Scene 2022 | AMOC Senior Contest Solutions | 156
After squaring both sides, we obtain

(x1 + x2 + · · · + xn )2
(x1 x2 + x2 x3 + · · · + xn−1 xn ) + (extra terms) ≤ .
4
It then suffices to observe that the extra terms are sums of products of the original
non-negative numbers x1 , x2 , . . . , xn .

3
Mathematics Contests The Australian Scene 2022 | AMOC Senior Contest Solutions | 157
3. Let ABC be a triangle with incentre I. Let K be the circle that passes through I and is
tangent to the circumcircle of ABC at A. Suppose that K intersects AB and AC again
at P and Q, respectively.
Prove that the angle bisectors of ∠BP Q and ∠P QC intersect on BC.

Solution 1 (Sampson Wong)


Extend AI to intersect BC at R.

P Q

B C
R

It suffices to show that ∠BP R = ∠RP Q (and similarly ∠P QR = ∠RQC). By dilating


the small circle to the larger one about A, we see that P Q ∥ BC. So it suffices to show
that ∠BP R = ∠P RB = ∠RP Q. Hence it is enough to show that BP = BR.
Now,

∠BP I = ∠P AI + ∠AIP
= ∠IAQ + ∠AQP (AP IQ is cyclic, I is the incentre)
= ∠RAC + ∠ACR (P Q ∥ BC)
= ∠IRB.

In △BP I and △IRB, we have ∠BP I = ∠IRB, ∠IBP = ∠RBI, and IB is common.
Therefore △BP I ∼
= △IRB, and BP = BR as required.

Solution 2 (Ivan Guo)


Let AI meet BC at R and the circumcircle at S. Then since ∠BAS = ∠RAC (angle
bisector) and ∠ASB = ∠ACB (ABSC is cyclic), we must have △BAS ∼ △RAC and
hence AB × AC = AS × AR.

4
Mathematics Contests The Australian Scene 2022 | AMOC Senior Contest Solutions | 158
A

P Q

B C
R

Next, let E be the excentre of △ABC opposite A, so ∠EBI = 90◦ (internal and external
angle bisectors are perpendicular). Then ∠BAE = ∠IAC (angle bisector) and

∠EBA = ∠EBI + ∠IBA


∠CBA
= 90◦ +
2
180◦
− ∠BAC − ∠ACB
= 90◦ +
2
= 180 − ∠IAC − ∠ACI

= ∠CIA.

Hence △BAE ∼ △IAC and AB × AC = AI × AE.


Combining the above we get

AS × AR = AB × AC = AI × AE =⇒ AI/AS = AR/AE.

Hence the dilation that sends ABSC to AP IQ also sends E to R. Thus R is the excentre
of △AP Q opposite A, as required.

Solution 3 (Sally Tsang)

5
Mathematics Contests The Australian Scene 2022 | AMOC Senior Contest Solutions | 159
We will use reverse reconstruction. Let the angles of △ABC at A, B and C be 2α, 2β
and 2γ, respectively. Let the bisectors of ∠BP Q and ∠P QC meet at R. Let the line P R
intersect BC at R′ , and intersect BI at X.
By the alternate segment theorem at the common tangent (or by dilation), ∠AP Q =
∠ABC = 2β. Thus ∠BP R = 90◦ − β and so ∠P XB = 90◦ = ∠R′ XB. Hence △P XB ∼ =
△R′ XB (AAS) and so P X = XR′ .
Since AP IQ is cyclic, ∠P IQ = 180◦ − 2α. Also, since AI is the angle bisector of ∠P AQ,
we have IP = IQ. Angle-chasing in △P RQ yields ∠P RQ = β + γ = 90◦ − α. Hence I
is the circumcentre of △P RQ. Then IX must be the perpendicular bisector of P R. But
we know that P X = XR′ , so R = R′ , which is on BC.

6
Mathematics Contests The Australian Scene 2022 | AMOC Senior Contest Solutions | 160
4. For a real number x, let ⌈x⌉ be the smallest integer greater than or equal to x.
Find all nonempty sets S of positive integers
 such that whenever a and b (not necessarily
a
distinct) are in S, then both ab and b are in S.

Solution 1 (Angelo Di Pasquale)


Answer: S = N+ or S = Sr = {1, r, r2 , r3 , . . .} for some positive integer r.
Taking b = a shows that 1 ∈ S. Note that S = S1 = {1} works. So suppose that |S| > 1.
Let r denote the smallest element of S that is bigger than 1. It follows that Sr =
{1, r, r2 , r3 , . . .} ⊆ S. Note that S = Sr also works. So from here on suppose that S ̸= Sr .
Let s be the smallest element of S not in Sr . Then rn−1 < s < rn for some positive
integer n.
 s 
Consider t = rn−1 ∈ S. Clearly 1 < t ≤ r. By the minimality of r, we have t = r.
Therefore
rn−1 (r − 1) < s < rn .
 n
Consider u = rs ∈ S. We have
   
rn r
1 < u ≤ n−1 = = 2.
r (r − 1) r−1

Thus u = 2 ∈ S. So r = 2, and we have 2n−1 < s < 2n for some positive integer n. This
implies n ≥ 2.
 
If 2n−1 < s ≤ 2n − 2, then 2n−2 < 2s < 2n−1 yields a smaller member of S not in S2 ,
which contradicts the minimality of s. Hence s = 2n − 1.
Here is a little lemma: If k, k + 1 ∈ S and k ≥2 then also k + 3 ∈ S. This is because
2  
k, k + 1 ∈ S implies (k + 1)2 ∈ S which implies (k+1) k
= k + 2 + 1
k
= k + 3 ∈ S.
 n 
Applying the little lemma to s, s+1 ∈ S, we have 2n +2 ∈ S, and so 2 2+2 = 2n−1 +1 ∈ S.
But this contradicts the minimality of s unless s = 3. Thus 3 ∈ S. But now since
1, 2, 3, 4 ∈ S, the little lemma inductively implies that all positive integers are in S.

Solution 2 (Angelo Di Pasquale)


Again note that 1 ∈ S and suppose that |S| > 1.
Let r be the smallest
 sm  member of S that is greater than 1. Let s > 1 be any other member
of S. Then also rn ∈ S for any m, n ∈ Z. Let
 m
s
F (m, n) = logr = m logr s − n.
rn

If logr s is irrational, then the values of F (m, n) form a dense subset of the real number
line. Hence sm /rn form a dense subset of the positive reals. This implies that S = N+ .
If logr s is rational then ru = sv for some relatively prime positive integers u, v. By
examining their prime factorisations, we must have r = tv and s = tu for some integer
t > 1. By Bézout’s lemma, gcd(u, v) can be written as an integer linear combination of u
and v, which implies tgcd(u,v) = t ∈ S. By the minimality of r it follows that t = r, and so
s is a power of r. Since this is true for every s ∈ S, it follows that S = {1, r, r 2 , r3 , . . .}.

7
Mathematics Contests The Australian Scene 2022 | AMOC Senior Contest Solutions | 161
5. Hugo and Maryna are playing a game with n buckets and an infinite pile of stones, where
n is a positive integer. Initially, all buckets are empty.
Hugo and Maryna alternate their turns, with Hugo going first. During Hugo’s turn, he
picks up two stones from the pile and either puts them into one bucket of his choice or
into two separate buckets of his choice. During Maryna’s turn, she chooses one of the n
buckets and empties it back onto the pile.
Hugo wins if, at any point, one of the buckets contains at least 50 stones.
For which values of n does Hugo have a winning strategy?

Solution (William Steinberg)


Answer: n ≥ 247 + 1.
We assign each bucket a rating of 2i if it contains i stones where i > 0, and a rating of 0
if it is empty. We claim Maryna can ensure the sum of ratings stays strictly less than 2n
after each of her moves.

If Hugo puts 2 stones into the same bucket, then Maryna can empty this bucket,
thus maintaining the sum below 2n.
If Hugo chooses two buckets with ratings a and b with a ≥ b > 0, then the new ratings
are 2a and 2b. Maryna can empty the bucket with rating 2a. Since 2b ≤ a + b, the
sum stays below 2n.
If a > b = 0, then the new ratings are 2a and 2. Again Maryna can empty the
bucket with rating 2a. Since 2 ≤ a, the sum stays below 2n.
If a = b = 0, then Hugo’s move increases the sum by 4.
– If any bucket has at least 2 stones, Maryna can empty this bucket, decreasing
the sum by at least 4.
– If no bucket has more than 1 stone, Maryna can empty any bucket, and the sum
of ratings will be below 2n.

Thus the claim is proven.


For n ≤ 247 Maryna can ensure the sum of rating remains less than 248 after each of her
turns. So there is no bucket with 48 stones or more after her turn and thus no bucket
can ever reaches 50 stones.
We now show that for n = 247 +1 Hugo can in fact win. Initially Hugo always chooses two
empty buckets. Since Hugo chooses two buckets each turn and Maryna can only empty
one, eventually after one of Hugo’s turns every bucket will have 1 stone. After Maryna’s
next turn there will be exactly n − 1 = 247 buckets with 1 stone.
We now prove a lemma. If there are 2k buckets with r stones each before Hugo’s turn,
then Hugo can choose buckets in such a way so that after k rounds there are k buckets
with r+1 stones each. To do this, he pairs up the 2k buckets and each turn he chooses two
buckets from one pair, until all 2k are chosen after k rounds. In the meantime, Maryna
can only empty at most k of these 2k buckets, so at least k buckets will survive with r + 1
stones each. So the lemma is proven.
Hugo repeatedly applies this lemma until there is at least 1 bucket with 48 stones at the
start of his turn. At this point he can win by simply putting 2 more stones into this
bucket.

8
Mathematics Contests The Australian Scene 2022 | AMOC Senior Contest Solutions | 162
2022 AMOC Senior Contest
Results

Name School Year


Perfect Score and Gold
William Cheah Scotch College VIC 8
Gold
Juan Lucas Callo North Sydney Boys' High School NSW 11
Eunsu Choi Brisbane State High School QLD 11
Leo Li James Ruse Agricultural High School NSW 11
Xiangyue (Laura) Nan Fintona Girls' School VIC 9
Levi Isaak Pesin Pesin Home School SA 8
Dylan (Ky-Dang) Phung Yarra Valley Grammar School VIC 10
Zian Shang Scotch College VIC 11
Rohan Timmaraju Sydney Grammar School NSW 11
Christopher Wei Han Tran The University High School VIC 11
Justin Tran Sydney Grammar School NSW 9
Leonardo Wang Scotch College VIC 11
Cloris Xu Baulkham Hills High School NSW 10
Dongqiang (Ryan) Yang John Monash Science School VIC 11
Eason Yang Trinity Grammar School VIC 9
Silver
Xiaohan (Taiga) Chen Yarra Valley Grammar School VIC 11
Michael (Zixiang) Jiang Melbourne Grammar School VIC 11
Crystal (Yunxin) Jin James Ruse Agricultural High School NSW 11
Jerry Li James Ruse Agricultural High School NSW 11
William Liu The Scots College NSW 11
Jayden Pan Shore School NSW 8
Sizhe Pan James Ruse Agricultural High School NSW 11
Joshua Prince St Aloysius' College NSW 11
Atharva Sathe Perth Modern School WA 10
Daniel Seo James Ruse Agricultural High School NSW 11
Harrison Su James Ruse Agricultural High School NSW 10
Arthur Sun Scotch College VIC 11
Ben Szabo-Virag Matthew Flinders Anglican College QLD 10
Arlo Taylor Marist Regional College TAS 11
Polar Xiong North Sydney Boys' High School NSW 9
Iris Xu Baulkham Hills High School NSW 10
Ethan Yap Perth Modern School WA 10
Zean Zhang Christ Church Grammar School WA 11
Bronze
Anubhav Ammangi Redeemer Baptist School NSW 10
Joel Bariss Perth Modern School WA 10
George Chen James Ruse Agricultural High School NSW 9
Jordan Chi Scotch College VIC 11
Adam El-Fayoumi Willetton Senior High School WA 10

Mathematics Contests The Australian Scene 2022 | AMOC Senior Contest Results | 163
Name School Year
Bronze
Matilda Grant A. B. Paterson College QLD 9
Yaohong (Sherry) Jiang North Sydney Girls' High School NSW 11
Ian Lai Queensland Academy for Science, Mathematics & Technology QLD 9
Elizabeth (Xuan Xian) Lee Queensland Academies: Health Science Campus QLD 10
Amber (Yanfei) Li Pymble Ladies' College NSW 8
Jiamu Li Scotch College VIC 10
Jingni Liao Knox Grammar School NSW 10
Jason Miao Knox Grammar School NSW 10
Quan Nguyen Anglican Church Grammar School QLD 10
Yangyang (Qiyang) Ning James Ruse Agricultural High School NSW 9
Witter Onggara Scotch College VIC 9
Alex Qiu James Ruse Agricultural High School NSW 10
Ee Lin Saw Somerville House QLD 9
Mohijit Singh Baulkham Hills High School NSW 9
Miranda Sui Presbyterian Ladies' College VIC 10
Johnson Tang The King's School NSW 10
Oliver Taylor Caulfield Grammar School - Caulfield Campus VIC 10
Yang (Andrew) Wang Melbourne High School VIC 10
Liah (Zizhen) Wu Presbyterian Ladies' College VIC 10
Jerry (Qihao) Xia Scotch College VIC 8
Qixuan (Eric) Xia Hale School WA 10
Andrew Yan Smith's Hill High School NSW 11
Beikun (Max) Yan Anglican Church Grammar School QLD 10
Jayee Zhang Scotch College VIC 7
Yitong Zheng Indooroopilly State High School QLD 10
Joseph Zhu Varsity College QLD 7
Honourable Mention
Dylan Aberdeen St Peter's College QLD 11
Alec Artemov Scotch College VIC 9
Jeremy Arulampalam Glenunga International High School SA 8
Max Beckmann St Joseph's College Gregory Terrace QLD 10
Casper Cai St Peter's College SA 10
Max Frederick Campbell Shenton College WA 10
Albert Chen Melbourne Grammar School VIC 10
Sunguo (Jerry) Chen Kelvin Grove State College QLD 10
Beatrice Chong Perth Modern School WA 9
Chengji (Tommy) Chu Scotch College VIC 9
Inez Crosby North Sydney Girls' High School NSW 10
Yvonne Cui Presbyterian Ladies' College WA 11
Joshua (Kezhuo) Deng Scotch College VIC 8
Evan Hee Rossmoyne Senior High School WA 11
Bright Hu Knox Grammar School NSW 7
Anthony Ivanov North Sydney Boys' High School NSW 10
Jerry Jin Camberwell Grammar School VIC 10
Aidan Lim Christ Church Grammar School WA 9

Mathematics Contests The Australian Scene 2022 | AMOC Senior Contest Results | 164
Name School Year
Thomas Lin Canberra Grammar School ACT 11
Hangzhi (Kevin) Liu Haileybury College VIC 6
Callum Loughlin Queensland Academy for Science, Mathematics & Technology QLD 11
Maxwell MacDougall Narrabundah College ACT 11
Zavier Noonan Bunbury Catholic College WA 11
Kathryn Petersen Brisbane Girls' Grammar School QLD 10
Jason Phan James Ruse Agricultural High School NSW 11
Kathlin (Si Lin) Qiu Lauriston Girls' School (Armadale campus) VIC 10
Nicholas Shuttleworth Narrabundah College ACT 11
Frank Tang Scotch College VIC 10
Cunmeng (Matthew) Wang Scotch College VIC 7
Yiheng You James Ruse Agricultural High School NSW 10
Victor Yuan James Ruse Agricultural High School NSW 9
Ben Zhang St Joseph's College, Gregory Terrace QLD 10
Leon Zhou James Ruse Agricultural High School NSW 8

Mathematics Contests The Australian Scene 2022 | AMOC Senior Contest Results | 165
2022 AMOC Senior Contest
Statistics

Score Distribution/Problem

Mark/Problem Q1 Q2 Q3 Q4 Q5

0 5 51 63 74 55

1 1 25 12 23 31

2 3 5 9 7 31

3 4 2 4 7 3

4 4 6 4 1 1

5 4 0 0 4 1

6 21 4 1 5 1

7 88 37 37 9 7

Average 6.1 2.7 2.5 1.3 1.3

Mathematics Contests The Australian Scene 2022 | AMOC Senior Contest Statistics | 166
Origin of Some Questions

AMOC Senior Contest 2022


Question 1 was submitted by William Steinberg.

Question 2 was submitted by Kevin McAvaney.

Question 3 was submitted by Sampson Wong.

Question 4 was submitted by Angelo Di Pasquale.

Question 5 was submitted by William Steinberg.

Australian Mathematical Olympiad 2022


Question 1 was submitted by Mike Clapper.

Question 2 was submitted by Angelo Di Pasquale.

Question 3 was submitted by William Steinberg.

Question 4 was submitted by Jongmin Lim.

Question 5 was submitted by Kevin McAvaney.

Question 6 was submitted by Ian Wanless.

Question 7 was submitted by Angelo Di Pasquale.

Question 8 was submitted by Ethan Tan.

Asian Pacific Mathematics Olympiad 2022


Question 1 was composed by Alan Offer and submitted by the AMOC Senior Problems Committee.

Question 4 was composed by Trevor Tao and submitted by the AMOC Senior Problems Committee.

European Girls’ Mathematical Olympiad 2022


Question 6 was composed by Ethan Tan and submitted by the AMOC Senior Problems Committee.

International Mathematical Olympiad 2022


G8 on the IMO Shortlist was composed by William Steinberg and submitted by the AMOC Senior

Problems Committee.

Mathematics Contests The Australian Scene 2022 | Origin of Some Questions | 167
Honour Roll
Honour Roll 1979–2022

Because of changing titles and affiliations, the most senior title achieved and later affiliations are
generally used, except for the Interim committee, where they are listed as they were at the time.

Interim Committee 1979–1980


Mr P J O'Halloran Canberra College of Advanced Education, Chair
Prof A L Blakers University of Western Australia
Dr J M Gani Australian Mathematical Society
Prof B H Neumann Australian National University
Prof G E Wall University of Sydney
Mr J L Williams University of Sydney

Australian Mathematical Olympiad Committee


The Australian Mathematical Olympiad Committee was founded at a meeting of the Australian Academy of
Science at its meeting of 2–3 April 1980.
* Denotes Executive Position

Chair*
Prof B H Neumann Australian National University 7 years; 1980–1986
Prof G B Preston Monash University 10 years; 1986–1995
Prof A P Street University of Queensland 6 years; 1996–2001
Prof C Praeger University of Western Australia 18 years; 2002–2019
Dr N Do Monash University 2 years; 2020–2022

Deputy Chair*
Prof P J O'Halloran University of Canberra 15 years; 1980–1994
Prof A P Street University of Queensland 1 year; 1995
Prof C Praeger University of Western Australia 7 years; 1996–2001, 2020
Assoc Prof D Hunt University of New South Wales 14 years; 2002–2015
Prof Andrew Hassell Australian National University 3 years; 2016–2018
Dr N Do Monash University 1 year; 2019

Executive Director*
Prof P J O'Halloran University of Canberra 15 years; 1980–1994
Prof P J Taylor University of Canberra 18 years; 1994–2012
Adj Prof M Clapper University of Canberra 4 years; 2013–2016
Mr N Ford University of Canberra 5 years; 2017–2022

AMT Mathematician
Mr M Clapper Australian Maths Trust 5 years; 2017–2022

Secretary
Prof J C Burns Australian Defence Force Academy 9 years; 1980–1988
Vacant 4 years; 1989–1992
Mrs K Doolan Victorian Chamber of Mines 6 years; 1993–1998

Treasurer*
Prof J C Burns Australian Defence Force Academy 8 years; 1981–1988
Prof P J O'Halloran University of Canberra 2 years; 1989–1990
Ms J Downes CPA 5 years; 1991–1995
Dr P Edwards Monash University 8 years; 1995–2002
Prof M Newman Australian National University 6 years; 2003–2008
Dr P Swedosh The King David School, Victoria 13 years; 2009–2022

Director of Mathematics for Young Australians*


Mr J B Henry Deakin University 17 years; 1990–2006
Dr K McAvaney Deakin University 16 years; 2006–2022

Mathematics Contests The Australian Scene 2022 | Honour Roll 1979–2022 | 169
Chair, Senior Problems Committee
Prof B C Rennie James Cook University 1 year; 1980
Mr J L Williams University of Sydney 6 years; 1981–1986
Assoc Prof H Lausch Monash University 27 years; 1987–2013
Dr N Do Monash University 6 years; 2014–2019
Dr I Guo Monash University, Victoria 2 years; 2020–2022

Director of Training*
Mr J L Williams University of Sydney 7 years; 1980–1986
Mr G Ball University of Sydney 3 years; 1987–1989
Dr D Paget University of Tasmania 6 years; 1990–1995
Dr M Evans Scotch College, Victoria 3 months; 1995
Assoc Prof D Hunt University of New South Wales 5 years; 1996–2000
Dr A Di Pasquale University of Melbourne 21 years; 2001–2022

IMO Team Leader


Mr J L Williams University of Sydney 5 years; 1981–1985
Assoc Prof D Hunt University of New South Wales 9 years; 1986, 1989, 1990, 1996–2001
Dr E Strzelecki Monash University 2 years; 1987, 1988
Dr D Paget University of Tasmania 5 years; 1991–1995
Mr I Guo University of Sydney 1 year; 2011
Dr A Di Pasquale University of Melbourne 19 years; 2002–2010, 2012–2022

IMO Deputy Team Leader


Prof G Szekeres University of New South Wales 2 years; 1981–1982
Mr G Ball University of Sydney 7 years; 1983–1989
Dr D Paget University of Tasmania 1 year; 1990
Mr J Graham University of Sydney 3 years; 1991–1993
Dr M Evans Scotch College, Melbourne 3 years; 1994–1996
Dr A Di Pasquale University of Melbourne 5 years; 1997–2001
Mr D Mathews University of Melbourne 3 years; 2002–2004
Mr N Do University of Melbourne 4 years; 2005–2008
Mr I Guo University of Sydney 4 years; 2009–2010, 2012–2013
Mr G White University of Sydney 1 year; 2011
Mr A Elvey Price Melbourne University 5 years; 2014–2018
Mr S Wong University of Sydney 3 years; 2019–2022

EGMO Team Leader


Ms T Shaw SCEGGS Darlinghurst 4 years; 2018–2022

EGMO Deputy Team Leader


Miss M Chen Melbourne University 4 years; 2018–2022

State Directors
Australian Capital Territory
Prof M Newman Australian National University 1 year; 1980
Mr D Thorpe ACT Department of Education 2 years; 1981–1982
Dr R A Bryce Australian National University 7 years; 1983–1989
Mr R Welsh Canberra Grammar School 1 year; 1990
Mrs J Kain Canberra Grammar School 5 years; 1991–1995
Mr J Carty ACT Department of Education 17 years; 1995–2011
Mr J Hassall Burgmann Anglican School 2 years; 2012–2013
Dr C Wetherell Australian Maths Trust 8 years; 2014–2022

New South Wales


Dr M Hirschhorn University of New South Wales 1 year; 1980
Mr G Ball University of Sydney 16 years; 1981–1996
Dr W Palmer University of Sydney 19 years; 1997–2015
A/Prof D Daners University of Sydney 1 year; 2016
Dr D Badziahin University of Sydney 5 years; 2017–2022

Mathematics Contests The Australian Scene 2022 | Honour Roll 1979–2022 | 170
Northern Territory
Dr I Roberts Charles Darwin University 9 years; 2013–2022

Queensland
Dr N H Williams University of Queensland 21 years; 1980–2000
Dr G Carter Queensland University of Technology 10 years; 2001–2010
Dr V Scharaschkin University of Queensland 4 years; 2011–2014
Dr A Offer Queensland 5 years; 2015–2019
Dr B Gray Queensland 3 years; 2019–2022

South Australia
Mr K Hamann Rostrevor 19 years; 1980–1982, 1991–2005; 2013
Mr V Treilibs SA Department of Education 8 years; 1983–1990
Dr M Peake Adelaide 7 years; 2006–2012
Mr D Martin Adelaide 5 years; 2014–2018
Mr M Bammann Adelaide 3 years; 2019–2022

Tasmania
Mr J Kelly Tasmanian Department of Education 8 years; 1980–1987
Dr D Paget University of Tasmania 8 years; 1988–1995
Mr W Evers St Michael's Collegiate School 9 years; 1995–2003
Dr K Dharmadasa University of Tasmania 17 years; 2004–2020

Victoria
Dr D Holton University of Melbourne 3 years; 1980–1982
Mr B Harridge Melbourne High School 1 year; 1982
Ms J Downes CPA 6 years; 1983–1988
Mr L Doolan Melbourne Grammar School 9 years; 1989–1998
Dr P Swedosh The King David School 24 years; 1998–2022

Western Australia
Dr N Hoffman WA Department of Education 3 years; 1980–1982
Assoc Prof P Schultz University of Western Australia 14 years; 1983–1988,
1991–1994, 1996–1999
Assoc Prof W Bloom Murdoch University 2 years; 1989–1990
Dr E Stoyanova WA Department of Education 7 years; 1995, 2000–2005
Dr G Gamble University of Western Australia 16 years; 2006–2022

Editor
Prof P J O'Halloran University of Canberra 1 year; 1983
Dr A W Plank University of Southern Queensland 11 years; 1984–1994
Dr A Storozhev Australian Maths Trust 15 years; 1994–2008

Editorial Consultant
Dr O Yevdokimov University of Southern Queensland 13 years; 2009–2022

Other Members of AMOC (showing organisations represented where applicable)


Mr W J Atkins Australian Mathematics Foundation 18 years; 1995–2012
Mr J Bament O'Loughlin Catholic College 2 years; 2020–2022
Mr M Bammann South Australia 1 year; 2018
Dr S Britton University of Sydney 8 years; 1990–1998
Prof G Brown Australian Academy of Science 10 years; 1980, 1986–1994
Dr R A Bryce Australian Mathematical Society 8 years; 1991–1998
Mathematics Challenge for Young Australians 14 years; 1999–2012
Mr G Cristofani Department of Education and Training 2 years; 1993–1994
Mr J Dean Tasmania 2 years; 2017–2018
Ms L Davis IBM Australia 4 years; 1991–1994
Dr W Franzsen Australian Catholic University 9 years; 1990–1998
Dr J Gani Australian Mathematical Society 1 year; 1980
Assoc Prof T Gagen ANU AAMT Summer School 6 years; 1993–1998
Ms P Gould Department of Education and Training 2 years; 1995–1996
Mr K Hamann South Australia 4 years; 2015–2018
Mr N Hoang Western Australia 4 years; 2018–2022
Prof G M Kelly University of Sydney 6 years; 1982–1987
Prof R B Mitchell University of Canberra 5 years; 1991–1995

Mathematics Contests The Australian Scene 2020 | Honour Roll 1979–2022 | 171
Ms A Nakos Mathematics for Young Australians 19 years; 2003–2022
Mr S Neal Department of Education and Training 4 years; 1990–1993
Prof M Newman Australian National University 13 years; 1986–1998
Mathematics Challenge for Young Australians 21 years; 1999–2019,
(Treasurer during the interim) 2003–2008
Prof R B Potts University of Adelaide 1 year; 1980
Mr H Reeves Australian Association of Maths Teachers/ 16 years; 1988–1998,
Australian Mathematics Foundation 2014–2018
Mr N Reid IBM Australia 3 years; 1988–1990
Mr M Roberts Tasmania 4 years; 2014–2017
Mr R Smith Telecom Australia 5 years; 1990–1994
Ms J Sprakel Mathematics Challenge for Young Australians 9 years; 2012–2020
Prof P J Taylor Australian Mathematics Foundation 6 years; 1990–1994, 2013
Dr S Thornton University of New South Wales 2 years; 2020–2022
Prof N S Trudinger Australian Mathematical Society 3 years; 1986–1988
Assoc Prof I F Vivian University of Canberra 1 year; 1990
Dr M W White IBM Australia 9 years; 1980–1988

Associate Membership (inaugurated in 2000)


Ms S Britton 17 years; 2000–2016
Dr M Evans 22 years; 2000–2022
Dr W Franzsen 17 years; 2000–2016
Prof T Gagen 17 years; 2000–2016
Mr H Reeves 15 years; 2000–2014
Mr G Ball 13 years; 2004–2016
Mr I Guo 2 years; 2018–2019

Maths for Young Australians


Maths Challenge Committee
Dr K McAvaney Deakin University (Director) 16 years; 2006–2022,
Member 2 years; 2005–2006
Mr B Henry Victoria (Director) 17 years; 1990–2006
Member 15 years; 2007–2022
Prof P J O'Halloran University of Canberra 5 years; 1990–1994
Dr A Albrecht University of South Australia 3 years; 2019–2022
Dr R A Bryce Australian National University 23 years; 1990–2012
Mr M Clapper Australian Maths Trust 9 years; 2013–2022
Ms L Corcoran Australian Capital Territory 3 years; 1990–1992
Ms B Denney New South Wales 12 years; 2010–2022
Mr J Dowsey University of Melbourne 8 years; 1995–2002
Mr A R Edwards Department of Education, Queensland 32 years; 1990–2022
Dr M Evans Scotch College, Victoria 6 years; 1990–1995
Assoc Prof H Lausch Monash University 24 years; 1990–2013
Mrs L Mottershead New South Wales 30 years; 1992–2022
Miss A Nakos Temple Christian College, South Australia 29 years; 1993–2022
Dr M Newman Australian National University 32 years; 1990–2022
Ms F Peel St Peter's College, South Australia 2 years; 1999–2000
Dr I Roberts Charles Darwin University, NT 9 years; 2013–2022
Ms T Shaw SCEGGS, NSW 9 years; 2013–2022
Ms K Sims Blue Mountains Grammar School 23 years; 1999–2022
Ms J Sprakel Australian Mathematical Sciences Institute 19 years; 2002–2020
Dr A Storozhev Attorney General's Department 23 years; 1994–2016
Prof P Taylor Australian Maths Trust 20 years; 1995–2014
Mrs A Thomas New South Wales 18 years; 1990–2007
Dr S Thornton Exec. Dir. reSolve 24 years; 1998–2022
Miss G Vardaro Wesley College, Victoria 28 years; 1993–2006, 2008–2022
Dr C Wetherell Australian Maths Trust 5 years; 2017–2022

Visiting members of Challenge Committee


Prof E Barbeau University of Toronto, Canada 1991, 2004, 2008
Prof G Berzsenyi Rose Hulman Institute of Technology, USA 1993, 2002

Mathematics Contests The Australian Scene 2022 | Honour Roll 1979–2022 | 172
Dr L Burjan Department of Education, Slovakia 1993
Dr V Burjan Institute for Educational Research, Slovakia 1993
Mrs A Ferguson Canada 1992
Prof B Ferguson University of Waterloo, Canada 1992, 2005
Dr D Fomin St Petersburg State University, Russia 1994
Prof F Holland University College, Cork, Ireland 1994
Dr A Liu University of Alberta, Canada 1995, 2006, 2009
Prof Q Zhonghu Academy of Science, China 1995
Dr A Gardiner University of Birmingham, United Kingdom 1996
Prof P H Cheung Hong Kong 1997
Prof R Dunkley University of Waterloo, Canada 1997
Dr S Shirali India 1998
Mr M Starck New Caledonia 1999
Dr R Geretschlager Austria 1999, 2013
Dr A Soifer United States of America 2000
Prof M Falk de Losada Colombia 2000
Mr H Groves United Kingdom 2001
Prof J Tabov Bulgaria 2001, 2010
Prof A Andzans Latvia 2002
Prof Dr H-D Gronau University of Rostock, Germany 2003
Prof J Webb University of Cape Town, South Africa 2003, 2011
Mr A Parris Lynwood High School, Christchurch, NZ 2004
Dr A McBride University of Strathclyde, United Kingdom 2007
Prof P Vaderlind Stockholm University, Sweden 2009, 2012
Prof A Jobbings United Kingdom 2014
Assoc Prof D Wells United States of America 2015
Emer. Fellow P Neumann Queen's College Oxford UK 2016
Dr C Jones Alabama State University, USA 2018, 2020, 2022
Dr P Couch Lamar University, USA 2018, 2020, 2022
Mr S Blasberg West Valley College, USA 2019, 2020, 2022

Past and present moderators for Maths Challenge


Mr W Akhurst New South Wales
Ms N Andrews ACER, Camberwell, Victoria
Dr Z Bacskai Australian Capital Territory
Mr L Bao Leopold Primary School, Victoria
Prof E Barbeau University of Toronto, Canada
Mr M Bamman South Australia
Mr R Blackman Victoria
Dr S Blasberg USA
Ms J Breidahl St Paul's Woodleigh, Victoria
Ms S Brink Glen Iris, Victoria
Prof J C Burns Australian Defence Force Academy
Mr J Carty ACT Department of Education, Australian Capital Territory
Mr A Canning Queensland
Mrs F Cannon New South Wales
Dr E Casling Australian Capital Territory
Mr P Christensen Queensland
Mr B Darcy South Australia
Ms B Denney New South Wales
Mr J Dowsey Victoria
Mr S Ewington Sydney Grammar School, New South Wales
Br K Friel Trinity Catholic College, New South Wales
Dr D Fomin St Petersburg University, Russia
Mrs P Forster Penrhos College, Western Australia
Mr T Freiberg Queensland
Mr W Galvin University of Newcastle, New South Wales
Mr S Gardiner University of Sydney, New South Wales
Mr M Gardner North Virginia, USA
Ms P Graham Tasmania
Mr B Harridge University of Melbourne
Ms J Hartnett Queensland
Mr G Harvey Australian Capital Territory

Mathematics Contests The Australian Scene 2022 | Honour Roll 1979–2022 | 173
Ms I Hill South Australia
Ms N Hill Victoria
Dr N Hoffman Edith Cowan University, Western Australia
Prof F Holland University College, Cork, Ireland
Ms R Humberstone Scotch College, Vixctoria
Mr D Jones Coffs Harbour High School, New South Wales
Ms R Jorgensen Australian Capital Territory
Dr T Kalinowski University of New England, New South Wales
Mr L Katz Victoria
Assoc Prof H Lausch Victoria
Mr J Lawson St Pius X School, Chatswood, New South Wales
Mr R Longmuir China
Dr K McAvaney Deakin University
Ms K McAsey Penleigh and Essendon Grammar School, Victoria
Ms J McIntosh University of Melbourne
Ms N McKinnon Victoria
Ms T McNamara Victoria
Mr G Meiklejohn Queensland School Curriculum Council, Queensland
Mr M O'Connor AMSI, Victoria
Mr T Ogawa New South Wales
Mr J Oliver Northern Territory
Mr S Palmer New South Wales
Dr W Palmer University of Sydney
Mr A Peck Mt Carmel College, Tasmania
Mr G Pointer Marryatville High School, South Australia
Prof H Reiter University of North Carolina, USA
Mr M Richardson Yarraville Primary School, Victoria
Mr J Rizos Victoria
Dr F Rose Victoria
Mr G Samson Nedlands Primary School, Western Australia
Mr J Sattler Parramatta High School, New South Wales
Mr A Saunder Victoria
Mr W Scott Seven Hills West Public School, New South Wales
Mr R Shaw Hale School, Western Australia
Ms T Shaw New South Wales
Dr B Sims University of Newcastle
Dr H Sims Victoria
Ms K Sims Blue Mountains Grammar School, New South Wales
Prof J Smit The Netherlands
Ms C Smith St Paul's School, Queensland
Ms D Smith Inquisitive Minds, New South Wales
Mrs M Spandler New South Wales
Mr G Spyker Curtin University
Ms C Stanley Queensland Curriculum & Assessment Authority (Primary), Queensland
Ms R Stone Plympton Primary School, South Australia
Dr E Strzelecki Monash University, Victoria
Dr M Sun New South Wales
Mr P Swain Ivanhoe Girls Grammar School, Victoria
Dr P Swedosh The King David School, Victoria
Prof J Tabov Academy of Sciences, Bulgaria
Mrs A Thomas New South Wales
Ms K Trudgian Queensland
Ms J Vincent Melbourne Girls Grammar School, Victoria
Mr J Watts New South Wales
Prof J Webb University of Capetown, South Africa
Assoc Prof D Wells North Carolina, USA

Maths Enrichment Development


Mathematics Enrichment Series Committee – Development Team (1992–1995)
Mr B Henry Victoria (Chairman)
Prof P O'Halloran University of Canberra (Director)
Mr G Ball University of Sydney

Mathematics Contests The Australian Scene 2019 | Honour Roll 1979–2022 | 174
Dr M Evans Scotch College, Victoria
Mr K Hamann South Australia
Assoc Prof H Lausch Monash University
Dr A Storozhev Australian Maths Trust, Australian Capital Territory

Pólya Enrichment Series – Development Team (1992–1995)


Mr G Ball University of Sydney (Editor)
Mr K Hamann South Australia (Editor)
Prof J Burns Australian Defence Force Academy
Mr J Carty Merici College, Australian Capital Territory
Dr H Gastineau-Hill University of Sydney
Mr B Henry Victoria
Assoc Prof H Lausch Monash University
Prof P O'Halloran University of Canberra
Dr A Storozhev Australian Maths Trust, Australian Capital Territory

Pólya Enrichment Series – Development Team (2013–2015)


Adj Prof M Clapper Australian Maths Trust (Editor)
Dr R Atkins New Zealand
Dr A Di Pasquale University of Melbourne
Dr R Geretschlager Austria
Dr D Mathews Monash University
Dr K McAvaney Australian Maths Trust

Euler Enrichment Series – Development Team (1992–1995)


Dr M Evans Scotch College, Victoria (Editor)
Mr B Henry Victoria (Editor)
Mr L Doolan Melbourne Grammar School, Victoria
Mr K Hamann South Australia
Assoc Prof H Lausch Monash University
Prof P O'Halloran University of Canberra
Mrs A Thomas Meriden School, New South Wales

Gauss Enrichment Series – Development Team (1993–1995)


Dr M Evans Scotch College, Victoria (Editor)
Mr B Henry Victoria (Editor)
Mr W Atkins University of Canberra
Mr G Ball University of Sydney
Prof J Burns Australian Defence Force Academy
Mr L Doolan Melbourne Grammar School, Victoria
Mr A Edwards Mildura High School, Victoria
Mr N Gale Hornby High School, New Zealand
Dr N Hoffman Edith Cowan University
Prof P O'Halloran University of Canberra
Dr W Pender Sydney Grammar School, New South Wales
Mr R Vardas Dulwich Hill High School, New South Wales

Noether Enrichment Series – Development Team (1994–1995)


Dr M Evans Scotch College, Victoria (Editor)
Dr A Storozhev Australian Maths Trust, Australian Capital Territory (Editor)
Mr B Henry Victoria
Dr D Fomin St Petersburg University, Russia
Mr G Harvey New South Wales

Newton Series – Development Team (2001–2002)


Mr B Henry Victoria (Editor)
Mr J Dowsey University of Melbourne
Mrs L Mottershead New South Wales
Ms G Vardaro Annesley College, South Australia

Mathematics Contests The Australian Scene 2022 | Honour Roll 1979–2022 | 175
Ms A Nakos Temple Christian College, South Australia
Mrs A Thomas New South Wales

Dirichlet Series – Development Team (2001–2003)


Mr B Henry Victoria (Editor)
Mr A Edwards Ormiston College, Queensland
Ms A Nakos Temple Christian College, South Australia
Mrs L Mottershead New South Wales
Mrs K Sims Chapman Primary School, Australian Capital Territory
Mrs A Thomas New South Wales

Ramanujan Enrichment Series – Development Team (2014–2016)


Mr B Henry Victoria (Editor)
Adj Prof M Clapper Australian Maths Trust
Ms A Nakos Temple Christian College, South Australia
Mr A Edwards Department of Education, Queensland
Dr K McAvaney Australian Maths Trust
Dr I Roberts Charles Darwin University, NT

Australian Intermediate Mathematics Olympiad Committee


Dr K McAvaney Deakin University (Chair) 15 years; 2007–2022
Mr M Clapper Australian Maths Trust 8 years; 2014–2022
Mr J Dowsey University of Melbourne 18 years; 1999–2016
Dr M Evans Australian Mathematical Sciences Institute 23 years; 1999–2022
Mr B Henry Victoria (Chair) 8 years; 1999–2006
Member 15 years; 2007–2022
Assoc Prof H Lausch Monash University 17 years; 1999–2015
Mr R Longmuir China 2 years; 1999–2000
Dr D Mathews Monash University 5 years; 2017–2022
Dr D Treeby Presbyterian Ladies' College, Victoria 2 years; 2020–2022

AMOC Senior Problems Committee


Current members
Mr M Clapper Australian Maths Trust 9 years; 2013–2022
Miss M Chen Melbourne University 3 years; 2019–2022
Dr A Devillers University of Western Australia 6 years; 2016–2022
Dr A Di Pasquale University of Melbourne 21 years; 2001–2022
Dr N Do Monash University (Chair) 6 years; 2014–2019
(Member) 13 years; 2003–2013; 2020–2022
Dr I Guo Monash University (Chair) 2 years; 2020–2022
(Member) 12 years; 2008–2019
Dr K McAvaney Deakin University 26 years; 1996–2022
Dr D Mathews Monash University 21 years; 2001–2022
Dr A Offer Queensland 10 years; 2012–2022
Mr A Elvey Price Melbourne University 4 years; 2018–2022
Dr C Rao Victoria 21 years; 1999–2020
Dr B Saad Monash University 28 years; 1994–2022
Ms T Shaw SCEGGS Darlinghurst 4 years; 2018–2022
Dr I Wanless Monash University 22 years; 2000–2022
Dr C Wetherell Australian Maths Trust 3 years; 2019–2022
Mr S Wong University of Sydney 2 years; 2020–2022

Previous members
Mr G Ball University of Sydney 16 years; 1982–1997
Mr M Brazil LaTrobe University 5 years; 1990–1994
Dr M S Brooks University of Canberra 8 years; 1983–1990
Dr G Carter Queensland University of Technology 10 years; 2001–2010
Dr M Evans Australian Mathematical Sciences Institute 28 years; 1990–2016
Mr J Graham University of Sydney 1 year; 1992

Mathematics Contests The Australian Scene 2022 | Honour Roll 1979–2022 | 176
Dr M Herzberg Telecom Australia 1 year; 1990
Assoc Prof D Hunt University of New South Wales 29 years; 1986–2014
Dr J Kupka Monash University 16 years; 2003–2018
Assoc Prof H Lausch Monash University (Chair) 28 years; 1987–2013
(Member) 2 years; 2014–2015
Dr L Kovacs Australian National University 5 years; 1981–1985
Dr D Paget University of Tasmania 7 years; 1989–1995
Prof P Schultz University of Western Australia 8 years; 1993–2000
Dr J Simpson Curtin University 20 years; 1999–2018
Dr L Stoyanov University of Western Australia 5 years; 2001–2005
Dr E Strzelecki Monash University 5 years; 1986–1990
Dr E Szekeres University of New South Wales 7 years; 1981–1987
Prof G Szekeres University of New South Wales 7 years; 1981–1987
Emer Prof P Taylor Australian Maths Trust 1 year; 2013
Dr N H Williams University of Queensland 20 years; 1981–2000

Mathematics School of Excellence


Dr S Britton University of Sydney (Coordinator) 2 years; 1990–1991
Mr L Doolan Melbourne Grammar (Coordinator) 6 years; 1992, 1993–1997
Mr W Franzsen Australian Catholic University (Coordinator) 2 years; 1990–1991
Dr D Paget University of Tasmania (Director) 6 years; 1990–1995
Assoc Prof D Hunt University of New South Wales (Director) 5 years; 1996–2000
Dr A Di Pasquale University of Melbourne (Director) 20 years; 2001–2022

International Mathematical Olympiad Selection School


Mr G Ball University of Sydney (Director) 6 years; 1984–1989
Mr L Doolan Melbourne Grammar (Coordinator) 3 years; 1989–1991
Dr S Britton University of Sydney (Coordinator) 7 years; 1992–1998
Mr W Franzsen Australian Catholic University (Coordinator) 8 years; 1992–1996; 1999–2001
Dr D Paget University of Tasmania (Director) 6 years; 1990–1995
Mr J L Williams University of Sydney (Director) 2 years; 1982–1983
Assoc Prof D Hunt University of New South Wales (Director) 5 years; 1996–2000
Dr A Di Pasquale University of Melbourne (Director) 21 years; 2001–2022

Mathematics Contests The Australian Scene 2022 | Honour Roll 1979–2022 | 177

You might also like